SlideShare a Scribd company logo
1 of 1061
Lesson1–IntroductiontoCBAP®Certification
CBAP®Exam Preparation Course
CBAP, CCBA and BABOK are registered certification marks owned by International Institute of Business Analysis.
WHAT’S IN IT FOR ME
About IIBA ®
Benefits of IIBA® Professional
Certification
Criteria for CBAP® certification
The exam application process
The exam outline
Recertification requirements
ABOUT IIBA®
IIBA®
Defines standards for business
analysis
Identifies the skills necessary
to be effective in the role of a
business analyst
Recognizes business analysis
professionals
International Institute of Business Analysis
Through the BABOK® Guide
Through the IIBA Business
Analysis Competency Model
and Self-Assessment Tool
Through the ECBA®, CCBA®,
and CBAP® certifications
IIBA® MEMBERSHIP BENEFITS
IIBA®
Membership
Elevate
your skill
set
Develop
leadership
skills
Expand
your
network
• BABOK® Guide
• Competency
Model
• BBC Conference
• IIBA® Webinar
Series
• Online Library
• Attend networking
and chapter events
• Join online
community
• Learn from experts
• Meet professional
contacts
• Volunteer with the organization
• Develop communication skills
BENEFITS OF CBAP® CERTIFICATION
INDIVIDUAL

ORGANIZATION
BENEFITS OF CBAP® CERTIFICATION (contd.)
Competence in the principles
and practices of business
analysis
Advanced career potential
Helps ensure continuous self
improvement
Participation in a recognized
professional group
Personal satisfaction of
accomplishing a milestone
Potential for higher income
Recognition of professional
competence
Improved overall
performance, removal of
uncertainty, and wider market
opportunities
Recognition for having
invested in a business
analysis career
INDIVIDUAL
INDIVIDUAL
Benefits to the individual

ORGANIZATION
BENEFITS OF CBAP® CERTIFICATION (contd.)
ORGANIZATION
Professional development, advancement, and
recognition for staff
Improves staff responsibility, commitment, and
motivation
Enables to identify professional business analysts
to clients and business partners
Demonstrates use of industry-standard
business analysis practices
Helps in establishment and implementation of
business analysis practices as outlined in the
BABOK® Guide
Demonstrates to the stakeholders that
business is run effectively
Ensures more reliable and higher quality results
are produced with increased efficiency and
consistency
Demonstrates commitment to the field of business
analysis
Benefits to the organization

ORGANIZATION
INDIVIDUAL
ELIGIBILITY CRITERIA
CBAP® CERTIFICATION
Criteria Requirements for CBAP®® Certification
Work Experience Minimum 7500 Hours of Business Analysis experience in the last 10
years
Knowledge Area Expertise Minimum 900 hours in four of the six knowledge areas
Professional Development 35 Hours in the last 4 Years
References 2
Signed Code of Conduct Yes
Step 4
Prepare for
the exam
Step 1
Apply for and
pay for the
certification
Step 2
Pay for the
exam
Step 3
Register for
the exam
APPLICATION AND EXAM PROCESS
Step 5
T
ake the
exam
 
  
APPLICATION AND EXAM FEES
• Application and Exam Fees
• Other Fees
Fee Member Non-member
Application Fee (non-refundable) $125 $125
Exam Fee – English and Japanese $325 $450
Exam Fee – German *$770 $880
Fee Member Non-member
Exam Cancellation Fee (CBT) $50 $50
Exam Re-write Fee – English and Japanese $250 $375
Exam Re-write Fee – German *$430 $540
NOTE:
• *IIBA® is partnering with The EuropeanAssociation of BusinessAnalysis (EABA) and, in a joint effort, is now offering business analysis certification exams
in German-speaking Europe and at all test center locations where IIBA exams are offered.
• All fees are payable in U.S. dollars (USD) plus GST/HST if you are a Canadian resident or a GST/HST registrant.
• The application fee is not refundable regardless of whether an application is approved or approved pending audit and if an application audit is not passed.
PREPARING FOR THE EXAM
The CBAP® exam is a 3.5-hours long exam and consists of 150 multiple choice questions with four possible
answers to select from.
To prepare for the exam:
Review the BABOK® Learning Guide
found in the Online Library on the
Community Network
Review FrequentlyAsked Questions
(FAQ) on the IIBA ® website
Review recommended resources on the
IIBA® website
Attend training as needed
Review the IIBA® BABOK® Guide
Find a business analysis mentor
Join a study group
Attend local IIBA® Chapter meetings
Review available study guide(s)
Find opportunities to practice tasks by
following the BABOK® Guide
CBAP® EXAM BLUEPRINT
Domain Percentage of Questions
Business Analysis Planning and Monitoring 14%
Elicitation and Collaboration 12%
Requirements Life Cycle Management 15%
Strategy Analysis 15%
Requirements Analysis and Design Definition 30%
Solution Evaluation 14%
RECERTIFICATION
REASONS
The CBAP® Recertification program helps an ongoing professional individuals demonstrate commitment to
the business analysis profession.
Recertification
Program
RECERTIFICATION
PURPOSE
The Recertification program supports the ongoing professional development of individuals who have attained
the CBAP® designation.
Recertification
Program
Encourages and recognizes
individualized learning
Encourages the efforts of
CBAP® recipients to give
back to the profession
Encourages the ongoing
professional development of
CBAP® recipients
Offers a standard and objective
mechanism for recording
professional development
activities
Sustains the global recognition
and value of the CBAP®
designation

RECERTIFICATION
NOTES
All CBAP® recipients are required to meet continuing proficiency requirements to maintain their designation.
CERTIFICATION RENEWAL
IIBA® certification to be renewed
every three years
CBAP® Recertification Handbook
and related forms are posted on
the IIBA® website
ADDITIONAL NOTES
To ensure receiving all IIBA®
communication:
-Update your email address in your
profile on the IIBA® website.
- Check bulk mail folders
-Add certification@iiba.org to your
personal address book
Report 60 accepted CDUs and
submit the recertification application
prior to the end of the three-year
cycle
CDU CATEGORIES
1. Formal AcademicEducation
2.Professional Development
3.Professional Activities
4.Self-directedLearning
5.VolunteerService
6.ProfessionalExperience
KEY TAKEAWAYS
The International Institute of Business Analysis or IIBA® assists
business analysts by defining standards for business analysis,
identifying the skills needed for business analysis, and recognizing
business analysis professionals.
With an IIBA® membership, you can elevate your skill set, develop
leadership skills, and expand your network.
Membership of IIBA® provides several benefits, both to the
individual as well as to organizations.
Topreparefortheexam,reviewtheBABOK®Guide,reviewtheFAQsontheIIBA®
website,attendtrainingsasneeded,practicetasksbyfollowingtheBABOK®
Guide, findaBAmentor, andreviewavailablestudyguides.
Applying and taking the exam is a 5-step process.
The CBAP® exam is a 3.5-hours long exam and consists of 150
multiple choice questions with four possible answers to select from.
The CBAP® Recertification program helps individuals demonstrate
an ongoing professional commitment to the business analysis
profession.
This concludes 0Introduction to CBAP® Certification.”
Thenextlessonis “Introduction toBABOK®GuideVersion3.”
THANKYOU
Lesson2- IntroductiontoBABOK® V3
CBAP ®Exam Preparation Course
CBAP, CCBA and BABOK are registered certification marks owned by International Institute of Business Analysis.
WHAT’S IN IT FOR ME
What is business analysis?
What does a business analyst do?
Business Analysis Core Concept
Model Model TM — the Knowledge
Areas defined in BABOK® Version 3
What are the underlying
competencies of a business
analyst?
Techniques and perspectives in
business analysis
INTRODUCTION – WHY BUSINESS ANALYSIS
INTRODUCTION – WHY BUSINESS ANALYSIS (contd.)
 Who is a Business Analyst?
 Business Analysis Core Concept Model
 Business Analysis Knowledge Areas
 Re
D
q
e
u
fe
irc
etm
pe
en
rtU
sn
ciltassification
 Requirements and Design
 Who is a Stakeholder?
Lesson 2: Introduction to BABOK® V3
Topic2.1:KeyConceptsofBusinessAnalysis
BUSINESS ANALYSIS
WHAT IS BUSINESS ANALYSIS
Where?
Why?
When?
Business
Analysis
Define
Needs
Recommend
Solutions
Enable
BUSINESS ANALYSIS
WHO IS A BUSINESS ANALYST
Performs
tasks
defined in
the BABOK®
Guide
Discovers,
synthesizes,
and
analyzes
information
BUSINESS
ANALYST
Elicits the
needs from
stakeholders
Investigates
and clarifies
stakeholder
desires
Determines
issues and
their causes
Usefulness to a stakeholder
within a context
Circumstances that influence or are
influenced by, and provide
understanding of change
A group or individual related to a
change, need, or solution
Way to satisfy one or more
needs in a context
A problem or opportunity
The act of transformation in response to need
Change
BUSINESS ANALYSIS
BUSINESS ANALYSIS CORE CONCEPT MODEL TM
Solution
Need
Stakeholder Context
Value
BUSINESS ANALYSIS BODY OF KNOWLEDGE
KNOWLEDGE AREAS
1. Business Analysis Planning and Monitoring
2. Elicitation and Collaboration
3. Requirements Lifecycle Management
4. StrategyAnalysis
5. Requirements Analysis and Design Definition
6. Solution Evaluation
BUSINESS ANALYSIS BODY OF KNOWLEDGE (contd.)
KNOWLEDGE AREAS
1. Business Analysis Planning and Monitoring
2. Elicitation and Collaboration
3. Requirements Lifecycle Management
4. StrategyAnalysis
5. Requirements Analysis and Design Definition
6. Solution Evaluation
It describes the tasks to be performed to
organize and co-ordinate the effort of
business analysts and stakeholders.
Output from this knowledge area tasks is
used as a key input or as guidelines for the
tasks performed in other knowledge areas.
BUSINESS ANALYSIS BODY OF KNOWLEDGE (contd.)
KNOWLEDGE AREAS
1. Business Analysis Planning and Monitoring
2. Elicitation and Collaboration
3. Requirements Lifecycle Management
4. StrategyAnalysis
5. Requirements Analysis and Design Definition
6. Solution Evaluation
It describes the tasks to be performed to
prepare for elicitation, conduct the
elicitation activity, and confirm elicitation
results. It also explains how communication
of business analysis information and
ongoing collaboration must be done with
the stakeholder.
BUSINESS ANALYSIS BODY OF KNOWLEDGE (contd.)
KNOWLEDGE AREAS
1. Business Analysis Planning and Monitoring
2. Elicitation and Collaboration
3. Requirements Lifecycle Management
4. StrategyAnalysis
5. Requirements Analysis and Design Definition
6. Solution Evaluation
It describes the tasks to be performed to
manage and maintain requirements and
design information from inception to
retirement.
BUSINESS ANALYSIS BODY OF KNOWLEDGE (contd.)
KNOWLEDGE AREAS
1. Business Analysis Planning and Monitoring
2. Elicitation and Collaboration
3. Requirements Lifecycle Management
4. StrategyAnalysis
5. Requirements Analysis and Design Definition
6. Solution Evaluation
It describes the tasks to be performed for
collaboration with the stakeholder to identify
the real need and enable the organization
to address that need. It also explains the
tasks to align the resulting strategy for the
change with higher- and lower-level
strategies to address the need.
BUSINESS ANALYSIS BODY OF KNOWLEDGE (contd.)
KNOWLEDGE AREAS
1. Business Analysis Planning and Monitoring
2. Elicitation and Collaboration
3. Requirements Lifecycle Management
4. StrategyAnalysis
5. Requirements Analysis and Design Definition
6. Solution Evaluation
It describes the tasks to be performed to
structure and organize requirements
discovered during the elicitation activity,
specify and model requirements and
design, validate and verify information, and
identify solution options that meet the
business need.
BUSINESS ANALYSIS BODY OF KNOWLEDGE (contd.)
KNOWLEDGE AREAS
1. Business Analysis Planning and Monitoring
2. Elicitation and Collaboration
3. Requirements Lifecycle Management
4. StrategyAnalysis
5. Requirements Analysis and Design Definition
6. Solution Evaluation
It describes the tasks to be performed to
assess the performance of and value
delivered by a solution and recommend
actions to be taken to realize the full value
of the solution.
BUSINESS ANALYSIS BODY OF KNOWLEDGE
KNOWLEDGE AREAS – RELATIONSHIPS
Business
Analysis Planning
and Monitoring
Strategy
Analysis
Requirements
Analysis and
Design
Definition
Solution
Evaluation
Requirements
Lifecycle
Management
Elicitation and
Collaboration
REQUIREMENTS
REQUIREMENTS CLASSIFICATION
Business
Requirements
Stakeholder
Requirements
Solution
Requirements
Transition
Requirements
Describe the needs of the
stakeholders that must be met to
achieve the business requirements
Statements of goals, objectives,
andoutcomethatdescribewhya
changehasbeeninitiated
Describe the capabilities that the
solution must have to facilitate
transition of the organization from
the current state to the future state
Describe the capabilities and
qualities of a solution that meets
stakeholder requirements; can be
Functional requirements or Non-
functional requirements
REQUIREMENTS (contd.)
REQUIREMENTS DOCUMENTS
Business Requirements Documents that
comprise both business and
stakeholder requirements
Functional Specification Document
Solution Requirements Specification
R
R
e
e
q
qu
uiir
re
em
me
e
n
n
tt
s
s
D
D
o
o
c
c
u
u
m
m
e
e
n
n
t
t
s
s
BRD
FSD
SRS
REQUIREMENTS AND DESIGN
DESIGN CYCLE
Requirements
Design
Needs
Solution
Requirements lead to design.
Requirements may be used to define design.
Requirements are focused on needs and
design is focused on the solution.
Requirements and design could be recursive.
A Business Analyst reviews designs to
ensure they align with the requirements.
BABOK® GUIDE SPECIFICATIONS
STAKEHOLDERS
Business
Analyst
Project
Manager
Implementation
SME
Regulator
Sponsor
Supplier
Customer
End User
Domain
SME
Tester
Operational
Support
A stakeholder is an individual or group that a Business Analyst is likely to interact with directly or indirectly.
 Competencies of a business analyst
Lesson 2: Introduction to BABOK® V3
Topic2.2:CompetenciesofaBusinessAnalyst
COMPETENCIES OF A BUSINESS ANALYST
CATEGORIES
Analytical
Thinking and
Problem-
Solving Skills
(7)
Behavioral
Characteristics
(5)
Business
Knowledge
(5)
Communication
Skills
(4)
Tools and
T
echnology
(3)
Interaction
Skills
(5)
Competencies
COMPETENCIES OF A BUSINESS ANALYST (contd.)
ANALYTICAL THINKING AND PROBLEM SOLVING
Creative Thinking
Ability to generate new
ideas and approaches to
solve problems
Learning
Ability to quickly absorb
new and different types of
information
Conceptual Thinking
Ability to understand how
disparate information fits
into a larger picture
Decision Making
Ability to understand the
criteria needed to make
decisions
Ability to understand the
underlying cause of a
problem and use problem-
solving techniques
Visual Thinking
Ability to visually
communicate complex
concepts and models to
stakeholders
 
 
Problem Solving
 
System Thinking
Ability to understand the
interactions between
people, process, and
technology

Analytical Thinking and
Problem-Solving Skills
COMPETENCIES OF A BUSINESS ANALYST (contd.)
BEHAVIORAL CHARACTERISTCS
Behavioral
Characteristics
Ethics
Enable a business analyst
to earn the respect of
stakeholders
Trustworthiness
Perception that one is
worthy of trust
Personal Accountability
Enables a business
analyst to achieve target
goals
Organization and Time
Management
Helps perform tasks
effectively and use work
time efficiently
 


Adaptability
Ability to adjust behavioral
style, methods, and
approach to the
environment

COMPETENCIES OF A BUSINESS ANALYST (contd.)
BUSINESS KNOWLEDGE
Business Knowledge
Business Acumen
Ability to understand
business principles and
practices and apply them
at work
Organization Knowledge
Understanding the
management structure and
the business architecture
of the enterprise
Industry Knowledge
Understanding how a
company is positioned
within an industry
Solution Knowledge
Understanding solutions
that will help expedite the
discovery of potential
changes through elicitation
 


Methodology Knowledge
Enables a business
analyst to quickly adapt to,
and perform in, a new
environment

COMPETENCIES OF A BUSINESS ANALYST (contd.)
COMMUNICATION SKILLS
Communication Skills
Verbal Communication
Used to convey ideas,
concepts, facts, and
opinions to stakeholders
Written Communication
Practice of using text,
symbols, models, pictures,
or sketches to convey and
share information
Enables sending and
receiving messages through
body movements, postures,
facial expressions, gestures,
and eye contact
Nonverba

l Communication
Listening
Enables accurate
understanding of verbal
communication

 
Communication skills refers to
the ability to communicate
effectively.
COMPETENCIES OF A BUSINESS ANALYST (contd.)
INTERACTION SKILLS
Interaction Skills
Facilitation
Ability to moderate
discussions within a group
Teamwork
Enables working with
team members effectively
Leadership and
Influencing
Involves motivating people
to act in ways that enable
them to work together
Negotiation
Involves mediating
discussions and resolving
differences in teams
 


Teaching
Enables a business analyst
to effectively communicate
business information,
concepts, ideas, and issues

Interaction Skills
include the ability
to relate to and co-
operate and
communicate with
different kinds of
people.
COMPETENCIES OF A BUSINESS ANALYST (contd.)
TOOLS AND TECHNOLOGY SKILLS
Tools and Technology
Skills
Office Productivity
Helps document and track
information and artifacts
Business Analysis
Helps model, diagram,
document, and manage the
output of business analysis
activities


Helps perform business
analysis activities, manage
teams, and collaborate
with stakeholders
Com

munication
A business analyst uses a variety of software to support
communication and collaboration, create and maintain
requirement artifacts, and increase overall productivity.
 T
eDcehfneicqtupeesrrUefneirtredto by BABOK V3
Lesson 2: Introduction to BABOK® V3
Topic2.3:BusinessAnalysisTechniques
BUSINESS ANALYSIS TECHNIQUES
50 TECHNIQUES
10.1 Acceptance and
Evaluation Criteria
10.11 Concept Modelling 10.21 Focus Groups 10.31 Observation 10.41 Scope Modelling
10.2 Backlog Management 10.12 Data Dictionary
10.22 Functional
Decomposition
10.32 Organizational
Modelling
10.42 Sequence Diagrams
10.3 Balanced Scorecard 10.13 Data Flow Diagrams 10.23 Glossary 10.33 Prioritization
10.43 Stakeholder List, Map,
or Personas
10.4 Benchmarking and
Market Analysis
10.14 Data Mining 10.24 Interface Analysis 10.34 Process Analysis 10.44 State Modelling
10.5 Brainstorming 10.15 Data Modelling 10.25 Interviews 10.35 Process Modelling
10.45 Survey or
Questionnaire
10.6 Business Capability
Analysis
10.16 Decision Analysis 10.26 Item Tracking 10.36 Prototyping 10.46 SWOT Analysis
10.7 Business Cases 10.17 Decision Modelling 10.27 Lessons Learned 10.37 Reviews
10.47 Use Cases and
Scenarios
10.8 Business Model
Canvas
10.18 Document Analysis
10.28 Metrics and Key
Performance Indicators
(KPIs)
10.38 Risk Analysis and
Management
10.48 User Stories
10.9 Business Rules
Analysis
10.19 Estimation 10.29 Mind Mapping
10.39 Roles and
Permissions Matrix
10.49 Vendor Assessment
10.10 Collaborative Games 10.20 Financial Analysis
10.30 Non functional
Requirements Analysis
10.40 Root Cause Analysis 10.50 Workshops
 Bu
D
s
e
in
fe
ec
stspA
en
r a
U
ly
ns
itis perspectives
Lesson 2: Introduction to BABOK® V3
Topic2.4:BusinessAnalysisPerspectives
BUSINESS ANALYSIS PERSPECTIVES
FIVE BUSINESS ANALYSIS PERSPECTIVES
Perspectives provide focus to tasks and techniques specific to the context of an initiative.
Perspectives are not mutually exclusive. Most initiatives are likely to engage one or more perspectives.
BABOK® includes five perspectives.
Agile Business
Intelligence
Information
Technology
Business
Architecture
Business
Process
Management
BUSINESS ANALYSIS PERSPECTIVES (contd.)
FIVE BUSINESS ANALYSIS PERSPECTIVES
Agile Business
Intelligence
Information
Technology
Business
Architecture
Business
Process
Management
An Agile initiative involves:
• An agile mindset
• Agile values and principles
• Progressive elaboration of business planning, elicitation, analysis, design, and construction
of solution
A BusinessAnalyst is an active member of an Agile team.
BUSINESS ANALYSIS PERSPECTIVES (contd.)
FIVE BUSINESS ANALYSIS PERSPECTIVES
Agile Business
Intelligence
Information
Technology
Business
Architecture
Business
Process
Management
Business Intelligence:
• Highlights the characteristics of business analysis in the context of transformation,
integration, and enhancing data
• Is the transformation of data into valuable information
• Helps stakeholders make informed decisions
• Helps stakeholders manage strategic, tactical, and operational performance
BUSINESS ANALYSIS PERSPECTIVES (contd.)
FIVE BUSINESS ANALYSIS PERSPECTIVES
Agile Business
Intelligence
Information
Technology
Business
Architecture
Business
Process
Management
Information Technology:
• This perspective highlights the characteristics of business analysis in the context of the
impact of change on IT systems in an organization.
• Organizations need to undertake initiatives to upgrade or replace IT systems.
• A business analyst plays the role of a translator between business and technology teams
in the change process.
BUSINESS ANALYSIS PERSPECTIVES (contd.)
FIVE BUSINESS ANALYSIS PERSPECTIVES
Agile Business
Intelligence
Information
Technology
Business
Architecture
Business
Process
Management
BusinessArchitecture:
• Highlights the unique characteristics of business analysis in the context of business
architecture
• Involves analytical thinking and applying architectural principles at the enterprise level
• Supports ongoing business transformation efforts
• Could include changes to business, operational model, or structure
BUSINESS ANALYSIS PERSPECTIVES (contd.)
FIVE BUSINESS ANALYSIS PERSPECTIVES
Agile Business
Intelligence
Information
Technology
Business
Architecture
Business
Process
Management
Business Process Management:
• Highlights the characteristics of business analysis in the context of developing or improving
business processes
• Focuses on how an organization works across multiple functional areas to improve
business processes
• Is an integral part of ongoing management and operations of an organization
Quiz
a.
b.
c.
d.
Which of the following is NOT a component of the Business Analysis Core Concept
Model TM (BACCMTM)?
QUIZ
1
Stakeholder
Content
Solution
Change
The correct answer is c.
Explanation: Content is not a component of Business Analysis Core Concept Model TM
(BACCM). The components are Change, Need, Solution, Stakeholder, Value, and Context.
a.
b.
c.
d.
Which of the following is NOT a component of the Business Analysis Core Concept
Model TM (BACCMTM)?
QUIZ
1
Change
Solution
Content
Stakeholder
a.
b.
c.
d.
are focused on the needs, and are focused on the solution.
QUIZ
2
Designs; Requirements
Solutions; Requirements
Requirements; Designs
Problems; Requirements
The correct answer is b.
Explanation: According to the Requirements and Design Cycle, Requirements are focused
on the needs and Designs are focused on the solution.
a.
b.
c.
d.
are focused on the needs, and are focused on the solution.
QUIZ
2
Problems; Requirements
Requirements; Designs
Solutions; Requirements
Designs; Requirements
a.
b.
c.
d.
Which of the following is a business analysis knowledge area?
QUIZ
3
Requirements Analysis and Management
Solution Assessment and Validation
Enterprise Analysis
Elicitation and Collaboration
The correct answer is a.
Explanation: Elicitation and Collaboration is one of the business analysis knowledge areas.
.
a.
b.
c.
d.
Which of the following is a business analysis knowledge area?
QUIZ
3
Elicitation and Collaboration
Enterprise Analysis
Solution Assessment and Validation
Requirements Analysis and Management
a.
b.
c.
d.
Which of the following is NOT a category of business analyst competencies as
defined in BABOK®?
QUIZ
4
Interaction
Organizational Knowledge
Communication
Analytical Thinking and Problem Solving
The correct answer is c.
Explanation: Organizational Knowledge is not a category of business analyst competencies.
It is, in fact, a core competency of the Business Knowledge category.
a.
b.
c.
d.
Which of the following is NOT a category of business analyst competencies as
defined in BABOK®?
QUIZ
4
Analytical Thinking and Problem Solving
Communication
Organizational Knowledge
Interaction
KEY TAKEAWAYS
Business analysis is the practice of enabling change in an
enterprise by defining needs and recommending solutions
that deliver value to stakeholders.
A business analyst is a person who performs business
analysis tasks described in the BABOK® Guide.
The BusinessAnalysis Core Concept Model TM (BACCMTM)
defines the conceptual framework for the practice of
business analysis.
The BABOK® Guide groups business analysis tasks into six
knowledge areas.
The BABOK® GUIDE describes 29 business analysis
competencies, 50 techniques, and 5 perspectives.
This concludes Introduction to BABOK® V3.
Thenextlessonis“Business AnalysisPlanningandMonitoring.”
THANKYOU
Lesson3- BusinessAnalysisPlanningandMonitoring
CBAP®Exam Preparation Course
CBAP, CCBA and BABOK are registered certification marks owned by International Institute of Business Analysis.
WHAT’S IN IT FOR ME
How to plan the Business Analysis
Approach
How to plan Stakeholder
Engagement
How to plan Business Analysis
Information Management and
Governance
How to identify Business Analysis
Performance Improvements
Understand tools and techniques
used in Business Analysis Planning
and Monitoring
INTRODUCTION
BUSINESS ANALYSIS PLANNING AND MONITORING
Circumstances that influence or are
influenced by, and provide
understanding of change
Way to satisfy one or
more needs in a context
A problem or opportunity
INTRODUCTION
The act of transformation in response to need
Change
Solution
Need
A group or individual
related to a change, need, or solution
Stakeholder Context
Value
Usefulness to a stakeholder
within a context
BUSINESS ANALYSIS PLANNING AND MONITORING
INPUT
, TASKS, AND OUTPUT
External
Influences
Internal
Influences
Needs
• Business AnalysisApproach
• Stakeholder Engagement Approach
• Governance Approach
• Information Management Approach
• BusinessAnalysis Performance
Assessment
Plan BusinessAnalysis
Approach
Plan Stakeholder
Engagement
Plan BusinessAnalysis
Governance
Plan BusinessAnalysis
Information Management
Identify Business Analysis
Performance Improvements
 define overall method to conduct business analysis activities
 Defect per Unit
Lesson 3: Business Analysis Planning and Monitoring
Topic3.1:PlanBusinessAnalysisApproach
PLAN BUSINESS ANALYSIS APPROACH
PURPOSE
Need
• Creation or
Selection of
Methodology
• Identify BA
Deliverables
• Define
Activities and
Tasks
Planning
of
Business
Analysis
• Who will
perform
• Timing
• Sequencing
Business
Analysis
Approach
Input Output
PLAN BUSINESS ANALYSIS APPROACH
ELEMENTS
Planning methods falls between Predictive and
Adaptive approaches
Formality and level of details of BA deliverables are
based on the planning approach
Identify and break down activities into tasks
Timing for the business analysis tasks to be
performed
Complexity and size of the change, and overall risk
Business analysis approach reviewed and agreed
upon by key stakeholders
PLAN BUSINESS ANALYSIS APPROACH
GUIDELINES AND TOOLS
Guidelines and
Tools while
planning BA
approach
BusinessAnalysis Performance
Assessment
Stakeholder Engagement Approach
Guidance while
planning BA
approach
Business Policies
Methodologies and Framework
Expert judgement
PLAN BUSINESS ANALYSIS APPROACH
Brainstorming
Business Cases Estimation
TECHNIQUES
Functional
Document Analysis Financial Analysis
Decomposition
Interviews
Item Tracking
Lessons Learned
Process Modeling
Reviews
Risk Analysis and
Management
Scope Modeling
Survey and
Questionnaire
Workshops
1
2
3
4
5
6
7
8
9
10
11
12
13
14
15
PLAN BUSINESS ANALYSIS APPROACH
TECHNIQUE – LESSONS LEARNED – WHY?
WHY?
Lessons
Learned
Successes
Opportunities for improvement
Failures
Recommendations for
improvement
PLAN BUSINESS ANALYSIS APPROACH
TECHNIQUE – LESSONS LEARNED - OVERVIEW
Lessons
Learned
Also known as
Retrospective
Formal or Informal
Reinforcing positive
experiences and
successes
! Participants may be reluctant to document and
discuss problems. Honest discussion may not occur.
PLAN BUSINESS ANALYSIS APPROACH
TECHNIQUE – LESSONS LEARNED - ELEMENTS
Business
analysis
activities and
deliverables
The final solution,
service or product
Impact to
organizational
processes
Performance
expectations
and results
Positive or
negative
variance
Root cause of
results impacting
performance
Recommendations
for behavioral
approaches
Domain Subject
Matter Expert
Project Manager
Regulator Sponsor
Stakeholders
PLAN BUSINESS ANALYSIS APPROACH
STAKEHOLDERS
 plan an approach for establishing and maintaining effective working
relationship with stakeholders
 Defect per Unit
Lesson 3: Business Analysis Planning and Monitoring
Topic3.2:PlanStakeholderEngagement
PLAN STAKEHOLDER ENGAGEMENT
PURPOSE
Plan an approach for establishing
and maintaining effective working
relationship with stakeholders
Identify Stakeholder
Analyze Stakeholder
Characteristics
Consider Stakeholder complexity while planning for stakeholder
engagement.
Needs and BA
Approach
Plan Stakeholder
Engagement
Approach
PLAN STAKEHOLDER ENGAGEMENT
ELEMENTS
Perform
Stakeholder
Analysis
Roles
Attitude
Decision Making Authority
Level of Power or Influence
Stakeholder
Communication
Needs
Planning
Stakeholder
Collaboration
PLAN STAKEHOLDER ENGAGEMENT
GUIDELINES AND TOOLS
Guidelines and Tools
while planning
Stakeholder
Engagement
BusinessAnalysis Performance Assessment
Change Strategy
Current State Description
PLAN STAKEHOLDER ENGAGEMENT
TECHNIQUES
Business Rules
Brainstorming
Analysis
Document Analysis
Interviews
Lessons Learned
Mind Mapping
Organizational
Modeling
Process Modeling
Stakeholder List,
Map or Personas
Risk Analysis and
Management
Scope Modeling Survey &
Questionnaire
Workshops
1
2
3
4
5
6
7
8
9
10
11
12
13
ORGANIZATIONAL MODEL
PURPOSE
Organizational Modeling
• Describe the roles,
responsibilities, and reporting
structure
• Align these structures with the
organizational goals
Organizational Model
• Boundaries of a group
• Formal relationships between
members
• Functional role of each person
• Interfaces between unit or
stakeholders
• Common in most organizations
ORGANIZATIONAL MODEL
ELEMENTS
Types of Organizational
Models
Functionally oriented
Market oriented
Matrix model
Organization unit includes
number of roles
Interfaces with other organization
units
ORGANIZATIONAL MODEL
ELEMENTS (contd.)
Identify informal lines of communication which can influence business analysis
activities
Organization
Chart
Box depicts organization units, roles and
people
Line depicts reporting or formal
communication channel
ORGANIZATIONAL MODEL
EXAMPLE
Business Unit
Head
Program
Management
Sr. Project
Manager
Project
Manager
Development
Development
Manager
Lead
Quality
Analyst
Architect
Operations
Operations
Manager
Development Infrastructure
Manager
Logistics
Finance
Sr. Finance
Manager
Auditor
Human
Resource
Sr. Human
Resource
Manager
On boarding
specialist
Sales and
Marketing
Marketing
Manager
Brand
Manager
Sales
Manager
Event
Manager
Purchase
Sr. Purchase
manager
Vendor
Manager
STAKEHOLDER LIST, MAP, OR PERSONAS
OVERVIEW
Identify
Stakeholder
Stakeholder
Analysis
• Identifying
stakeholders
List of
stakeholders
Stakeholder characteristics
• Level of authority within the domain of change
• Attitude toward or interest in the change
• Attitude toward business analysis activities
• Level of decision-making authority
STAKEHOLDER LIST, MAP OR PERSONAS
ELEMENTS
Stakeholder Map
• Stakeholder Matrix
• Stakeholder Onion Diagram
RACI Matrix (Responsible, Accountable, Consulted and Informed)
Personas
STAKEHOLDER LIST, MAP, OR PERSONAS
EXAMPLE
Ensure stakeholders remain
satisfied
Work closely with
stakeholders and engage
with them effectively
Monitor to ensure
stakeholder interest or
influence does not change
Keep stakeholders informed
as they are likely to be very
concerned
Low
Low
High
High
Interest / Impact of
Stakeholder
Influence of
Stakeholder
STAKEHOLDER LIST, MAP OR PERSONAS
STAKEHOLDER ONION DIAGRAM
Solution
Delivery
Affected Organizational Unit
Organization or Enterprise
Affected External Stakeholders
Customers, suppliers,
regulators, and others
Sponsors, executives,
domain SMEs, and
others who interact with
the affected group
End users, help desk,
and other whose work
changes when solution is
delivered
Project team and others
directly involved with
creating the solution
STAKEHOLDER LIST, MAP OR PERSONAS
RACI – RESPONSIBLE, ACCOUNTABLE, CONSULTED, INFORMED
Task Sponsor
Project
Manager
Implementation
Team
Operations
Business
Analyst
Regulators
Identify problem or
opportunity
R R C R A I
Identify risk associated
with business analysis work
R R R R A I
Conduct feasibility study C R R C A
Recommend solution R C R R A I
Approve Business Analysis
Deliverables
A C C C R I
Analyze Requirements I I C I A
Communicate
Requirements and Design
I C I I A I
Assess solution
performance
C C C A R C
STAKEHOLDER LIST, MAP OR PERSONAS
PERSONAS
Paul Atkins,
Marketing
Manager (Health
Care)
32 Years,
Married, One Child
Role
What is your job role?
What is a typical day like?
What skills, knowledge, and tools do you require?
Who reports to you and whom do you report to?
Goals
What are you responsible for?
Challenges
What are your biggest challenges in your work?
Company
Which industry does your company work in?
What is the total revenue of your company?
How many employees work in your company?
Personal Background: Age, Family (single, married,
children), Education
Preferences
How do you prefer to interact (email, phone, in person)?
PLAN STAKEHOLDER ENGAGEMENT
STAKEHOLDERS
Customer
End User
Supplier
Regulator
Sponsor
Project
Manager
Domain Subject
Matter Expert
 define how decisions are made about requirements and designs, including
reviews, change control, approval and prioritization
 Defect per Unit
Lesson 3: Business Analysis Planning and Monitoring
Topic3.3:PlanBusinessAnalysisGovernance
PLAN BUSINESS ANALYSIS GOVERNANCE
PURPOSE
How decisions are made about
requirements and designs,
including reviews, change control,
approval and prioritization.
BAApproach
and Stakeholder
Engagement
Approach
Governance
Approach
PLAN BUSINESS ANALYSIS GOVERNANCE
ELEMENTS
Elements
Decision
Making
Change
Control
Process
Plan
Prioritization
Approach
Planfor
Approval
PLAN BUSINESS ANALYSIS GOVERNANCE
GUIDELINES AND TOOLS
Guidelines and
Tools while planning
Business Analysis
Governance
BusinessAnalysis Performance
Assessment
Business Policies
Current State Description
Legal / Regulatory Information
PLAN BUSINESS ANALYSIS GOVERNANCE
TECHNIQUES
Brainstorming Document Analysis
Interviews
Lessons Learned
Organizational
Modeling
Item Tracking
Process Modeling
Survey &
Questionnaire
Workshops
1
2
3
4
5
6
7
8
9
10
Reviews
PLAN BUSINESS ANALYSIS GOVERNANCE
STAKEHOLDERS
Domain
Subject Matter
Expert
Project
Manager
Regulator
Sponsor
 Defect per Unit
 Storing and accessing business analysis information
Lesson 3: Business Analysis Planning and Monitoring
Topic3.4:PlanBusinessAnalysisInformationManagement
PLAN BUSINESS ANALYSIS INFORMATION MANAGEMENT
PURPOSE
How to develop business
analysis information
management plan
Business analysis
information management
approach
• Business Analysis
Approach
• Stakeholder Engagement
Approach
• Governance Approach
Inputs Output
PLAN BUSINESS ANALYSIS INFORMATION MANAGEMENT
ELEMENTS
Organization of
Business
Analysis
information
Level of
Abstraction
Plan Traceability
Approach
Plan for
Requirements
Reuse
Storage and
Access
Requirements
Attributes
PLAN BUSINESS ANALYSIS INFORMATION MANAGEMENT
GUIDELINES AND TOOLS
Guidelines and Tools
while planning Business
Analysis Information
Management
BusinessAnalysis Performance
Assessment
Business Policies
Information Management Tools
Legal / Regulatory Information
PLAN BUSINESS ANALYSIS INFORMATION MANAGEMENT
TECHNIQUES
Brainstorming
Interviews
Lessons Learned
Mind Mapping
Item Tracking
Process Modeling
Surveys and
Questionnaires
Workshops
1
2
3
4
5
6
7
8
PLAN BUSINESS ANALYSIS INFORMATION MANAGEMENT
STAKEHOLDERS
Regulator
 Assess business analysis work and to plan to improve processes where
required
 Defect per Unit
Lesson 3: Business Analysis Planning and Monitoring
Topic3.5:Identify BusinessAnalysisPerformanceImprovements
IDENTIFY BUSINESS ANALYSIS PERFORMANCE IMPROVEMENTS
PURPOSE
Assess Business Analysis Work
and Plan to Improve Processes
BAApproach, and
Performance Objectives
Business Analysis
Performance Assessment
Input Output
Performance
Analysis
Assessment
Measures
Analyze
Results
Recommend
Actions for
Improvements
IDENTIFY BUSINESS ANALYSIS PERFORMANCE IMPROVEMENTS
ELEMENTS
IDENTIFY BUSINESS ANALYSIS PERFORMANCE IMPROVEMENTS
GUIDELINES AND TOOLS
Organizational Performance Standards includes
details of performance metrics or expectations for
business analysis work mandated by the organization.
IDENTIFY BUSINESS ANALYSIS PERFORMANCE IMPROVEMENTS
TECHNIQUES
Brainstorming
Interviews Lessons Learned
Process modeling
Item Tracking Process Analysis
Metrics and Key
Performance
Indicators (KPIs)
1
2
3
4
5
8
9
10
6
7
11
12
13
Observation
Reviews
Risk Analysis and
Management
Root Cause Analysis
Surveys and
Questionnaires
Workshops
METRICS AND KEY PERFORMANCE INDICATORS (KPIs)
OVERVIEW
Metrics and Key Performance Indicators (KPIs) measure the
performance of solutions, solution components, and other matters of
interest to stakeholders.
A Metrics is quantifiable level of an indicator.
A Key Performance Indicator (KPI) measures progress towards a
strategic goal or objective. KPIs derive from metrics.
KPIs to measure the effectiveness of business analysis.
Reporting is the process of informing stakeholder of metrics or
indicators in specified format and specified intervals.
METRICS AND KEY PERFORMANCE INDICATORS (KPIs)
ELEMENTS
Characteristics
of Indicator
• Clear
• Relevant
• Economical
• Adequate
• Quantifiable
• Trustworthy
and Credible
Metrics
• Specific point
• Threshold
• Range
Structure
• Monitoring
• Evaluation
Reporting
• Baseline
• Current and
Target metrics
Quality of
metrics and KPIs
• Reliability
• Validity
• Timeliness
IDENTIFY BUSINESS ANALYSIS PERFORMANCE IMPROVEMENTS
STAKEHOLDERS
Stakeholders
Domain
Subject
Matter
Expert
Project
Manager
Sponsor
 Defect per Unit
Lesson 3: Business Analysis Planning and Monitoring
CASESTUDYEXERCISE
CASE STUDY
OVERVIEW
BATONICS
Pharma Company
Unable to
deliver products
on time
Customers switching
to competitor
products
CASE STUDY
BUSINESS ANALYSIS ACTIVITIES
Identified the business analysis deliverables:
Documentation of existing supply chain management
process
Root cause analysis
Solution options
Recommendation of tools and technologies
Recommended optimal solution
CASE STUDY
BUSINESS ANALYSIS ACTIVITIES
You have identified and
analyzed the stakeholders
involved in the process.
You have gone through existing documents, meetings with
subject matter experts and people involved in this process and
come up with the activities and task.
You have also estimated the
task and who are involved in
performing the task.
You have established the
process of decision making,
change control,
prioritization, and approval.
You have determined how are you going to store and retrieve
business analysis information.
You have had meetings with
the key stakeholders to review
and get approval on the
business analysis approach.
CASE STUDY
EXERCISE
Questions Response
1 Which document is created to define decision making,
change control, prioritization and approval process?
Information Management Approach
BA Governance Approach
2 Which methodology is used to define business
analysis approach?
Adaptive
Predictive
3 Which document specifies the level of responsibility
expected from each stakeholder?
RACI Matrix
Stakeholder Matrix
4 Which technique is used to identify roles and
responsibilities within the organization?
Organizational Modeling Technique
BA Performance Assessment
5 Which document indicates how stakeholder is involved
with the solution?
Stakeholder Matrix
Stakeholder Onion Diagram
6 Which task is performed to identify and analyze the
stakeholders?
Plan Stakeholder Engagement
Personas
7 Among stakeholders, who are not likely to get involved
in this initiative?
Regulators
Domain Subject Matter Expert
CASE STUDY
ANSWERS
Questions Answers
1 Which document have your created to define decision making,
change control, prioritization and approval process?
BA Governance Approach
2 Which methodology is used to define business analysis
approach?
Adaptive
3 Which document specifies the level of responsibility expected
from each stakeholder?
RACI Matrix
4 Which technique is used to identify roles and responsibilities
within organization?
Organizational Modeling
5 Which document indicates how stakeholder is involved with
the solution?
Stakeholder Onion Diagram
6 Which task is performed to identify and analyze stakeholders? Plan Stakeholder Engagement
7 Among stakeholders, who are not likely to get involved in this
initiative?
Regulators, Customers
Quiz
QUIZ
1
a.
b.
c.
d.
What does RACI stand for?
Responsible, Accountable, Collaborate, Informed
Responsible, Author, Consulted, Informed
Responsible, Accountable, Consulted, Informed
Responsible, Authorized, Consulted, Informed
QUIZ
1
The correct answer is c.
Explanation: RACI stands for Responsible, Accountable, Consulted, Informed
a.
b.
c.
d.
What does RACI stand for?
Responsible, Accountable, Collaborate, Informed
Responsible, Author, Consulted, Informed
Responsible, Accountable, Consulted, Informed
Responsible, Authorized, Consulted, Informed
a.
b.
c.
d.
Which one of the following is not a characteristics of an indicator
QUIZ
2
Relevant
Economical
Transparent
Adequate
QUIZ
2
The correct answer is c
Explanation: Transparent is not a characteristics of an indicator. Characteristics of an
indicator are Clear, Relevant, Economical, Adequate, Quantifiable, Trustworthy and Credible
a.
b.
c.
d.
Which one of the following is not a characteristics of an indicator
Relevant
Economical
Transparent
Adequate
QUIZ
3
a.
b.
c.
d.
Which one of the following is not an input to plan business analysis information
management approach?
Business Analysis Governance Approach
Business Analysis Approach
Stakeholder Engagement Approach
Business Analysis Traceability Approach
QUIZ
3
The correct answer is d
Explanation: Business Analysis Traceability Approach is not an input to plan business
analysis information management approach. Traceability approach is one of the element of
Business Analysis Information Management Approach
a.
b.
c.
d.
Which one of the following is not an input to plan business analysis information
management approach?
Business Analysis Governance Approach
Business Analysis Approach
Stakeholder Engagement Approach
Business Analysis Traceability Approach
KEY TAKEAWAYS (1 of 2)
BA approach or plan documents -
BA approach, Stakeholder engagement approach, BA
governance approach, and BA information management
approach.
Business Analysis approach or plan is developed
based on need, methodology, complexity, size and risk.
The stakeholder engagement, governance, and
information management plan can be separate or part
of the business analysis plan, depending on the size of
the initiative or organizational standard.
Output of business analysis planning and monitoring
knowledge area governs other tasks in all knowledge
area.
KEY TAKEAWAYS (contd.)
Business analysis plan documents are reviewed and
approved by key stakeholders.
Performance baseline, metrics and KPIs, and the
process of monitoring and evaluation are defined to
improve performance of business analysis work.
Business Analysis performance assessment is used as a
guideline in all planning tasks.
Business analysis techniques – Lessons Learned,
Organizational Modeling, Stakeholder List, Map or
Persona, RACI Matrix, Metrics and Key Performance
Indicators or KPIs – are used when performing
business analysis tasks.
This concludes 0Business Analysis Planning and
Monitoring.”
Thenextlessonis“Elicitation andCollaboration.”
THANKYOU
Lesson4- ElicitationandCollaboration
CBAP®Exam Preparation Course
CBAP, CCBA and BABOK are registered certification marks owned by International Institute of Business Analysis.
WHAT’S IN IT FOR ME
Confirm Elicitation Results
Prepare and Conduct Elicitation
Communicate Business Analysis
Information
Manage Stakeholder
Collaboration
ELICITATION AND COLLABORATION KNOWLEDGE AREA
OVERVIEW
Change
Solution
Need
Stakeholder Context
Value
ELICITATION AND COLLABORATION KNOWLEDGE AREA (contd.)
OVERVIEW
Identify business analysis information
about the context that may effect the
change
Manage collaboration with the
stakeholders
• Elicit, confirm and communicate
needs
• Needs may evolve over time
• Identify the characteristics of the change
• Identify stakeholders concerns
• Determine the extent of elicitation and collaboration
required
Change
Solution
Elicit, confirm and communicate desired
characteristics of proposed solutions
Need
Stakeholder Context
Value
Assess the relative value of information
provided during elicitation
ELICITATION AND COLLABORATION KNOWLEDGE AREA
TASKS AND OUTPUT
Tasks
1. Prepare for Elicitation
2. Conduct Elicitation
3.Confirm Elicitation
Results
4.Communicate
Business Analysis
Information
5.Manage Stakeholder
Collaboration
ELICITATION AND COLLABORATION KNOWLEDGE AREA (contd.)
1. Prepare for Elicitation
2. Conduct Elicitation
4.Communicate
Business Analysis
Information
5.Manage Stakeholder
Collaboration
3. Confirm Elicitation
Results
Tasks
Inputs
• Needs
• Business Analysis
Information
Input from other
Knowledge Areas
• Stakeholder
Engagement Approach
• Business Analysis
Performance
Assessment
TASKS AND OUTPUT
Sequential tasks
ELICITATION AND COLLABORATION KNOWLEDGE AREA (contd.)
1. Prepare for Elicitation
2. Conduct Elicitation
4.Communicate
Business Analysis
Information
5.Manage Stakeholder
Collaboration
3. Confirm Elicitation
Results
Tasks Outputs
Inputs
• Needs
• Business Analysis
Information
Input from other
Knowledge Area
• Stakeholder
Engagement Approach
• Business Analysis
Performance
Assessment
Elicitation Activity plan
Elicitation Results
(unconfirmed)
Business Analysis
Information (communicated)
Stakeholder Engagement
Elicitation Results
(confirmed)
TASKS AND OUTPUT
 Overview
 Elements
 GD
uie
dfe
elc
in
te
p
s
ea
r n
U
d
ni
T
tools
 Techniques
 Stakeholders
Lesson 4: Elicitation and Collaboration
Topic4.1:PrepareforElicitation
PREPARE FOR ELICITATION
OVERVIEW
Purpose
 Understand the scope of elicitation activity
 Select appropriate technique
 Plan for supporting material and resources
I
N
P
U
T
S
O
U
T
P
U
T
S
 Needs
 Stakeholder
Engagement Approach
Elicitation Activity Plan:
 Logistics
 Scope of the elicitation
activity
 Selected techniques
 Supporting Material
PREPARE FOR ELICITATION
ELEMENTS
Understand Scope
Select Technique
Set up Logistics
Secure Supporting
Material
Prepare Stakeholders


PREPARE FOR ELICITATION
GUIDELINES AND TOOLS
Business Analysis Approach Business Objectives
Potential Value
Existing Business Analysis
PREPARE FOR ELICITATION (contd.)
GUIDELINES AND TOOLS
Business Analysis Approach Business Objectives
Potential Value
Existing Business Analysis
Provides a general strategy
to be used as a guide
Provides directions towards
future state
Describes the value to be
realized
Provides better
understanding of scope
PREPARE FOR ELICITATION
TECHNIQUES
Stakeholder List, Map, or
Personas
Brainstorming
Mind Mapping
Data Mining
Estimation
Interviews
Risk Analysis and
Management
Document Analysis
Stakeholder List, Map or
Personas
Brainstorming
Mind Mapping
Data Mining
Estimation
Interviews
Risk Analysis and
Management
Document Analysis
PREPARE FOR ELICITATION (contd.)
TECHNIQUES
Who needs to be
consulted?
Identify information
for further
investigation
Identify existing
business analysis
information
Select elicitation
technique
Estimate time,
effort and cost
Identify concern
Identify source of
supporting material
Identify and assess
risk
PREPARE FOR ELICITATION
STAKEHOLDERS
Domain SME
Project Manager
Sponsor
Business Analyst
PREPARE FOR ELICITATION (contd.)
STAKEHOLDERS
Ensures the
appropriate
people and
resources
are available
Approves
or denies a
planned
elicitation
event
Provides
supporting
material
and
guidance.
Domain SME
Sponsor
Business Analyst
Project Manager
 Overview
 Elements
 GD
uie
dfe
elc
in
te
p
s
ea
r n
U
d
ni
T
tools
 Techniques
 Stakeholders
Lesson 4: Elicitation and Collaboration
Topic4.2:ConductElicitation
CONDUCT ELICITATION
OVERVIEW
Purpose
Information relevant to change:
 Draw out
 Explore
 Identify
I
N
P
U
T
S
O
U
T
P
U
T
 Supporting
material
 Resources
 Activity Plan
Unconfirmed Elicitation Results
CONDUCT ELICITATION (contd.)
OVERVIEW
 Interacts with the stakeholder
 Relies on the stakeholders’
experience
 Discovers information from
documents
 Stakeholders may not know
about this information
 Discovers information
without stakeholders or
documents
 Prototyping or Proof of
concept
Collaborative Research Experiments
CONDUCT ELICITATION
ELEMENTS
Guide the elicitation
session
Capture elicitation results
Engage someone else to
scribe
CONDUCT ELICITATION
GUIDELINES AND TOOLS
Business Analysis Approach
Existing Business Analysis
Information
Supporting Materials
Stakeholder Engagement
Approach
CONDUCT ELICITATION (contd.)
GUIDELINES AND TOOLS
Influences elicitation
techniques and business
analysis deliverables
Guides questions to be
asked during elicitation
Prepares the business
analyst, participants and
equipment for elicitation
Provides collaboration and
communication approach for
elicitation
Business Analysis Approach
Existing Business Analysis
Information
Supporting Materials
Stakeholder Engagement
Approach
CONDUCT ELICITATION
TECHNIQUES
Benchmarking
and Market
Analysis
Document
Analysis
Observation
Brainstorming
Collaborative
Games
Workshops
Focus Groups
Survey and
Questionnaire
Interface
Analysis
Interviews
Mind Mapping
Prototyping Data Mining
Concept
Modeling
Process
Modeling
Data Modeling
Business
Rules Analysis
Process
Analysis
CONDUCT ELICITATION
DOCUMENT ANALYSIS - OVERVIEW
Limitations
Strengths
 Elicit business analysis information
 Understand the context of a
business need
 Understand the solution currently
being implemented
 Validate findings from other
elicitation efforts
 Engage effectively with
stakeholders
 Wide range of sources make it
time consuming
 Outdated documents may
cause confusion
CONDUCT ELICITATION
DOCUMENT ANALYSIS - ELEMENTS
Preparation
Document Review
and Analysis
Record Findings

☑
Source document:
• Detailed review and recording
notes
• Identify conflicting or duplicated
notes
Source document:
• Relevant, current and has
data or information required
• Appropriate level of
detail
• Present information to
improve understanding
CONDUCT ELICITATION
OBSERVATION - OVERVIEW
Limitations
Strengths
 View and understand activities
 Job shadowing
 Approaches:
 Active / Noticeable
 Passive / Unnoticeable
 Gain Realistic and practical
insights
 May be disruptive to the day-to-
day operations
 Participants may alter their work
practices
 Unsuitable for knowledge based
activities
CONDUCT ELICITATION
OBSERVATION - ELEMENTS
Define the objectives of the
observation
Prepare for observation
Conduct observation
session
Confirm and present
observation session
findings

☑
CONDUCT ELICITATION
BRAINSTORMING - OVERVIEW
Strengths
 Produce numerous ideas and a
diverse set of options in a short
span of time
 Focus on the topic or problem
 Encourage participants to use new
ways
 Foster creativity with a non-
judgmental environment
Limitations
 Effectiveness depends on:
 Individual creativity
 Willingness to participate
CONDUCT ELICITATION
BRAINSTORMING - ELEMENTS
Preparation
Session
Wrap-Up
 Define area of interest
 Determine time limit
 Identify participants
 Establish evaluation
criteria
 Re-iterate session
details
 Share ideas
 Record ideas
 Build on each other
ideas
 Elicit many ideas
 Discuss and evaluate
 Create list
 Rate ideas
 Distribute final list
📋
CONDUCT ELICITATION
COLLABORATIVE GAMES - OVERVIEW
Strengths Limitations
 Encourages collaboration in
building common understanding
 Follows adaptive or agile
methodology
 Sets rules to focus on objectives
 Game environment may make some
participants uncomfortable
 Time consuming
 Unproductive if objectives are
unclear
CONDUCT ELICITATION
COLLABORATIVE GAMES - ELEMENTS
Purpose Process Outcome
Opening
Exploration
Closing
Learn Rules
Generate ideas
Assess and Select idea
?  
🖝
🞿

CONDUCT ELICITATION (contd.)
COLLABORATIVE GAMES - ELEMENTS
Product
Product Box
Identifyfeaturesofaproduct
Affinity Map
Identifyrelatedorsimilarfeatures
Fishbowl
Identifyhiddenassumptions
CONDUCT ELICITATION
WORKSHOPS - OVERVIEW
Strengths
 Most common technique
 Focused event
 Collaboration to achieve
predefined goals
 Make decisions and gain mutual
understanding
 Requires short period of time
 Requires low cost
Limitations
 Availability of stakeholders difficult
 Success depends on facilitator’s
knowledge of participants
 Too many participants slows down
the process
 Too few participants can lead to
overlooking of needs
CONDUCT ELICITATION
WORKSHOPS - ELEMENTS
Prepare
Identify roles
Conduct
Post workshop
Distribute outcome
o Sponsor
o Facilitator
o Scribe
o Timekeeper
o Participants
?
CONDUCT ELICITATION
FOCUS GROUPS - OVERVIEW
Strengths
 Interactive group environment
 Pre-qualified participants
 Homogenous or Heterogeneous
group
 Qualitative research
 Cost effective
Limitations
 Trust issues in participants
 More vocal participants may
influence the result
CONDUCT ELICITATION
FOCUS GROUPS- ELEMENTS
Define objectives
Plan
Prepare guide
Assign moderator and
recorder
Conduct post-discussion
activities
o Location
o Budget
o Timelines
o Outcomes
o Participants
?
After the focus group –
Transcribe the results as
soon as possible
CONDUCT ELICITATION
SURVEY AND QUESTIONNAIRE - OVERVIEW
Limitations
Strengths
 Distribution, collection and analysis
 Elicit business information about
customer, product, work practices
and attitudes in a structured way
and in a short period of time
 Quick and relatively inexpensive to
administer
 Effective and efficient across
geographies
 Closed and open ended questions
 Response may be low for
statistical significance
 Ambiguous questions not
answered
 Open-ended questions need
more analysis
CONDUCT ELICITATION
SURVEYS AND QUESTIONNAIRES - ELEMENTS
Prepare the survey or
questionnaire
Define
Select
Assign moderator and
recorder
Conduct and Post
discussion activities
o Objective
o Target group
o Appropriate survey type
o Sample group
o Distribution and collection
method
o Set target level and
response time
o Determine if individual
interviews are required
 






CONDUCT ELICITATION
PROTOTYPING - OVERVIEW
Strengths
 Iterative process
 Creation of a model or design
requirements
 Optimizes user experience to
evaluate design options
 Basis of the final business solution
 Identifies missing and incorrect
requirements
 Early inputs and feedback
Limitations
 Stakeholders may develop
unrealistic expectations
 Stakeholder may focus on the
design of the solution
 Focus of discussions may be on
how are we going to build rather
than what
 Underlying technology needs to
be understood
CONDUCT ELICITATION
PROTOTYPING - ELEMENTS
Determine Approach
Examples
Methods
Throw-away prototypes
are discarded once the
learning from the
developed prototype
and purpose are
achieved.
Evolutionary prototype
is incrementally built to
the final solution.
Proof of Concept Usability prototype Visual prototype
Functional
prototype
Storyboarding Paper prototyping Workflow modeling Simulation
CONDUCT ELICITATION
INTERFACE ANALYSIS - OVERVIEW
Where
What Why When
How
An Interface is a
connection between two
components or
solutions. Most
solutions require one or
more interfaces.
?
? ? ?
?
Identify
CONDUCT ELICITATION (contd.)
INTERFACE ANALYSIS - OVERVIEW
ATM
Laptop
Mobile
• External interface to the
solution
• Business processes
• Data interface between
systems
• Application Programming
Interfaces (APIs)
• Hardware devices
CONDUCT ELICITATION (contd.)
INTERFACE ANALYSIS - OVERVIEW
Understand interfaces used
in existing business
processes
Provide context for eliciting
more detailed functional
requirements
Use scope modelling,
observation and interviews
Provide interface of the
solution with a context
diagram
CONDUCT ELICITATION (contd.)
INTERFACE ANALYSIS - ELEMENTS
Prepare for identification
Conduct identification
Define
 Name
 Coverage
 Exchange method
 Message format
 Exchange frequency
?
CONDUCT ELICITATION
INTERVIEWS - OVERVIEW
Definition
Types
Success
A systematic approach to elicit business analysis information by asking relevant questions and
documenting the responses from a person or a group of people. One-on-one interviews are the
most common. Interviews can be synchronous, asynchronous, conducted face-to-face or
through video conferencing.
• Structured interview - Questions are pre-defined
• Unstructured interview - Questions may vary based on the interviewee
response
• Interviewers - Domain knowledge, experience and skills for documenting the discussion
• Interviewees - Readiness, degree of clarity about the goal of the interview
• Rapport of the interviewer with the interviewee
Constraint Significant time is required to plan and conduct interviews.
CONDUCT ELICITATION
INTERVIEWS - ELEMENTS
Follow-up
Plan Execute
Define goals
Communicate
goals
Identify
interviewees
Design
questions
Questions:
5W + 1H
Open-ended
questions
Closed-ended
questions
Arrange
logistics
Flow: Open –
During- Close
Organize and
confirm
results

CONDUCT ELICITATION
MIND MAPPING - OVERVIEW
Used to
Articulate
MIND MAPPING
Note
taking
T
axonom
y
Capture
ideas
Main Topic
T
opic
Sub-Topic
Branches
Color
Images
Non-linear
Structure
and logic
CONDUCT ELICITATION
MIND MAPPING - ELEMENTS
CONDUCT ELICITATION
STAKEHOLDERS
Sponsor
Implementation
SMEs
Domain SMEs
Other
Stakeholder
Customer End User
Business Analyst
 Overview
 Elements
 GD
uie
dfe
elc
in
te
p
s
ea
r n
U
d
ni
T
tools
 Techniques
 Stakeholders
Lesson 4: Elicitation and Collaboration
Topic4.3:ConfirmElicitationResults
CONFIRM ELICITATION RESULTS
OVERVIEW
Purpose
 Check gathered information
 Confirm accuracy and consistency with other information
I
N
P
U
T
O
U
T
P
U
T
 Elicitation Results
(Unconfirmed)
 Elicitation Results (Confirmed)
CONFIRM ELICITATION RESULTS
ELEMENTS
Confirmed elicitation
results
With other
elicitation
results
Against source
information
 
☑
CONFIRM ELICITATION RESULTS
GUIDELINES AND TOOLS
Elicitation Activity Plan Existing Business Analysis
CONFIRM ELICITATION RESULTS
TECHNIQUES
Document Analysis

Reviews
Interviews
Workshops
☑
CONFIRM ELICITATION RESULTS (contd.)
TECHNIQUES
Document Analysis
Reviews
Interviews
Workshops
• Confirm elicitation results with
existing documents
• Conduct review of elicitation
results and solicit feedback

☑
CONFIRM ELICITATION RESULTS
STAKEHOLDERS
Knowledge
Experience
Expertise
Domain SME Any Stakeholder
 Overview
 Elements
 GD
uie
dfe
elc
in
te
p
s
ea
r n
U
d
ni
T
tools
 Techniques
 Stakeholders
Lesson 4: Elicitation and Collaboration
Topic4.4:CommunicateBusinessAnalysisInformation
COMMUNICATE BUSINESS ANALYSIS INFORMATION
OVERVIEW
Purpose
 Stakeholders have a shared understanding of business analysis information
I
N
P
U
T
S
O
U
T
P
U
T
 Business
Analysis
Information
 Stakeholder
Engagement
Approach
 Business Analysis Information
(communicated)
COMMUNICATE BUSINESS ANALYSIS INFORMATION
ELEMENTS
Form of Package
Depends on the business
analysis approach
 Predictive
 Adaptive
May be:
 Formal documentation
 Information documentation
 Presentation
☑

 Communicate
business analysis
package
Determine objectives
and format of
communication


COMMUNICATE BUSINESS ANALYSIS INFORMATION
GUIDELINES AND TOOLS
Business Analysis Approach Information Management
COMMUNICATE BUSINESS ANALYSIS INFORMATION
TECHNIQUES
Interviews
Reviews
Workshops
☑
COMMUNICATE BUSINESS ANALYSIS INFORMATION (contd.)
TECHNIQUES
Communicate
information
Review business
analysis
information
Build consensus
and seek approval
Interviews
Reviews
Workshops
☑
COMMUNICATE BUSINESS ANALYSIS INFORMATION
STAKEHOLDERS
End User
Customer
T
ester
Domain
SME
Implementation
SME
Common Understanding
All
Stakeholders
 Overview
 Elements
 GD
uie
dfe
elc
in
te
p
s
ea
r n
U
d
ni
T
tools
 Techniques
 Stakeholders
Lesson 4: Elicitation and Collaboration
Topic4.5:ManageStakeholderCollaboration
MANAGE STAKEHOLDER COLLABORATION
OVERVIEW
Purpose
 Encourage stakeholders to work towards a common goal
I
N
P
U
T
S
O
U
T
P
U
T
 Stakeholder
Engagement Approach
 Business Analysis
Performance
Assessment
 Stakeholder engagement
MANAGE STAKEHOLDER COLLABORATION
ELEMENTS
☑ Gain agreements on
commitments
Monitor Stakeholder
Engagement

Collaboration
 Monitor the participation and
performance of stakeholders
 Monitor for risk related to
stakeholder engagements
 Maintain the free flow of
information
 Promote shared effort to
resolve problems
 Achieve desired results
 Time commitment
 Resource commitment
MANAGE STAKEHOLDER COLLABORATION
GUIDELINES AND TOOLS
Business Analysis Approach Business Objectives
Recommended Actions
Future State Description Risk Analysis
 
MANAGE STAKEHOLDER COLLABORATION (contd.)
GUIDELINES AND TOOLS
Describes the level of
engagement
Business Analysis Approach Business Objectives
Recommended Actions
Future State Description
Provides action to earn the
support and focus of
stakeholder
Defines the desired future
state and expected value
Provides action to ensure
stakeholder collaboration
Provides directions towards
future state
Risk Analysis
 
MANAGE STAKEHOLDER COLLABORATION
TECHNIQUES
Collaborative Games
Lessons Learned
Risk Analysis and
Management
Stakeholder list, Map or
Personas
MANAGE STAKEHOLDER COLLABORATION (contd.)
TECHNIQUES
T
eamwork and
collaboration
Stakeholder satisfaction
or dissatisfaction
Stakeholder
participation and
engagement risk
Who needs to be
involved?
Collaborative Games
Lessons Learned
Risk Analysis and
Management
Stakeholder list, Map or
Personas
MANAGE STAKEHOLDER COLLABORATION
STAKEHOLDERS
All Stakeholders involved during change
Sponsor
Customer
End User
Implementation
SMEs
Domain
SMEs
Other
Stakeholder
 Defect per Unit
Lesson 4: ELICITATION AND COLLABORATION
CASESTUDYEXERCISE
CASE STUDY
PROBLEM STATEMENT
BATONICS
Pharma Company
Unable to deliver
products on time
Customers switching
to competitor products
CASE STUDY
ACTIVITIES
The business analyst performed the following activities:

Spentsometimegoing
throughexistingdocuments
Createdahighlevelcontext
diagram
Createdalistofquestions
forstakeholdersto
understandtheirneeds
Scheduledacoupleof
meetingswithstakeholders

Documentedall thecollected
informationandorganizedandanalyzed
theinformation
CASE STUDY
EXERCISE
Questions Response
1 Which technique has Paul used to create
supporting material like a list of questions,
As-Is process or any other?
o Document Analysis
o Interviews
2 What Business Analysis Approach is used
to build the proposed system?
o Adaptive
o Restrictive
3 Which elicitation technique is used to
design screens to capture customer
information?
o Prototyping
o Group Discussion
4 Which elicitation technique is used to
understand the flow of information
between systems and users?
o Interface Analysis
o Workshops
CASE STUDY
EXERCISE
Questions Response
5 Who is the end user of the proposed
system?
o Field Sales Agents
o Underwriters
6 Which technique is used to understand how
stakeholders are interacting in the process
to complete their tasks or achieve goals?
o Process Analysis
o Customer Analysis
7 Who will be using the saved information of
the customer and the quote?
o Underwriters
o Stakeholder
CASE STUDY
ANSWERS
Questions Answers
1 Which technique has Paul used to create supporting material
like a list of questions, As-Is process or any other etc.?
Document Analysis
2 What Business Analysis Approach is used to build the a
proposed system?
Adaptive
3 Which elicitation technique is used to design screens to
capture customer information?
Prototyping
4 Which elicitation technique is used to understand the flow of
information between systems and users?
Interface Analysis
5 Who is the end user of the proposed system? Field Sales Agents
6 Which technique is used to understand how stakeholders are
interacting in the process to complete their tasks or achieve
goals?
Process Analysis
7 Who will be using the saved information of the customer and
the quote?
Underwriters
KEY TAKEAWAYS
Elicitation is defined as “to draw forth or bring out”
information.
There are 18 commonly used Elicitation
techniques.
There are 3 tasks associated with Elicitation, 1
task with Communicate Business Analysis
Information, and 1 task with Manage Stakeholder
Collaboration.
Select the appropriate technique or set of
techniques based on the need and Business
Analysis Approach.
Measure performance of Elicitation activities
throughout the project.
Each Elicitation technique has the following tasks
- prepare, execute or conduct, and wrap-up or
close tasks.
Frequently communicate Business Analysis
Information and Manage Stakeholder
Collaboration and communication during
Elicitation activities.
Quiz
a.
b.
c.
d.
In which elicitation technique is Discussion Guide used?
QUIZ
1
Workshops
Brainstorming
Focus Groups
Interviews
The correct answer is b.
Explanation: Discussion Guide is used in Focus Group Sessions. Discussion Guide contains
session objectives, topics for discussion, and scripts of specific questions.
a.
b.
c.
d.
In which elicitation technique is Discussion Guide used?
QUIZ
1
Interviews
Focus Groups
Brainstorming
Workshops
a.
b.
c.
d.
Which of the following elicitation techniques uses survey distribution and response
collection?
QUIZ
2
Interviews
Surveys and Questionnaires
Concept Modeling
Workshop
The correct answer is c.
Surveys and Questionnaires is a technique in which first the surveys are distributed and then
responses are collected for further analysis.
a.
b.
c.
d.
Which of the following elicitation technique uses survey distribution and response
collection?
QUIZ
2
Workshop
Concept Modeling
Surveys and Questionnaires
Interviews
a.
b.
c.
d.
Which one of the following is not an output element of the task ‘Prepare for
elicitation’?
QUIZ
3
Elicitation results
Elicitation technique
Participant list
Supporting material
The correct answer is d.
Explanation: Elicitation results is not an output element of the task ‘Prepare for elicitation’.
.
a.
b.
c.
d.
Which one of the following is not an output element of the task ‘Prepare for
elicitation’?
QUIZ
3
Supporting material
Participant list
Elicitation technique
Elicitation results
a.
b.
c.
d.
Which one of the following is not a common type of elicitation?
QUIZ
4
Exploratory
Experiments
Research
Collaborative
The correct answer is d.
Explanation: Exploratory is not a common type of elicitation. As per BABOK TM, there are
three common types of elicitation – Collaborative, Research, and Experiments.
a.
b.
c.
d.
Which one of the following is not a common type of elicitation?
QUIZ
4
Collaborative
Research
Experiments
Exploratory
This concludes 0Elicitation and Collaboration”
Thenextlessonis“RequirementsLifeCycleManagement”
THANKYOU
Lesson5- RequirementsLifeCycleManagement
CBAP®Exam Preparation Course
CBAP, CCBA and BABOK are registered certification marks owned by International Institute of Business Analysis.
WHAT’S IN IT FOR ME
Trace requirements
Maintain requirements
Prioritize requirements
Assess changes to
requirements
Approve requirements
INTRODUCTION
Business Analyst
 📋    Requirements
Information
• At start: The status of elicitation
results is unconfirmed
• Confirm elicitation results: The
results are confirmed
• Update status of elicitation results
i.e. requirements or designs:All
other knowledge areas
Elicitation and Collaboration
• Know the origin of the requirement or
design.
• Know whether one requirement is related
or has dependency with other
requirements.
• Know which solution components fulfill the
requirements.
• Know which test cases will verify the
functionality of the solution.
• Know what will help you in assessing
changes to the requirements.
To update Elicitation results status
INTRODUCTION
Business Analyst
Trace requirements Prioritize requirements
Reuse requirements Assess requirements
Approve requirements Allocate requirements
Changes are inevitable
🞿
• The requirements need to be maintained throughout the project life cycles.
CThhaengresuairreeminenvtitsanbelee,dhteonbcey
m
o
a
u
i
n
n
t
e
a
e
i
n
d
e
t
d
o
t
a
h
s
r
s
o
e
u
s
g
s
h
c
o
h
u
a
t
n
t
g
h
e
e
sp
t
o
r
o
t
jh
eec
t
r
el
i
q
f
eu
c
i
ry
ec
m
l
ees
n
.ts
and design, and recommend appropriate actions
• C
C
h
h
a
a
n
n
g
g
e
e
s
s
t
o
t
o
t
h
t
h
e
e
r
e
r
e
q
q
u
u
i
r
i
e
r
e
m
m
e
e
n
n
t
s
t
s
a
a
n
n
d
d
t
h
t
h
e
e
a
a
p
p
p
p
r
o
r
o
v
v
a
a
llppr
or
occeessd
d
e
e
p
p
e
e
n
n
d
d
u
u
p
p
o
o
n
n
t
h
t
h
e
e
p
p
r
o
r
o
j
e
j
e
c
c
t
t
m
m
e
e
t
h
t
h
o
o
d
d
o
o
l
o
l
o
g
g
y
y
🞿
INTRODUCTION
Business Analyst
Maintain
requirements
Trace
requirements
Prioritize
requirements
Approve
requirements
The Requirements Life
Cycle Management
knowledge area describes
the tasks that the business
analysts performs in order to
manage and maintain the
requirements and design
information from the point of
inception till retirement.
REQUIREMENTS LIFE CYCLE MANAGEMENT
OVERVIEW
Change
Solution
Need
Stakeholder Context
Value
REQUIREMENTS LIFE CYCLE MANAGEMENT
OVERVIEW
Maintain requirements for reuse
Support tracking and prioritization
activities
Work closely with key stakeholders
Trace requirements and designs to
solution components
Trace, prioritize and maintain
requirements
Evaluate requirements and designs
Change
Solution
Need
Stakeholder Context
Value
REQUIREMENTS LIFE CYCLE MANAGEMENT
TASKS AND OUTPUT
Tasks
Trace requirements
Maintain requirements
Prioritize requirements
Assess requirements
changes
Approve requirements
ELICITATION AND COLLABORATION KNOWLEDGE AREA
TASKS AND OUTPUT
Inputs
• Requirements
• Designs
Tasks
Trace requirements
Maintain requirements
Prioritize requirements
Assess requirements changes
Approve requirements
Outputs
Requirements
• Traced, maintained,
and prioritized
Designs
• Traced, maintained,
and prioritized
ELICITATION AND COLLABORATION KNOWLEDGE AREA
TASKS AND OUTPUT
Inputs
• Requirements
• Business Analysis
Information
• Proposed changes
Tasks
Trace requirements
Maintain requirements
Prioritize requirements
Assess requirements changes
Approve requirements
Outputs
• Requirements
o Traced, maintained,
and prioritized
• Designs
o Traced, maintained,
and prioritized
• Requirements Change
Assessment
• Design Change
Assessment
ELICITATION AND COLLABORATION KNOWLEDGE AREA
TASKS AND OUTPUT
Inputs
• Requirements
• Business Analysis
Information
• Proposed changes
Input from other
Knowledge Areas
• Requirements (verified)
Tasks
Trace requirements
Maintain requirements
Prioritize requirements
Assess requirements changes
Approve requirements
Outputs
Requirements
• Traced, maintained,
and prioritized
Designs
• Traced, maintained,
and prioritized
• Requirements Change
Assessment
• Design Change
Assessment
• Requirements approved
• Design approved
What is traceability?
 Overview
 Elements
 GD
uie
dfe
e
lc
in
te
p
s
ea
r n
U
d
ni
T
tools
 Techniques
 Stakeholders
Lesson 5: Requirements Life Cycle Management
Topic5.1:TraceRequirements
TRACE REQUIREMENTS
OVERVIEW
Purpose
 Ensure requirements and designs at different levels are aligned to one another
 Manage effects of change
I
N
P
U
T
S
O
U
T
P
U
T
S
 Requirements
 Designs
 Traced requirements
 Traced designs
TRACE REQUIREMENTS
ELEMENTS
Low level
Origin High level
Backward traceability Forward traceability
Consider:
• Value that each traceability link
delivers
• Use of specific relationship
created
!
Effort increases with:
• Number of requirements and
• Level of formality
TRACE REQUIREMENTS
ELEMENTS
Low level
Origin High level
Backward traceability Forward traceability
Consider:
• Value that each traceability link
delivers
• Use of specific relationship
created
!
Effort increases with:
• Number of requirements and
• Level of formality
Validate
Satisfy

 Depend
One requirement is dependent
on another requirement
Derive
🖝
Implementation of test cases
Solution components are
implemented
One requirement is derived
from another requirement
TRACE REQUIREMENTS
GUIDELINES AND TOOLS


 Legal or regulatory
information
Domain knowledge
Information
management approach
Requirements
management tool or
repository
TRACE REQUIREMENTS
TECHNIQUES
Business rules analysis Functional decomposition
Scope modelling
Process modelling
TRACE REQUIREMENTS
TECHNIQUES
Business rules analysis Functional decomposition
Scope modelling
Process modelling
Decompose solution
scope into smaller
components
Trace the requirements to
the business rules
Show requirements tracing
to future state process
Show the scope of the
business analysis work
TRACE REQUIREMENTS
STAKEHOLDERS
Business Analyst
Domain
SME
Implementation
SME
Operational
support team
Sponsor
End user
Supplier
T
ester
Customers
Project Manager
What is requirements maintenance?
 Overview
 Elements
 GD
uie
dfe
e
lc
in
te
p
s
ea
r n
U
d
ni
T
tools
 Techniques
 Stakeholders
Lesson 5: Requirements Life Cycle Management
Topic5.2:MaintainRequirements
MAINTAIN REQUIREMENTS
OVERVIEW
Purpose
 Maintain accuracy and consistency of requirements
 Support reuse of requirements
I
N
P
U
T
S
O
U
T
P
U
T
S
 Requirements
 Designs
 Maintained requirements
 Maintained designs
MAINTAIN REQUIREMENTS
ELEMENTS
Correctness, accuracy, relationship
etc.
Maintainattributes
Source, category, requested by,
created date, updated date,
priority, complexity, status, target
date, reusability etc.
Reusedrequirements
Clearly label, define, and store


Maintainrequirements
MAINTAIN REQUIREMENTS
GUIDELINES AND TOOLS
 Information
management approach
Business Analyst
Manage
requirements for
reuse
☑
MAINTAIN REQUIREMENTS
TECHNIQUES
Business rules
analysis
Data flow
diagrams
Data modeling
Process
modeling
Use cases and
scenarios
User stories
Document
analysis
Functional
decomposition
MAINTAIN REQUIREMENTS
TECHNIQUES
Business rules
analysis
Data flow
diagrams
Data modeling
Process
modeling
Use cases and
scenarios
User stories
Document
analysis
Functional
decomposition
Identify which business rules,
information flows, data structures,
processes and requirements are
reusable.
Analyze existing document to
provide a basis for
maintaining and reusing
requirements
Identify requirements
associated with components
and available for reuse
MAINTAIN REQUIREMENTS
STAKEHOLDERS
Business Analyst
Domain
SME
Operational
support team
Regulators
T
ester
What is prioritizing requirements?
 Overview
 Elements
 GD
uie
dfe
e
lc
in
te
p
s
ea
r n
U
d
ni
T
tools
 Techniques
 Stakeholders
Lesson 5: Requirements Life Cycle Management
Topic5.3:PrioritizeRequirements
PRIORITIZE REQUIREMENTS
OVERVIEW
Purpose
 To rank requirements in the order of relative importance to maximize value delivery
I
N
P
U
T
S
O
U
T
P
U
T
S
 Requirements
 Designs
 Prioritized requirements
 Prioritized designs
PRIORITIZE REQUIREMENTS
ELEMENTS
Factors Challenges
Process
? 

Benefit Dependencies
Time
sensitivity
Stability
Regulatory or
policy compliance
Risk
Cost
Penalty
Ongoing
Relative value Stakeholders
PRIORITIZE REQUIREMENTS
GUIDELINES AND TOOLS
Repository
Solution scope
Business constraints
Change strategy
Domain knowledge
Governance approach
Requirements architecture
Requirements Management Tools /

PRIORITIZE REQUIREMENTS
TECHNIQUES
Backlog management
Business cases
Decision analysis
Estimation
Interviews
Workshops
Risk analysis and
management
Item tracking
Financial analysis Prioritization


☑
PRIORITIZE REQUIREMENTS
TECHNIQUES
Benefit
Cost
Penalty
Risk
Assign weights (1 to 9) to factors benefit, penalty, cost and risk associated to a feature.
Derive the total value using the formula as defined by Karl Weigers in relative weighting process.
PRIORITIZE REQUIREMENTS
BACKLOG MANAGEMENT - OVERVIEW
Backlog  Record
 Track
 Prioritize remaining work items
Items at the top have the highest
business value and priority
Backlog
management
• What work should be formally
included
• How to describe the work item
• How to track the work items
• How to periodically review and
prioritize the work items in
relation to all other items
• How to select the work items to
work on
• How to remove the work items
PRIORITIZE REQUIREMENTS
BACKLOG MANAGEMENT - ELEMENTS
Items in the backlog
Prioritization
Estimation
Managing changes to
backlog
Use cases, user stories, change requests, defects, maintenance
work records, documentation work records, functional
requirements, non-functional requirements.
New or changed requirements are
identified and added to the backlog and
ordered relative to the other items
already there.
Always emerging;
Items are removed when they are
completed or it is decided not to
work on the item.
PRIORITIZE REQUIREMENTS
ESTIMATION - OVERVIEW
 Forecast cost
and effort Support
decision
making
Iterative
process
Communicate
constraints
Communicate
assumptions
PRIORITIZE REQUIREMENTS
ESTIMATION - ELEMENTS
T
op down Bottom up
Parametric
estimation
Rough order of
estimation
(ROM)
Rolling wave Delphi
Three-point
estimation
Program
evaluation and
review
• Accuracy
• Source
• Precision
• Reliable
• Deterring
PRIORITIZE REQUIREMENTS
TOP DOWN Vs BOTTOM UP
Analogous estimation and expert
judgment
Used when quick estimates are required
with low level of accuracy
Fast
No need to decompose work in greater
detail
Less costly to create and
Good for similar projects
Low level of accuracy
Requires expertise and historical data
Three point, Parametric or Analogous
estimation
Estimate require higher level of accuracy
Higher level of accuracy
Better team buy-in and commitment
Best for complex projects
Provides more accurate baseline to track.
Takes more time and incurs more
expenses
Work needs to be defined in great detail
PRIORITIZE REQUIREMENTS
THREE POINT AVERAGE AND PERT ESTIMATION
Optimistic – Most likely – Pessimistic
Optimistic – Best case
Pessimistic – Worst case
Triangular distribution:
Estimate based on the three points,
optimistic, most likely and pessimistic
Final estimate is the average of all three
estimates
PERT = Program (or Project) Evaluation
and Review Technique
Uses weighted average of three points
It’s more accurate than 3-point average
PERT is also referred as a “Beta
Distribution” OR “Double Triangular
Distribution”
PRIORITIZE REQUIREMENTS
THREE POINT AVERAGE AND PERT ESTIMATION
Optimistic: 3 days, Most Likely: 5 days and Pessimistic:
10 days
Then the final estimate, with averaging formula is 6
days
Applying weighted average formula the final
estimate = 5.5 days
Standard deviation = 1.167
Optimistic
3
Most
likely
Pessimistic
3
3
5 10
Optimistic
6
Pessimistic
4* Most
likely
Pessimistic
6
Optimistic
THREE POINTAVERAGE PERT ESTIMATION
PRIORITIZE REQUIREMENTS
PARAMETRIC AND ROLLING WAVE ESTIMATION
Uses a mathematical model.
Extremely accurate
Reduces overall risk
Based on historical information
Takes time,
Requires expensive tools,
Requires considerable experience
Difficult for calculating “soft” costs
Difficult for broadly defined projects
Iterative estimation
Rough Order of Magnitude (ROM)
estimate for overall initiative or project.
High level of accuracy - Activities performed
are in the near term
Low level of accuracy – Activities performed
are in the longer duration
This estimation approach is used in change
driven plans or adaptive or agile plans, also
when planning is iterative, and hence is
referred to as rolling wave planning.
PRIORITIZE REQUIREMENTS
PRIORITIZATION - OVERVIEW
Backlog
management
Provides a framework for the BusinessAnalyst:
• To facilitate stakeholder decisions
• To understand the relative importance of business analysis information
Importance of business analysis information is based on:
Value, Risk, Difficulty of implementation
Business analyst revisit priorities when changes occur in the business environment, stakeholder
needs
PRIORITIZE REQUIREMENTS
PRIORITIZATION - APPROACHES
Grouping
Ranking
Business analysis information is classified into
categories such as, high, medium and low
priority.
Business analysis information is ordered based
on relative importance.
Business analysis information is prioritized based
on the amount of work a project team can deliver
in given time or budget.
Requirements are prioritized based on the
consensus among stakeholders.
 Time boxing or
Budgeting
Negotiation
🞿
PRIORITIZE REQUIREMENTS
ITEM TRACKING - OVERVIEW
Backlog
management
• Item tracking is used to capture and assign responsibility for issues and stakeholder concerns
• Viable stakeholder concern is classified into specific item types
• Item types are Actions,Assumptions, Constraints, Dependencies, Defects, Enhancements,
and Issues
• Items are assigned to one or more stakeholders who are responsible for its resolution
• Item tracking may be shared with stakeholders to ensure transparency and visibility into the
status
PRIORITIZE REQUIREMENTS
ITEM TRACKING - ELEMENTS
Item-identifier
Summary or
Description
Category
Type
Date identified
Identified by
Impact
Priority
Resolution date
Owner
Resolver or
assigned to
Agreed strategy
Status
Resolution
updates
Escalation matrix
PRIORITIZE REQUIREMENTS
STAKEHOLDERS
Customer Sponsor
Regulator
End user
Implementation
SMEs
Project manager
Business Analyst
How changes to the requirements are assessed?
 Overview
 Elements
 GD
uie
dfe
e
lc
in
te
p
s
ea
r n
U
d
ni
T
tools
 Techniques
 Stakeholders
Lesson 5: Requirements Life Cycle Management
Topic5.4:AssessRequirementsChanges
ASSESS REQUIREMENTS CHANGES
OVERVIEW
Purpose
 Evaluate the implications of proposed changes to requirements and designs
I
N
P
U
T
S
O
U
T
P
U
T
S
 Proposed changes
 Existing
requirements
 Existing designs
 Requirements change assessment
 Designs change assessment
ASSESS REQUIREMENTS CHANGES
ELEMENTS
Predictive
Approach
Adaptive
approach
Assessment
formality
Impact
analysis
Review related requirements
and components
Assess the impact by considering:
• Benefit
• Cost
• Schedule
• Impact
• Urgency
Impact resolution
• All proposed changes need to be approved, rejected or
deferred
• All impacts and resolutions are documented and
communicated to the stakeholders.
ASSESS REQUIREMENTS CHANGES
GUIDELINES AND TOOLS
 Domain knowledge
Legal or regulatory
information
Solution scope
Requirements
architecture
Governance
approach
Change strategy

 
ASSESS REQUIREMENTS CHANGES
TECHNIQUES
Business case
Financial analysis
Document analysis
Business rules analysis
Estimation Interface analysis
Risk analysis and
management
Item tracking
Workshops and interviews Decision analysis


☑
ASSESS REQUIREMENTS CHANGES
STAKEHOLDERS
Business Analyst
Customer
Domain SME
End user
Regulator
Operational
support
Sponsor
Project
manager
T
ester
How are requirements approved?
 Overview
 Elements
 GD
uie
dfe
e
lc
in
te
p
s
ea
r n
U
d
ni
T
tools
 Techniques
 Stakeholders
Lesson 5: Requirements Life Cycle Management
Topic5.5:ApproveRequirements
APPROVE REQUIREMENTS
OVERVIEW
Purpose
 To obtain agreement on requirements and designs
 To obtain approval on requirements and designs
I
N
P
U
T
S
O
U
T
P
U
T
S
 Verified requirements
 Verified designs
 Approved requirements
 Approved designs
APPROVE REQUIREMENTS
ELEMENTS
Who need to be involved in decision making
Who are authorized for sign-off
Resolve conflicts, build consensus, make
stakeholders understand
Facilitate communication between
stakeholders, they may have different view
points and conflicting priority
☑Gain consensus
Obtain approvals from the stakeholder
Record the approval decisions
Maintain change log
☑
☑
☑ Track and
communicate approval
Conflict and issue
management
Understand
stakeholder roles
APPROVE REQUIREMENTS
GUIDELINES AND TOOLS
Change Strategy
Requirement
Management Tools /
Repository
Governance
approach
Legal or Regulatory
information


Solution scope

APPROVE REQUIREMENTS
TECHNIQUES
Acceptance and evaluation
criteria
Item tracking
Decision analysis
Reviews
Workshops
☑
APPROVE REQUIREMENTS
ACCEPTANCE AND EVALUATION CRITERIA - OVERVIEW
management
Backlog
Acceptance criteria –
• Used to define the requirements, outcome or conditions that must be met in order to consider
solution to be acceptable to the key stakeholders.
• Minimum set of requirements that must be met. It’s typically used when only one possible
solution is being evaluated.
Evaluation criteria –
• Used to assess a set of requirements in order to choose between multiple solutions.
• May be cost, performance, usability, performance etc.
APPROVE REQUIREMENTS
ACCEPTANCE AND EVALUATION CRITERIA - OVERVIEW
Strengths
• All requirements with testable acceptance criteria
• Agreement upon acceptance criteria required
• Acceptance criteria is necessary in case of contractual
obligations
• Evaluation criteria assist in the delivery of potential value
• Evaluation criteria helps in defining priorities
Limitations
• Acceptance criteria may express
contractual obligation
• Achieving agreement on evaluation
criteria for different needs can be
challenging
APPROVE REQUIREMENTS
ACCEPTANCE AND EVALUATION CRITERIA - ELEMENTS
Value attributes
Testability
Measures
Usability, security, reliability, scalability,
performance, availability of specific feature, ability
to perform specific operations etc.
User acceptance testing

☑

Continuous or
discrete scale
APPROVE REQUIREMENTS
STAKEHOLDERS
Business Analyst
Customer
Domain SME
End user
Regulator
Operational
support
Sponsor
Project
manager
T
ester
KEY TAKEAWAYS
5 Tasks in the Requirements Life Cycle
Management Knowledge Area.
Trace requirements and designs at different levels,
to ensure alignment to one another. Maintain
backward traceability and forward traceability.
Prioritize requirements and design in the order of
relative importance to maximize value delivery.
Maintain requirements for reusability. Retain
requirements accuracy and consistency throughout
the entire requirements life cycle.
Assess implications of proposed changes to
requirements and designs.
Obtain agreement on and approval of
requirements and designs for business analysis
work to continue solution construction.
Analysis, specification, modelling, verification,
validation, and communication of requirements are
performed in other knowledge areas.
 fect per Unit
Lesson 5: Requirements Life Cycle Management
CASESTUDYEXERCISE
CASE STUDY
PROBLEM STATEMENT
 .....
 ….
 ….
To deliver a solution for improving
‘customer connect’ that:
 Reduces the time between the customers
search for a policy and the field agent’s
response
 Ensures that the customer’s questions are
clarified as they are searching for the policy
actively (Hot Lead).
CASE STUDY
PROBLEM STATEMENT
Unabletoimplementone
oftheapproved
requirements.
Implementationhastaken
moretime.
Theoutofscope
requirementswere
allocatedtothesolution
component.
Therewereconflictsamongthe
stakeholdersontheprioritization
process.
Keystakeholderswerenotinvitedforthe
prioritizationmeeting.
CASE STUDY
ACTIVITIES
Thehighlevelscopeandreleaseplanareinplace.
Requirementsthatcanbeused
infutureprojectshavebeen
identified.
Thekeystakeholderhasdecidedtodesign,
develop, andimplementthesolution
incrementally.
Requirementsare
progressivelyelaborated.
CASE STUDY
EXERCISE
Questions Response
1 In the given case study, what is the approach
for business analysis?
Iterative Approach
Incremental Approach
Predictive
Adaptive
2 What should Paul do when he identifies a
couple of requirements, which can be used in
future projects?
Hold for the next projects
Label and store for reusability
Out of scope as it is general
requirement
None of the above
3 What can be the reason for the requirements
not in scope getting approved and allocated to
a solution component for implementation?
Change control process was not effectively
implemented
Impact Analysis was not performed
Missing traceability
Stakeholder urgency
CASE STUDY
EXERCISE
Questions Response
4 What can be the reason for missing to invite
key stakeholders for the prioritization
meeting, when they are required for
providing approvals?
Forgot to invite
Prioritization approach was not adequately
defined in the business analysis approach
Prioritization approach was not adequately
defined in the business analysis governance
approach
Prioritization approach was not adequately
defined in the business analysis information
management approach
5 When requirements are prioritized based on
only value, what flaw does the approach
have?
No impact
Must prioritize high value requirements
Missed considering relationship with other
requirements
None of the above
CASE STUDY
ANSWERS
Questions Answers
1 In the given case study, what is the approach for business
analysis?
Adaptive
2 What should Paul do when he identifies a couple of
requirements, which can be used in future projects?
Label and store for reusability
3 What can be the reason for the requirements not in scope
getting approved and allocated to a solution component for
implementation?
Missing traceability
4 What can be the reason for missing to invite a key
stakeholder, who is required for an approval into prioritization
meeting?
Prioritization approach was not
adequately defined in the
business analysis governance
approach.
5 When requirements are prioritized based on only value, what
flaw does the approach has?
Missed considering relationship
with other requirements.
Quiz
a.
b.
c.
d.
Which one of the following business analysis technique is not used when prioritizing
requirements?
QUIZ
1
Workshops
Brainstorming
Item Tracking
Decision Analysis
The correct answer is a, b and d.
Explanation: Decision analysis, item tracking and workshops are the techniques used to
prioritize requirements.
a.
b.
c.
d.
Which one of the following business analysis techniques is used when prioritizing
requirements?
QUIZ
1
Decision Analysis
Item Tracking
Brainstorming
Workshops
a.
b.
c.
d.
What is Double Triangular Distribution? If most likely is 9, best case is 5 and worst
case estimate is 11.
QUIZ
2
1
8.33
8.66
8
CBAP_eBook for business analyst for certification.pptx
CBAP_eBook for business analyst for certification.pptx
CBAP_eBook for business analyst for certification.pptx
CBAP_eBook for business analyst for certification.pptx
CBAP_eBook for business analyst for certification.pptx
CBAP_eBook for business analyst for certification.pptx
CBAP_eBook for business analyst for certification.pptx
CBAP_eBook for business analyst for certification.pptx
CBAP_eBook for business analyst for certification.pptx
CBAP_eBook for business analyst for certification.pptx
CBAP_eBook for business analyst for certification.pptx
CBAP_eBook for business analyst for certification.pptx
CBAP_eBook for business analyst for certification.pptx
CBAP_eBook for business analyst for certification.pptx
CBAP_eBook for business analyst for certification.pptx
CBAP_eBook for business analyst for certification.pptx
CBAP_eBook for business analyst for certification.pptx
CBAP_eBook for business analyst for certification.pptx
CBAP_eBook for business analyst for certification.pptx
CBAP_eBook for business analyst for certification.pptx
CBAP_eBook for business analyst for certification.pptx
CBAP_eBook for business analyst for certification.pptx
CBAP_eBook for business analyst for certification.pptx
CBAP_eBook for business analyst for certification.pptx
CBAP_eBook for business analyst for certification.pptx
CBAP_eBook for business analyst for certification.pptx
CBAP_eBook for business analyst for certification.pptx
CBAP_eBook for business analyst for certification.pptx
CBAP_eBook for business analyst for certification.pptx
CBAP_eBook for business analyst for certification.pptx
CBAP_eBook for business analyst for certification.pptx
CBAP_eBook for business analyst for certification.pptx
CBAP_eBook for business analyst for certification.pptx
CBAP_eBook for business analyst for certification.pptx
CBAP_eBook for business analyst for certification.pptx
CBAP_eBook for business analyst for certification.pptx
CBAP_eBook for business analyst for certification.pptx
CBAP_eBook for business analyst for certification.pptx
CBAP_eBook for business analyst for certification.pptx
CBAP_eBook for business analyst for certification.pptx
CBAP_eBook for business analyst for certification.pptx
CBAP_eBook for business analyst for certification.pptx
CBAP_eBook for business analyst for certification.pptx
CBAP_eBook for business analyst for certification.pptx
CBAP_eBook for business analyst for certification.pptx
CBAP_eBook for business analyst for certification.pptx
CBAP_eBook for business analyst for certification.pptx
CBAP_eBook for business analyst for certification.pptx
CBAP_eBook for business analyst for certification.pptx
CBAP_eBook for business analyst for certification.pptx
CBAP_eBook for business analyst for certification.pptx
CBAP_eBook for business analyst for certification.pptx
CBAP_eBook for business analyst for certification.pptx
CBAP_eBook for business analyst for certification.pptx
CBAP_eBook for business analyst for certification.pptx
CBAP_eBook for business analyst for certification.pptx
CBAP_eBook for business analyst for certification.pptx
CBAP_eBook for business analyst for certification.pptx
CBAP_eBook for business analyst for certification.pptx
CBAP_eBook for business analyst for certification.pptx
CBAP_eBook for business analyst for certification.pptx
CBAP_eBook for business analyst for certification.pptx
CBAP_eBook for business analyst for certification.pptx
CBAP_eBook for business analyst for certification.pptx
CBAP_eBook for business analyst for certification.pptx
CBAP_eBook for business analyst for certification.pptx
CBAP_eBook for business analyst for certification.pptx
CBAP_eBook for business analyst for certification.pptx
CBAP_eBook for business analyst for certification.pptx
CBAP_eBook for business analyst for certification.pptx
CBAP_eBook for business analyst for certification.pptx
CBAP_eBook for business analyst for certification.pptx
CBAP_eBook for business analyst for certification.pptx
CBAP_eBook for business analyst for certification.pptx
CBAP_eBook for business analyst for certification.pptx
CBAP_eBook for business analyst for certification.pptx
CBAP_eBook for business analyst for certification.pptx
CBAP_eBook for business analyst for certification.pptx
CBAP_eBook for business analyst for certification.pptx
CBAP_eBook for business analyst for certification.pptx
CBAP_eBook for business analyst for certification.pptx
CBAP_eBook for business analyst for certification.pptx
CBAP_eBook for business analyst for certification.pptx
CBAP_eBook for business analyst for certification.pptx
CBAP_eBook for business analyst for certification.pptx
CBAP_eBook for business analyst for certification.pptx
CBAP_eBook for business analyst for certification.pptx
CBAP_eBook for business analyst for certification.pptx
CBAP_eBook for business analyst for certification.pptx
CBAP_eBook for business analyst for certification.pptx
CBAP_eBook for business analyst for certification.pptx
CBAP_eBook for business analyst for certification.pptx
CBAP_eBook for business analyst for certification.pptx
CBAP_eBook for business analyst for certification.pptx
CBAP_eBook for business analyst for certification.pptx
CBAP_eBook for business analyst for certification.pptx
CBAP_eBook for business analyst for certification.pptx
CBAP_eBook for business analyst for certification.pptx
CBAP_eBook for business analyst for certification.pptx
CBAP_eBook for business analyst for certification.pptx
CBAP_eBook for business analyst for certification.pptx
CBAP_eBook for business analyst for certification.pptx
CBAP_eBook for business analyst for certification.pptx
CBAP_eBook for business analyst for certification.pptx
CBAP_eBook for business analyst for certification.pptx
CBAP_eBook for business analyst for certification.pptx
CBAP_eBook for business analyst for certification.pptx
CBAP_eBook for business analyst for certification.pptx
CBAP_eBook for business analyst for certification.pptx
CBAP_eBook for business analyst for certification.pptx
CBAP_eBook for business analyst for certification.pptx
CBAP_eBook for business analyst for certification.pptx
CBAP_eBook for business analyst for certification.pptx
CBAP_eBook for business analyst for certification.pptx
CBAP_eBook for business analyst for certification.pptx
CBAP_eBook for business analyst for certification.pptx
CBAP_eBook for business analyst for certification.pptx
CBAP_eBook for business analyst for certification.pptx
CBAP_eBook for business analyst for certification.pptx
CBAP_eBook for business analyst for certification.pptx
CBAP_eBook for business analyst for certification.pptx
CBAP_eBook for business analyst for certification.pptx
CBAP_eBook for business analyst for certification.pptx
CBAP_eBook for business analyst for certification.pptx
CBAP_eBook for business analyst for certification.pptx
CBAP_eBook for business analyst for certification.pptx
CBAP_eBook for business analyst for certification.pptx
CBAP_eBook for business analyst for certification.pptx
CBAP_eBook for business analyst for certification.pptx
CBAP_eBook for business analyst for certification.pptx
CBAP_eBook for business analyst for certification.pptx
CBAP_eBook for business analyst for certification.pptx
CBAP_eBook for business analyst for certification.pptx
CBAP_eBook for business analyst for certification.pptx
CBAP_eBook for business analyst for certification.pptx
CBAP_eBook for business analyst for certification.pptx
CBAP_eBook for business analyst for certification.pptx
CBAP_eBook for business analyst for certification.pptx
CBAP_eBook for business analyst for certification.pptx
CBAP_eBook for business analyst for certification.pptx
CBAP_eBook for business analyst for certification.pptx
CBAP_eBook for business analyst for certification.pptx
CBAP_eBook for business analyst for certification.pptx
CBAP_eBook for business analyst for certification.pptx
CBAP_eBook for business analyst for certification.pptx
CBAP_eBook for business analyst for certification.pptx
CBAP_eBook for business analyst for certification.pptx
CBAP_eBook for business analyst for certification.pptx
CBAP_eBook for business analyst for certification.pptx
CBAP_eBook for business analyst for certification.pptx
CBAP_eBook for business analyst for certification.pptx
CBAP_eBook for business analyst for certification.pptx
CBAP_eBook for business analyst for certification.pptx
CBAP_eBook for business analyst for certification.pptx
CBAP_eBook for business analyst for certification.pptx
CBAP_eBook for business analyst for certification.pptx
CBAP_eBook for business analyst for certification.pptx
CBAP_eBook for business analyst for certification.pptx
CBAP_eBook for business analyst for certification.pptx
CBAP_eBook for business analyst for certification.pptx
CBAP_eBook for business analyst for certification.pptx
CBAP_eBook for business analyst for certification.pptx
CBAP_eBook for business analyst for certification.pptx
CBAP_eBook for business analyst for certification.pptx
CBAP_eBook for business analyst for certification.pptx
CBAP_eBook for business analyst for certification.pptx
CBAP_eBook for business analyst for certification.pptx
CBAP_eBook for business analyst for certification.pptx
CBAP_eBook for business analyst for certification.pptx
CBAP_eBook for business analyst for certification.pptx
CBAP_eBook for business analyst for certification.pptx
CBAP_eBook for business analyst for certification.pptx
CBAP_eBook for business analyst for certification.pptx
CBAP_eBook for business analyst for certification.pptx
CBAP_eBook for business analyst for certification.pptx
CBAP_eBook for business analyst for certification.pptx
CBAP_eBook for business analyst for certification.pptx
CBAP_eBook for business analyst for certification.pptx
CBAP_eBook for business analyst for certification.pptx
CBAP_eBook for business analyst for certification.pptx
CBAP_eBook for business analyst for certification.pptx
CBAP_eBook for business analyst for certification.pptx
CBAP_eBook for business analyst for certification.pptx
CBAP_eBook for business analyst for certification.pptx
CBAP_eBook for business analyst for certification.pptx
CBAP_eBook for business analyst for certification.pptx
CBAP_eBook for business analyst for certification.pptx
CBAP_eBook for business analyst for certification.pptx
CBAP_eBook for business analyst for certification.pptx
CBAP_eBook for business analyst for certification.pptx
CBAP_eBook for business analyst for certification.pptx
CBAP_eBook for business analyst for certification.pptx
CBAP_eBook for business analyst for certification.pptx
CBAP_eBook for business analyst for certification.pptx
CBAP_eBook for business analyst for certification.pptx
CBAP_eBook for business analyst for certification.pptx
CBAP_eBook for business analyst for certification.pptx
CBAP_eBook for business analyst for certification.pptx
CBAP_eBook for business analyst for certification.pptx
CBAP_eBook for business analyst for certification.pptx
CBAP_eBook for business analyst for certification.pptx
CBAP_eBook for business analyst for certification.pptx
CBAP_eBook for business analyst for certification.pptx
CBAP_eBook for business analyst for certification.pptx
CBAP_eBook for business analyst for certification.pptx
CBAP_eBook for business analyst for certification.pptx
CBAP_eBook for business analyst for certification.pptx
CBAP_eBook for business analyst for certification.pptx
CBAP_eBook for business analyst for certification.pptx
CBAP_eBook for business analyst for certification.pptx
CBAP_eBook for business analyst for certification.pptx
CBAP_eBook for business analyst for certification.pptx
CBAP_eBook for business analyst for certification.pptx
CBAP_eBook for business analyst for certification.pptx
CBAP_eBook for business analyst for certification.pptx
CBAP_eBook for business analyst for certification.pptx
CBAP_eBook for business analyst for certification.pptx
CBAP_eBook for business analyst for certification.pptx
CBAP_eBook for business analyst for certification.pptx
CBAP_eBook for business analyst for certification.pptx
CBAP_eBook for business analyst for certification.pptx
CBAP_eBook for business analyst for certification.pptx
CBAP_eBook for business analyst for certification.pptx
CBAP_eBook for business analyst for certification.pptx
CBAP_eBook for business analyst for certification.pptx
CBAP_eBook for business analyst for certification.pptx
CBAP_eBook for business analyst for certification.pptx
CBAP_eBook for business analyst for certification.pptx
CBAP_eBook for business analyst for certification.pptx
CBAP_eBook for business analyst for certification.pptx
CBAP_eBook for business analyst for certification.pptx
CBAP_eBook for business analyst for certification.pptx
CBAP_eBook for business analyst for certification.pptx
CBAP_eBook for business analyst for certification.pptx
CBAP_eBook for business analyst for certification.pptx
CBAP_eBook for business analyst for certification.pptx
CBAP_eBook for business analyst for certification.pptx
CBAP_eBook for business analyst for certification.pptx
CBAP_eBook for business analyst for certification.pptx
CBAP_eBook for business analyst for certification.pptx
CBAP_eBook for business analyst for certification.pptx
CBAP_eBook for business analyst for certification.pptx
CBAP_eBook for business analyst for certification.pptx
CBAP_eBook for business analyst for certification.pptx
CBAP_eBook for business analyst for certification.pptx
CBAP_eBook for business analyst for certification.pptx
CBAP_eBook for business analyst for certification.pptx
CBAP_eBook for business analyst for certification.pptx
CBAP_eBook for business analyst for certification.pptx
CBAP_eBook for business analyst for certification.pptx
CBAP_eBook for business analyst for certification.pptx
CBAP_eBook for business analyst for certification.pptx
CBAP_eBook for business analyst for certification.pptx
CBAP_eBook for business analyst for certification.pptx
CBAP_eBook for business analyst for certification.pptx
CBAP_eBook for business analyst for certification.pptx
CBAP_eBook for business analyst for certification.pptx
CBAP_eBook for business analyst for certification.pptx
CBAP_eBook for business analyst for certification.pptx
CBAP_eBook for business analyst for certification.pptx
CBAP_eBook for business analyst for certification.pptx
CBAP_eBook for business analyst for certification.pptx
CBAP_eBook for business analyst for certification.pptx
CBAP_eBook for business analyst for certification.pptx
CBAP_eBook for business analyst for certification.pptx
CBAP_eBook for business analyst for certification.pptx
CBAP_eBook for business analyst for certification.pptx
CBAP_eBook for business analyst for certification.pptx
CBAP_eBook for business analyst for certification.pptx
CBAP_eBook for business analyst for certification.pptx
CBAP_eBook for business analyst for certification.pptx
CBAP_eBook for business analyst for certification.pptx
CBAP_eBook for business analyst for certification.pptx
CBAP_eBook for business analyst for certification.pptx
CBAP_eBook for business analyst for certification.pptx
CBAP_eBook for business analyst for certification.pptx
CBAP_eBook for business analyst for certification.pptx
CBAP_eBook for business analyst for certification.pptx
CBAP_eBook for business analyst for certification.pptx
CBAP_eBook for business analyst for certification.pptx
CBAP_eBook for business analyst for certification.pptx
CBAP_eBook for business analyst for certification.pptx
CBAP_eBook for business analyst for certification.pptx
CBAP_eBook for business analyst for certification.pptx
CBAP_eBook for business analyst for certification.pptx
CBAP_eBook for business analyst for certification.pptx
CBAP_eBook for business analyst for certification.pptx
CBAP_eBook for business analyst for certification.pptx
CBAP_eBook for business analyst for certification.pptx
CBAP_eBook for business analyst for certification.pptx
CBAP_eBook for business analyst for certification.pptx
CBAP_eBook for business analyst for certification.pptx
CBAP_eBook for business analyst for certification.pptx
CBAP_eBook for business analyst for certification.pptx
CBAP_eBook for business analyst for certification.pptx
CBAP_eBook for business analyst for certification.pptx
CBAP_eBook for business analyst for certification.pptx
CBAP_eBook for business analyst for certification.pptx
CBAP_eBook for business analyst for certification.pptx
CBAP_eBook for business analyst for certification.pptx
CBAP_eBook for business analyst for certification.pptx
CBAP_eBook for business analyst for certification.pptx
CBAP_eBook for business analyst for certification.pptx
CBAP_eBook for business analyst for certification.pptx
CBAP_eBook for business analyst for certification.pptx
CBAP_eBook for business analyst for certification.pptx
CBAP_eBook for business analyst for certification.pptx
CBAP_eBook for business analyst for certification.pptx
CBAP_eBook for business analyst for certification.pptx
CBAP_eBook for business analyst for certification.pptx
CBAP_eBook for business analyst for certification.pptx
CBAP_eBook for business analyst for certification.pptx
CBAP_eBook for business analyst for certification.pptx
CBAP_eBook for business analyst for certification.pptx
CBAP_eBook for business analyst for certification.pptx
CBAP_eBook for business analyst for certification.pptx
CBAP_eBook for business analyst for certification.pptx
CBAP_eBook for business analyst for certification.pptx
CBAP_eBook for business analyst for certification.pptx
CBAP_eBook for business analyst for certification.pptx
CBAP_eBook for business analyst for certification.pptx
CBAP_eBook for business analyst for certification.pptx
CBAP_eBook for business analyst for certification.pptx
CBAP_eBook for business analyst for certification.pptx
CBAP_eBook for business analyst for certification.pptx
CBAP_eBook for business analyst for certification.pptx
CBAP_eBook for business analyst for certification.pptx
CBAP_eBook for business analyst for certification.pptx
CBAP_eBook for business analyst for certification.pptx
CBAP_eBook for business analyst for certification.pptx
CBAP_eBook for business analyst for certification.pptx
CBAP_eBook for business analyst for certification.pptx
CBAP_eBook for business analyst for certification.pptx
CBAP_eBook for business analyst for certification.pptx
CBAP_eBook for business analyst for certification.pptx
CBAP_eBook for business analyst for certification.pptx
CBAP_eBook for business analyst for certification.pptx
CBAP_eBook for business analyst for certification.pptx
CBAP_eBook for business analyst for certification.pptx
CBAP_eBook for business analyst for certification.pptx
CBAP_eBook for business analyst for certification.pptx
CBAP_eBook for business analyst for certification.pptx
CBAP_eBook for business analyst for certification.pptx
CBAP_eBook for business analyst for certification.pptx
CBAP_eBook for business analyst for certification.pptx
CBAP_eBook for business analyst for certification.pptx
CBAP_eBook for business analyst for certification.pptx
CBAP_eBook for business analyst for certification.pptx
CBAP_eBook for business analyst for certification.pptx
CBAP_eBook for business analyst for certification.pptx
CBAP_eBook for business analyst for certification.pptx
CBAP_eBook for business analyst for certification.pptx
CBAP_eBook for business analyst for certification.pptx
CBAP_eBook for business analyst for certification.pptx
CBAP_eBook for business analyst for certification.pptx
CBAP_eBook for business analyst for certification.pptx
CBAP_eBook for business analyst for certification.pptx
CBAP_eBook for business analyst for certification.pptx
CBAP_eBook for business analyst for certification.pptx
CBAP_eBook for business analyst for certification.pptx
CBAP_eBook for business analyst for certification.pptx
CBAP_eBook for business analyst for certification.pptx
CBAP_eBook for business analyst for certification.pptx
CBAP_eBook for business analyst for certification.pptx
CBAP_eBook for business analyst for certification.pptx
CBAP_eBook for business analyst for certification.pptx
CBAP_eBook for business analyst for certification.pptx
CBAP_eBook for business analyst for certification.pptx
CBAP_eBook for business analyst for certification.pptx
CBAP_eBook for business analyst for certification.pptx
CBAP_eBook for business analyst for certification.pptx
CBAP_eBook for business analyst for certification.pptx
CBAP_eBook for business analyst for certification.pptx
CBAP_eBook for business analyst for certification.pptx
CBAP_eBook for business analyst for certification.pptx
CBAP_eBook for business analyst for certification.pptx
CBAP_eBook for business analyst for certification.pptx
CBAP_eBook for business analyst for certification.pptx
CBAP_eBook for business analyst for certification.pptx
CBAP_eBook for business analyst for certification.pptx
CBAP_eBook for business analyst for certification.pptx
CBAP_eBook for business analyst for certification.pptx
CBAP_eBook for business analyst for certification.pptx
CBAP_eBook for business analyst for certification.pptx
CBAP_eBook for business analyst for certification.pptx
CBAP_eBook for business analyst for certification.pptx
CBAP_eBook for business analyst for certification.pptx
CBAP_eBook for business analyst for certification.pptx
CBAP_eBook for business analyst for certification.pptx
CBAP_eBook for business analyst for certification.pptx
CBAP_eBook for business analyst for certification.pptx
CBAP_eBook for business analyst for certification.pptx
CBAP_eBook for business analyst for certification.pptx
CBAP_eBook for business analyst for certification.pptx
CBAP_eBook for business analyst for certification.pptx
CBAP_eBook for business analyst for certification.pptx
CBAP_eBook for business analyst for certification.pptx
CBAP_eBook for business analyst for certification.pptx
CBAP_eBook for business analyst for certification.pptx
CBAP_eBook for business analyst for certification.pptx
CBAP_eBook for business analyst for certification.pptx
CBAP_eBook for business analyst for certification.pptx
CBAP_eBook for business analyst for certification.pptx
CBAP_eBook for business analyst for certification.pptx
CBAP_eBook for business analyst for certification.pptx
CBAP_eBook for business analyst for certification.pptx
CBAP_eBook for business analyst for certification.pptx
CBAP_eBook for business analyst for certification.pptx
CBAP_eBook for business analyst for certification.pptx
CBAP_eBook for business analyst for certification.pptx
CBAP_eBook for business analyst for certification.pptx
CBAP_eBook for business analyst for certification.pptx
CBAP_eBook for business analyst for certification.pptx
CBAP_eBook for business analyst for certification.pptx
CBAP_eBook for business analyst for certification.pptx
CBAP_eBook for business analyst for certification.pptx
CBAP_eBook for business analyst for certification.pptx
CBAP_eBook for business analyst for certification.pptx
CBAP_eBook for business analyst for certification.pptx
CBAP_eBook for business analyst for certification.pptx
CBAP_eBook for business analyst for certification.pptx
CBAP_eBook for business analyst for certification.pptx
CBAP_eBook for business analyst for certification.pptx
CBAP_eBook for business analyst for certification.pptx
CBAP_eBook for business analyst for certification.pptx
CBAP_eBook for business analyst for certification.pptx
CBAP_eBook for business analyst for certification.pptx
CBAP_eBook for business analyst for certification.pptx
CBAP_eBook for business analyst for certification.pptx
CBAP_eBook for business analyst for certification.pptx
CBAP_eBook for business analyst for certification.pptx
CBAP_eBook for business analyst for certification.pptx
CBAP_eBook for business analyst for certification.pptx
CBAP_eBook for business analyst for certification.pptx
CBAP_eBook for business analyst for certification.pptx
CBAP_eBook for business analyst for certification.pptx
CBAP_eBook for business analyst for certification.pptx
CBAP_eBook for business analyst for certification.pptx
CBAP_eBook for business analyst for certification.pptx
CBAP_eBook for business analyst for certification.pptx
CBAP_eBook for business analyst for certification.pptx
CBAP_eBook for business analyst for certification.pptx
CBAP_eBook for business analyst for certification.pptx
CBAP_eBook for business analyst for certification.pptx
CBAP_eBook for business analyst for certification.pptx
CBAP_eBook for business analyst for certification.pptx
CBAP_eBook for business analyst for certification.pptx
CBAP_eBook for business analyst for certification.pptx
CBAP_eBook for business analyst for certification.pptx
CBAP_eBook for business analyst for certification.pptx
CBAP_eBook for business analyst for certification.pptx
CBAP_eBook for business analyst for certification.pptx
CBAP_eBook for business analyst for certification.pptx
CBAP_eBook for business analyst for certification.pptx
CBAP_eBook for business analyst for certification.pptx
CBAP_eBook for business analyst for certification.pptx
CBAP_eBook for business analyst for certification.pptx
CBAP_eBook for business analyst for certification.pptx
CBAP_eBook for business analyst for certification.pptx
CBAP_eBook for business analyst for certification.pptx
CBAP_eBook for business analyst for certification.pptx
CBAP_eBook for business analyst for certification.pptx
CBAP_eBook for business analyst for certification.pptx
CBAP_eBook for business analyst for certification.pptx
CBAP_eBook for business analyst for certification.pptx
CBAP_eBook for business analyst for certification.pptx
CBAP_eBook for business analyst for certification.pptx
CBAP_eBook for business analyst for certification.pptx
CBAP_eBook for business analyst for certification.pptx
CBAP_eBook for business analyst for certification.pptx
CBAP_eBook for business analyst for certification.pptx
CBAP_eBook for business analyst for certification.pptx
CBAP_eBook for business analyst for certification.pptx
CBAP_eBook for business analyst for certification.pptx
CBAP_eBook for business analyst for certification.pptx
CBAP_eBook for business analyst for certification.pptx
CBAP_eBook for business analyst for certification.pptx
CBAP_eBook for business analyst for certification.pptx
CBAP_eBook for business analyst for certification.pptx
CBAP_eBook for business analyst for certification.pptx
CBAP_eBook for business analyst for certification.pptx
CBAP_eBook for business analyst for certification.pptx
CBAP_eBook for business analyst for certification.pptx
CBAP_eBook for business analyst for certification.pptx
CBAP_eBook for business analyst for certification.pptx
CBAP_eBook for business analyst for certification.pptx
CBAP_eBook for business analyst for certification.pptx
CBAP_eBook for business analyst for certification.pptx
CBAP_eBook for business analyst for certification.pptx
CBAP_eBook for business analyst for certification.pptx
CBAP_eBook for business analyst for certification.pptx
CBAP_eBook for business analyst for certification.pptx
CBAP_eBook for business analyst for certification.pptx
CBAP_eBook for business analyst for certification.pptx
CBAP_eBook for business analyst for certification.pptx
CBAP_eBook for business analyst for certification.pptx
CBAP_eBook for business analyst for certification.pptx
CBAP_eBook for business analyst for certification.pptx
CBAP_eBook for business analyst for certification.pptx
CBAP_eBook for business analyst for certification.pptx
CBAP_eBook for business analyst for certification.pptx
CBAP_eBook for business analyst for certification.pptx
CBAP_eBook for business analyst for certification.pptx
CBAP_eBook for business analyst for certification.pptx
CBAP_eBook for business analyst for certification.pptx
CBAP_eBook for business analyst for certification.pptx
CBAP_eBook for business analyst for certification.pptx
CBAP_eBook for business analyst for certification.pptx
CBAP_eBook for business analyst for certification.pptx
CBAP_eBook for business analyst for certification.pptx
CBAP_eBook for business analyst for certification.pptx
CBAP_eBook for business analyst for certification.pptx
CBAP_eBook for business analyst for certification.pptx
CBAP_eBook for business analyst for certification.pptx
CBAP_eBook for business analyst for certification.pptx
CBAP_eBook for business analyst for certification.pptx
CBAP_eBook for business analyst for certification.pptx
CBAP_eBook for business analyst for certification.pptx
CBAP_eBook for business analyst for certification.pptx
CBAP_eBook for business analyst for certification.pptx
CBAP_eBook for business analyst for certification.pptx
CBAP_eBook for business analyst for certification.pptx
CBAP_eBook for business analyst for certification.pptx
CBAP_eBook for business analyst for certification.pptx
CBAP_eBook for business analyst for certification.pptx
CBAP_eBook for business analyst for certification.pptx
CBAP_eBook for business analyst for certification.pptx
CBAP_eBook for business analyst for certification.pptx
CBAP_eBook for business analyst for certification.pptx
CBAP_eBook for business analyst for certification.pptx
CBAP_eBook for business analyst for certification.pptx
CBAP_eBook for business analyst for certification.pptx
CBAP_eBook for business analyst for certification.pptx
CBAP_eBook for business analyst for certification.pptx
CBAP_eBook for business analyst for certification.pptx
CBAP_eBook for business analyst for certification.pptx
CBAP_eBook for business analyst for certification.pptx
CBAP_eBook for business analyst for certification.pptx
CBAP_eBook for business analyst for certification.pptx
CBAP_eBook for business analyst for certification.pptx
CBAP_eBook for business analyst for certification.pptx
CBAP_eBook for business analyst for certification.pptx
CBAP_eBook for business analyst for certification.pptx
CBAP_eBook for business analyst for certification.pptx
CBAP_eBook for business analyst for certification.pptx
CBAP_eBook for business analyst for certification.pptx
CBAP_eBook for business analyst for certification.pptx
CBAP_eBook for business analyst for certification.pptx
CBAP_eBook for business analyst for certification.pptx
CBAP_eBook for business analyst for certification.pptx
CBAP_eBook for business analyst for certification.pptx
CBAP_eBook for business analyst for certification.pptx
CBAP_eBook for business analyst for certification.pptx
CBAP_eBook for business analyst for certification.pptx
CBAP_eBook for business analyst for certification.pptx
CBAP_eBook for business analyst for certification.pptx
CBAP_eBook for business analyst for certification.pptx
CBAP_eBook for business analyst for certification.pptx
CBAP_eBook for business analyst for certification.pptx
CBAP_eBook for business analyst for certification.pptx
CBAP_eBook for business analyst for certification.pptx
CBAP_eBook for business analyst for certification.pptx
CBAP_eBook for business analyst for certification.pptx
CBAP_eBook for business analyst for certification.pptx
CBAP_eBook for business analyst for certification.pptx
CBAP_eBook for business analyst for certification.pptx
CBAP_eBook for business analyst for certification.pptx
CBAP_eBook for business analyst for certification.pptx
CBAP_eBook for business analyst for certification.pptx
CBAP_eBook for business analyst for certification.pptx
CBAP_eBook for business analyst for certification.pptx
CBAP_eBook for business analyst for certification.pptx
CBAP_eBook for business analyst for certification.pptx
CBAP_eBook for business analyst for certification.pptx
CBAP_eBook for business analyst for certification.pptx
CBAP_eBook for business analyst for certification.pptx
CBAP_eBook for business analyst for certification.pptx
CBAP_eBook for business analyst for certification.pptx
CBAP_eBook for business analyst for certification.pptx
CBAP_eBook for business analyst for certification.pptx
CBAP_eBook for business analyst for certification.pptx
CBAP_eBook for business analyst for certification.pptx
CBAP_eBook for business analyst for certification.pptx
CBAP_eBook for business analyst for certification.pptx
CBAP_eBook for business analyst for certification.pptx
CBAP_eBook for business analyst for certification.pptx
CBAP_eBook for business analyst for certification.pptx
CBAP_eBook for business analyst for certification.pptx
CBAP_eBook for business analyst for certification.pptx
CBAP_eBook for business analyst for certification.pptx
CBAP_eBook for business analyst for certification.pptx
CBAP_eBook for business analyst for certification.pptx
CBAP_eBook for business analyst for certification.pptx
CBAP_eBook for business analyst for certification.pptx
CBAP_eBook for business analyst for certification.pptx
CBAP_eBook for business analyst for certification.pptx
CBAP_eBook for business analyst for certification.pptx
CBAP_eBook for business analyst for certification.pptx
CBAP_eBook for business analyst for certification.pptx
CBAP_eBook for business analyst for certification.pptx
CBAP_eBook for business analyst for certification.pptx
CBAP_eBook for business analyst for certification.pptx
CBAP_eBook for business analyst for certification.pptx
CBAP_eBook for business analyst for certification.pptx
CBAP_eBook for business analyst for certification.pptx
CBAP_eBook for business analyst for certification.pptx
CBAP_eBook for business analyst for certification.pptx
CBAP_eBook for business analyst for certification.pptx
CBAP_eBook for business analyst for certification.pptx
CBAP_eBook for business analyst for certification.pptx
CBAP_eBook for business analyst for certification.pptx
CBAP_eBook for business analyst for certification.pptx
CBAP_eBook for business analyst for certification.pptx
CBAP_eBook for business analyst for certification.pptx
CBAP_eBook for business analyst for certification.pptx
CBAP_eBook for business analyst for certification.pptx
CBAP_eBook for business analyst for certification.pptx
CBAP_eBook for business analyst for certification.pptx
CBAP_eBook for business analyst for certification.pptx
CBAP_eBook for business analyst for certification.pptx
CBAP_eBook for business analyst for certification.pptx
CBAP_eBook for business analyst for certification.pptx
CBAP_eBook for business analyst for certification.pptx
CBAP_eBook for business analyst for certification.pptx
CBAP_eBook for business analyst for certification.pptx
CBAP_eBook for business analyst for certification.pptx
CBAP_eBook for business analyst for certification.pptx
CBAP_eBook for business analyst for certification.pptx
CBAP_eBook for business analyst for certification.pptx
CBAP_eBook for business analyst for certification.pptx
CBAP_eBook for business analyst for certification.pptx
CBAP_eBook for business analyst for certification.pptx
CBAP_eBook for business analyst for certification.pptx
CBAP_eBook for business analyst for certification.pptx
CBAP_eBook for business analyst for certification.pptx
CBAP_eBook for business analyst for certification.pptx
CBAP_eBook for business analyst for certification.pptx
CBAP_eBook for business analyst for certification.pptx
CBAP_eBook for business analyst for certification.pptx
CBAP_eBook for business analyst for certification.pptx
CBAP_eBook for business analyst for certification.pptx
CBAP_eBook for business analyst for certification.pptx
CBAP_eBook for business analyst for certification.pptx
CBAP_eBook for business analyst for certification.pptx
CBAP_eBook for business analyst for certification.pptx
CBAP_eBook for business analyst for certification.pptx
CBAP_eBook for business analyst for certification.pptx
CBAP_eBook for business analyst for certification.pptx
CBAP_eBook for business analyst for certification.pptx
CBAP_eBook for business analyst for certification.pptx
CBAP_eBook for business analyst for certification.pptx
CBAP_eBook for business analyst for certification.pptx
CBAP_eBook for business analyst for certification.pptx
CBAP_eBook for business analyst for certification.pptx
CBAP_eBook for business analyst for certification.pptx
CBAP_eBook for business analyst for certification.pptx
CBAP_eBook for business analyst for certification.pptx
CBAP_eBook for business analyst for certification.pptx
CBAP_eBook for business analyst for certification.pptx
CBAP_eBook for business analyst for certification.pptx
CBAP_eBook for business analyst for certification.pptx
CBAP_eBook for business analyst for certification.pptx
CBAP_eBook for business analyst for certification.pptx
CBAP_eBook for business analyst for certification.pptx
CBAP_eBook for business analyst for certification.pptx
CBAP_eBook for business analyst for certification.pptx
CBAP_eBook for business analyst for certification.pptx
CBAP_eBook for business analyst for certification.pptx
CBAP_eBook for business analyst for certification.pptx
CBAP_eBook for business analyst for certification.pptx
CBAP_eBook for business analyst for certification.pptx
CBAP_eBook for business analyst for certification.pptx
CBAP_eBook for business analyst for certification.pptx
CBAP_eBook for business analyst for certification.pptx
CBAP_eBook for business analyst for certification.pptx
CBAP_eBook for business analyst for certification.pptx
CBAP_eBook for business analyst for certification.pptx
CBAP_eBook for business analyst for certification.pptx
CBAP_eBook for business analyst for certification.pptx
CBAP_eBook for business analyst for certification.pptx
CBAP_eBook for business analyst for certification.pptx
CBAP_eBook for business analyst for certification.pptx
CBAP_eBook for business analyst for certification.pptx
CBAP_eBook for business analyst for certification.pptx
CBAP_eBook for business analyst for certification.pptx
CBAP_eBook for business analyst for certification.pptx
CBAP_eBook for business analyst for certification.pptx
CBAP_eBook for business analyst for certification.pptx
CBAP_eBook for business analyst for certification.pptx
CBAP_eBook for business analyst for certification.pptx
CBAP_eBook for business analyst for certification.pptx
CBAP_eBook for business analyst for certification.pptx
CBAP_eBook for business analyst for certification.pptx
CBAP_eBook for business analyst for certification.pptx
CBAP_eBook for business analyst for certification.pptx
CBAP_eBook for business analyst for certification.pptx
CBAP_eBook for business analyst for certification.pptx
CBAP_eBook for business analyst for certification.pptx
CBAP_eBook for business analyst for certification.pptx
CBAP_eBook for business analyst for certification.pptx
CBAP_eBook for business analyst for certification.pptx
CBAP_eBook for business analyst for certification.pptx
CBAP_eBook for business analyst for certification.pptx
CBAP_eBook for business analyst for certification.pptx
CBAP_eBook for business analyst for certification.pptx
CBAP_eBook for business analyst for certification.pptx
CBAP_eBook for business analyst for certification.pptx
CBAP_eBook for business analyst for certification.pptx
CBAP_eBook for business analyst for certification.pptx
CBAP_eBook for business analyst for certification.pptx
CBAP_eBook for business analyst for certification.pptx
CBAP_eBook for business analyst for certification.pptx
CBAP_eBook for business analyst for certification.pptx
CBAP_eBook for business analyst for certification.pptx
CBAP_eBook for business analyst for certification.pptx
CBAP_eBook for business analyst for certification.pptx
CBAP_eBook for business analyst for certification.pptx
CBAP_eBook for business analyst for certification.pptx
CBAP_eBook for business analyst for certification.pptx
CBAP_eBook for business analyst for certification.pptx
CBAP_eBook for business analyst for certification.pptx
CBAP_eBook for business analyst for certification.pptx
CBAP_eBook for business analyst for certification.pptx
CBAP_eBook for business analyst for certification.pptx
CBAP_eBook for business analyst for certification.pptx
CBAP_eBook for business analyst for certification.pptx
CBAP_eBook for business analyst for certification.pptx
CBAP_eBook for business analyst for certification.pptx
CBAP_eBook for business analyst for certification.pptx
CBAP_eBook for business analyst for certification.pptx
CBAP_eBook for business analyst for certification.pptx
CBAP_eBook for business analyst for certification.pptx
CBAP_eBook for business analyst for certification.pptx
CBAP_eBook for business analyst for certification.pptx
CBAP_eBook for business analyst for certification.pptx
CBAP_eBook for business analyst for certification.pptx
CBAP_eBook for business analyst for certification.pptx
CBAP_eBook for business analyst for certification.pptx
CBAP_eBook for business analyst for certification.pptx
CBAP_eBook for business analyst for certification.pptx
CBAP_eBook for business analyst for certification.pptx
CBAP_eBook for business analyst for certification.pptx
CBAP_eBook for business analyst for certification.pptx
CBAP_eBook for business analyst for certification.pptx
CBAP_eBook for business analyst for certification.pptx
CBAP_eBook for business analyst for certification.pptx
CBAP_eBook for business analyst for certification.pptx
CBAP_eBook for business analyst for certification.pptx
CBAP_eBook for business analyst for certification.pptx
CBAP_eBook for business analyst for certification.pptx
CBAP_eBook for business analyst for certification.pptx
CBAP_eBook for business analyst for certification.pptx
CBAP_eBook for business analyst for certification.pptx
CBAP_eBook for business analyst for certification.pptx
CBAP_eBook for business analyst for certification.pptx
CBAP_eBook for business analyst for certification.pptx
CBAP_eBook for business analyst for certification.pptx
CBAP_eBook for business analyst for certification.pptx
CBAP_eBook for business analyst for certification.pptx
CBAP_eBook for business analyst for certification.pptx
CBAP_eBook for business analyst for certification.pptx
CBAP_eBook for business analyst for certification.pptx
CBAP_eBook for business analyst for certification.pptx
CBAP_eBook for business analyst for certification.pptx
CBAP_eBook for business analyst for certification.pptx
CBAP_eBook for business analyst for certification.pptx

More Related Content

Similar to CBAP_eBook for business analyst for certification.pptx

CBAP_and_BABOK.ppt
CBAP_and_BABOK.pptCBAP_and_BABOK.ppt
CBAP_and_BABOK.pptAnilKumarARS
 
The IIBA Value Proposition and Our Vision for the Future
The IIBA Value Proposition and Our Vision for the FutureThe IIBA Value Proposition and Our Vision for the Future
The IIBA Value Proposition and Our Vision for the FutureTracy Cook
 
ABPMP International Overview
ABPMP International OverviewABPMP International Overview
ABPMP International Overviewtjbenedi
 
ABPMP International Overview
ABPMP International OverviewABPMP International Overview
ABPMP International OverviewTony Benedict
 
Adaptive integrated Business Analysis (CBAP + CPRE) workshop v8.0
Adaptive integrated Business Analysis (CBAP + CPRE)  workshop v8.0Adaptive integrated Business Analysis (CBAP + CPRE)  workshop v8.0
Adaptive integrated Business Analysis (CBAP + CPRE) workshop v8.0LN Mishra CBAP
 
Understanding The CBAP® Designation - Panel Discussion
Understanding The CBAP® Designation - Panel DiscussionUnderstanding The CBAP® Designation - Panel Discussion
Understanding The CBAP® Designation - Panel DiscussionTracy Cook
 
Understanding IIBA Certification Program - November 2011 (full deck)
Understanding IIBA Certification Program - November 2011 (full deck)Understanding IIBA Certification Program - November 2011 (full deck)
Understanding IIBA Certification Program - November 2011 (full deck)Marie Halsey
 
CBAP Certification Training
CBAP Certification TrainingCBAP Certification Training
CBAP Certification TrainingMultisoftsystems
 
CBAP Certification Training
CBAP Certification TrainingCBAP Certification Training
CBAP Certification TrainingAlisha Tope
 
How Best to Prepare for ECBA
How Best to Prepare for ECBAHow Best to Prepare for ECBA
How Best to Prepare for ECBALN Mishra CBAP
 
BABOK® Release Party Webinar
BABOK® Release Party WebinarBABOK® Release Party Webinar
BABOK® Release Party WebinarTracy Cook
 
CBAP- detailed brochure
CBAP- detailed brochure  CBAP- detailed brochure
CBAP- detailed brochure Naz Ish
 
Free ECBA study guide - Mastering ecba
Free ECBA study guide - Mastering ecbaFree ECBA study guide - Mastering ecba
Free ECBA study guide - Mastering ecbaLN Mishra CBAP
 
Certified Business Analysis Professional(CBAP)
Certified Business Analysis Professional(CBAP)Certified Business Analysis Professional(CBAP)
Certified Business Analysis Professional(CBAP)Multisoft Virtual Academy
 
Free ECBA V3 study guide
Free ECBA V3 study guideFree ECBA V3 study guide
Free ECBA V3 study guideLN Mishra CBAP
 

Similar to CBAP_eBook for business analyst for certification.pptx (20)

ECBA training
ECBA trainingECBA training
ECBA training
 
Understanding the CBAP Designation
Understanding the CBAP DesignationUnderstanding the CBAP Designation
Understanding the CBAP Designation
 
Business Analyst Training FAQs
Business Analyst Training FAQs Business Analyst Training FAQs
Business Analyst Training FAQs
 
masVenta Business Analysis Multilevel Certificates 2016 What's new?
masVenta Business Analysis Multilevel Certificates 2016 What's new?masVenta Business Analysis Multilevel Certificates 2016 What's new?
masVenta Business Analysis Multilevel Certificates 2016 What's new?
 
CBAP_and_BABOK.ppt
CBAP_and_BABOK.pptCBAP_and_BABOK.ppt
CBAP_and_BABOK.ppt
 
The IIBA Value Proposition and Our Vision for the Future
The IIBA Value Proposition and Our Vision for the FutureThe IIBA Value Proposition and Our Vision for the Future
The IIBA Value Proposition and Our Vision for the Future
 
ABPMP International Overview
ABPMP International OverviewABPMP International Overview
ABPMP International Overview
 
ABPMP International Overview
ABPMP International OverviewABPMP International Overview
ABPMP International Overview
 
Adaptive integrated Business Analysis (CBAP + CPRE) workshop v8.0
Adaptive integrated Business Analysis (CBAP + CPRE)  workshop v8.0Adaptive integrated Business Analysis (CBAP + CPRE)  workshop v8.0
Adaptive integrated Business Analysis (CBAP + CPRE) workshop v8.0
 
Understanding The CBAP® Designation - Panel Discussion
Understanding The CBAP® Designation - Panel DiscussionUnderstanding The CBAP® Designation - Panel Discussion
Understanding The CBAP® Designation - Panel Discussion
 
Understanding IIBA Certification Program - November 2011 (full deck)
Understanding IIBA Certification Program - November 2011 (full deck)Understanding IIBA Certification Program - November 2011 (full deck)
Understanding IIBA Certification Program - November 2011 (full deck)
 
CBAP Certification Training
CBAP Certification TrainingCBAP Certification Training
CBAP Certification Training
 
CBAP Certification Training
CBAP Certification TrainingCBAP Certification Training
CBAP Certification Training
 
How Best to Prepare for ECBA
How Best to Prepare for ECBAHow Best to Prepare for ECBA
How Best to Prepare for ECBA
 
BABOK® Release Party Webinar
BABOK® Release Party WebinarBABOK® Release Party Webinar
BABOK® Release Party Webinar
 
CBAP- detailed brochure
CBAP- detailed brochure  CBAP- detailed brochure
CBAP- detailed brochure
 
Cbap2 may2016 kuwait
Cbap2 may2016 kuwaitCbap2 may2016 kuwait
Cbap2 may2016 kuwait
 
Free ECBA study guide - Mastering ecba
Free ECBA study guide - Mastering ecbaFree ECBA study guide - Mastering ecba
Free ECBA study guide - Mastering ecba
 
Certified Business Analysis Professional(CBAP)
Certified Business Analysis Professional(CBAP)Certified Business Analysis Professional(CBAP)
Certified Business Analysis Professional(CBAP)
 
Free ECBA V3 study guide
Free ECBA V3 study guideFree ECBA V3 study guide
Free ECBA V3 study guide
 

Recently uploaded

Regression analysis: Simple Linear Regression Multiple Linear Regression
Regression analysis:  Simple Linear Regression Multiple Linear RegressionRegression analysis:  Simple Linear Regression Multiple Linear Regression
Regression analysis: Simple Linear Regression Multiple Linear RegressionRavindra Nath Shukla
 
VIP Call Girl Jamshedpur Aashi 8250192130 Independent Escort Service Jamshedpur
VIP Call Girl Jamshedpur Aashi 8250192130 Independent Escort Service JamshedpurVIP Call Girl Jamshedpur Aashi 8250192130 Independent Escort Service Jamshedpur
VIP Call Girl Jamshedpur Aashi 8250192130 Independent Escort Service JamshedpurSuhani Kapoor
 
RE Capital's Visionary Leadership under Newman Leech
RE Capital's Visionary Leadership under Newman LeechRE Capital's Visionary Leadership under Newman Leech
RE Capital's Visionary Leadership under Newman LeechNewman George Leech
 
Call Girls In Connaught Place Delhi ❤️88604**77959_Russian 100% Genuine Escor...
Call Girls In Connaught Place Delhi ❤️88604**77959_Russian 100% Genuine Escor...Call Girls In Connaught Place Delhi ❤️88604**77959_Russian 100% Genuine Escor...
Call Girls In Connaught Place Delhi ❤️88604**77959_Russian 100% Genuine Escor...lizamodels9
 
Progress Report - Oracle Database Analyst Summit
Progress  Report - Oracle Database Analyst SummitProgress  Report - Oracle Database Analyst Summit
Progress Report - Oracle Database Analyst SummitHolger Mueller
 
BEST Call Girls In Old Faridabad ✨ 9773824855 ✨ Escorts Service In Delhi Ncr,
BEST Call Girls In Old Faridabad ✨ 9773824855 ✨ Escorts Service In Delhi Ncr,BEST Call Girls In Old Faridabad ✨ 9773824855 ✨ Escorts Service In Delhi Ncr,
BEST Call Girls In Old Faridabad ✨ 9773824855 ✨ Escorts Service In Delhi Ncr,noida100girls
 
2024 Numerator Consumer Study of Cannabis Usage
2024 Numerator Consumer Study of Cannabis Usage2024 Numerator Consumer Study of Cannabis Usage
2024 Numerator Consumer Study of Cannabis UsageNeil Kimberley
 
Sales & Marketing Alignment: How to Synergize for Success
Sales & Marketing Alignment: How to Synergize for SuccessSales & Marketing Alignment: How to Synergize for Success
Sales & Marketing Alignment: How to Synergize for SuccessAggregage
 
Call Girls In Sikandarpur Gurgaon ❤️8860477959_Russian 100% Genuine Escorts I...
Call Girls In Sikandarpur Gurgaon ❤️8860477959_Russian 100% Genuine Escorts I...Call Girls In Sikandarpur Gurgaon ❤️8860477959_Russian 100% Genuine Escorts I...
Call Girls In Sikandarpur Gurgaon ❤️8860477959_Russian 100% Genuine Escorts I...lizamodels9
 
0183760ssssssssssssssssssssssssssss00101011 (27).pdf
0183760ssssssssssssssssssssssssssss00101011 (27).pdf0183760ssssssssssssssssssssssssssss00101011 (27).pdf
0183760ssssssssssssssssssssssssssss00101011 (27).pdfRenandantas16
 
Vip Dewas Call Girls #9907093804 Contact Number Escorts Service Dewas
Vip Dewas Call Girls #9907093804 Contact Number Escorts Service DewasVip Dewas Call Girls #9907093804 Contact Number Escorts Service Dewas
Vip Dewas Call Girls #9907093804 Contact Number Escorts Service Dewasmakika9823
 
Catalogue ONG NUOC PPR DE NHAT .pdf
Catalogue ONG NUOC PPR DE NHAT      .pdfCatalogue ONG NUOC PPR DE NHAT      .pdf
Catalogue ONG NUOC PPR DE NHAT .pdfOrient Homes
 
Call Girls In Radisson Blu Hotel New Delhi Paschim Vihar ❤️8860477959 Escorts...
Call Girls In Radisson Blu Hotel New Delhi Paschim Vihar ❤️8860477959 Escorts...Call Girls In Radisson Blu Hotel New Delhi Paschim Vihar ❤️8860477959 Escorts...
Call Girls In Radisson Blu Hotel New Delhi Paschim Vihar ❤️8860477959 Escorts...lizamodels9
 
FULL ENJOY - 9953040155 Call Girls in Chhatarpur | Delhi
FULL ENJOY - 9953040155 Call Girls in Chhatarpur | DelhiFULL ENJOY - 9953040155 Call Girls in Chhatarpur | Delhi
FULL ENJOY - 9953040155 Call Girls in Chhatarpur | DelhiMalviyaNagarCallGirl
 
Tech Startup Growth Hacking 101 - Basics on Growth Marketing
Tech Startup Growth Hacking 101  - Basics on Growth MarketingTech Startup Growth Hacking 101  - Basics on Growth Marketing
Tech Startup Growth Hacking 101 - Basics on Growth MarketingShawn Pang
 
rishikeshgirls.in- Rishikesh call girl.pdf
rishikeshgirls.in- Rishikesh call girl.pdfrishikeshgirls.in- Rishikesh call girl.pdf
rishikeshgirls.in- Rishikesh call girl.pdfmuskan1121w
 
Monte Carlo simulation : Simulation using MCSM
Monte Carlo simulation : Simulation using MCSMMonte Carlo simulation : Simulation using MCSM
Monte Carlo simulation : Simulation using MCSMRavindra Nath Shukla
 
(8264348440) 🔝 Call Girls In Keshav Puram 🔝 Delhi NCR
(8264348440) 🔝 Call Girls In Keshav Puram 🔝 Delhi NCR(8264348440) 🔝 Call Girls In Keshav Puram 🔝 Delhi NCR
(8264348440) 🔝 Call Girls In Keshav Puram 🔝 Delhi NCRsoniya singh
 
Call Girls Pune Just Call 9907093804 Top Class Call Girl Service Available
Call Girls Pune Just Call 9907093804 Top Class Call Girl Service AvailableCall Girls Pune Just Call 9907093804 Top Class Call Girl Service Available
Call Girls Pune Just Call 9907093804 Top Class Call Girl Service AvailableDipal Arora
 
Lowrate Call Girls In Sector 18 Noida ❤️8860477959 Escorts 100% Genuine Servi...
Lowrate Call Girls In Sector 18 Noida ❤️8860477959 Escorts 100% Genuine Servi...Lowrate Call Girls In Sector 18 Noida ❤️8860477959 Escorts 100% Genuine Servi...
Lowrate Call Girls In Sector 18 Noida ❤️8860477959 Escorts 100% Genuine Servi...lizamodels9
 

Recently uploaded (20)

Regression analysis: Simple Linear Regression Multiple Linear Regression
Regression analysis:  Simple Linear Regression Multiple Linear RegressionRegression analysis:  Simple Linear Regression Multiple Linear Regression
Regression analysis: Simple Linear Regression Multiple Linear Regression
 
VIP Call Girl Jamshedpur Aashi 8250192130 Independent Escort Service Jamshedpur
VIP Call Girl Jamshedpur Aashi 8250192130 Independent Escort Service JamshedpurVIP Call Girl Jamshedpur Aashi 8250192130 Independent Escort Service Jamshedpur
VIP Call Girl Jamshedpur Aashi 8250192130 Independent Escort Service Jamshedpur
 
RE Capital's Visionary Leadership under Newman Leech
RE Capital's Visionary Leadership under Newman LeechRE Capital's Visionary Leadership under Newman Leech
RE Capital's Visionary Leadership under Newman Leech
 
Call Girls In Connaught Place Delhi ❤️88604**77959_Russian 100% Genuine Escor...
Call Girls In Connaught Place Delhi ❤️88604**77959_Russian 100% Genuine Escor...Call Girls In Connaught Place Delhi ❤️88604**77959_Russian 100% Genuine Escor...
Call Girls In Connaught Place Delhi ❤️88604**77959_Russian 100% Genuine Escor...
 
Progress Report - Oracle Database Analyst Summit
Progress  Report - Oracle Database Analyst SummitProgress  Report - Oracle Database Analyst Summit
Progress Report - Oracle Database Analyst Summit
 
BEST Call Girls In Old Faridabad ✨ 9773824855 ✨ Escorts Service In Delhi Ncr,
BEST Call Girls In Old Faridabad ✨ 9773824855 ✨ Escorts Service In Delhi Ncr,BEST Call Girls In Old Faridabad ✨ 9773824855 ✨ Escorts Service In Delhi Ncr,
BEST Call Girls In Old Faridabad ✨ 9773824855 ✨ Escorts Service In Delhi Ncr,
 
2024 Numerator Consumer Study of Cannabis Usage
2024 Numerator Consumer Study of Cannabis Usage2024 Numerator Consumer Study of Cannabis Usage
2024 Numerator Consumer Study of Cannabis Usage
 
Sales & Marketing Alignment: How to Synergize for Success
Sales & Marketing Alignment: How to Synergize for SuccessSales & Marketing Alignment: How to Synergize for Success
Sales & Marketing Alignment: How to Synergize for Success
 
Call Girls In Sikandarpur Gurgaon ❤️8860477959_Russian 100% Genuine Escorts I...
Call Girls In Sikandarpur Gurgaon ❤️8860477959_Russian 100% Genuine Escorts I...Call Girls In Sikandarpur Gurgaon ❤️8860477959_Russian 100% Genuine Escorts I...
Call Girls In Sikandarpur Gurgaon ❤️8860477959_Russian 100% Genuine Escorts I...
 
0183760ssssssssssssssssssssssssssss00101011 (27).pdf
0183760ssssssssssssssssssssssssssss00101011 (27).pdf0183760ssssssssssssssssssssssssssss00101011 (27).pdf
0183760ssssssssssssssssssssssssssss00101011 (27).pdf
 
Vip Dewas Call Girls #9907093804 Contact Number Escorts Service Dewas
Vip Dewas Call Girls #9907093804 Contact Number Escorts Service DewasVip Dewas Call Girls #9907093804 Contact Number Escorts Service Dewas
Vip Dewas Call Girls #9907093804 Contact Number Escorts Service Dewas
 
Catalogue ONG NUOC PPR DE NHAT .pdf
Catalogue ONG NUOC PPR DE NHAT      .pdfCatalogue ONG NUOC PPR DE NHAT      .pdf
Catalogue ONG NUOC PPR DE NHAT .pdf
 
Call Girls In Radisson Blu Hotel New Delhi Paschim Vihar ❤️8860477959 Escorts...
Call Girls In Radisson Blu Hotel New Delhi Paschim Vihar ❤️8860477959 Escorts...Call Girls In Radisson Blu Hotel New Delhi Paschim Vihar ❤️8860477959 Escorts...
Call Girls In Radisson Blu Hotel New Delhi Paschim Vihar ❤️8860477959 Escorts...
 
FULL ENJOY - 9953040155 Call Girls in Chhatarpur | Delhi
FULL ENJOY - 9953040155 Call Girls in Chhatarpur | DelhiFULL ENJOY - 9953040155 Call Girls in Chhatarpur | Delhi
FULL ENJOY - 9953040155 Call Girls in Chhatarpur | Delhi
 
Tech Startup Growth Hacking 101 - Basics on Growth Marketing
Tech Startup Growth Hacking 101  - Basics on Growth MarketingTech Startup Growth Hacking 101  - Basics on Growth Marketing
Tech Startup Growth Hacking 101 - Basics on Growth Marketing
 
rishikeshgirls.in- Rishikesh call girl.pdf
rishikeshgirls.in- Rishikesh call girl.pdfrishikeshgirls.in- Rishikesh call girl.pdf
rishikeshgirls.in- Rishikesh call girl.pdf
 
Monte Carlo simulation : Simulation using MCSM
Monte Carlo simulation : Simulation using MCSMMonte Carlo simulation : Simulation using MCSM
Monte Carlo simulation : Simulation using MCSM
 
(8264348440) 🔝 Call Girls In Keshav Puram 🔝 Delhi NCR
(8264348440) 🔝 Call Girls In Keshav Puram 🔝 Delhi NCR(8264348440) 🔝 Call Girls In Keshav Puram 🔝 Delhi NCR
(8264348440) 🔝 Call Girls In Keshav Puram 🔝 Delhi NCR
 
Call Girls Pune Just Call 9907093804 Top Class Call Girl Service Available
Call Girls Pune Just Call 9907093804 Top Class Call Girl Service AvailableCall Girls Pune Just Call 9907093804 Top Class Call Girl Service Available
Call Girls Pune Just Call 9907093804 Top Class Call Girl Service Available
 
Lowrate Call Girls In Sector 18 Noida ❤️8860477959 Escorts 100% Genuine Servi...
Lowrate Call Girls In Sector 18 Noida ❤️8860477959 Escorts 100% Genuine Servi...Lowrate Call Girls In Sector 18 Noida ❤️8860477959 Escorts 100% Genuine Servi...
Lowrate Call Girls In Sector 18 Noida ❤️8860477959 Escorts 100% Genuine Servi...
 

CBAP_eBook for business analyst for certification.pptx

  • 1. Lesson1–IntroductiontoCBAP®Certification CBAP®Exam Preparation Course CBAP, CCBA and BABOK are registered certification marks owned by International Institute of Business Analysis.
  • 2. WHAT’S IN IT FOR ME About IIBA ® Benefits of IIBA® Professional Certification Criteria for CBAP® certification The exam application process The exam outline Recertification requirements
  • 3. ABOUT IIBA® IIBA® Defines standards for business analysis Identifies the skills necessary to be effective in the role of a business analyst Recognizes business analysis professionals International Institute of Business Analysis Through the BABOK® Guide Through the IIBA Business Analysis Competency Model and Self-Assessment Tool Through the ECBA®, CCBA®, and CBAP® certifications
  • 4. IIBA® MEMBERSHIP BENEFITS IIBA® Membership Elevate your skill set Develop leadership skills Expand your network • BABOK® Guide • Competency Model • BBC Conference • IIBA® Webinar Series • Online Library • Attend networking and chapter events • Join online community • Learn from experts • Meet professional contacts • Volunteer with the organization • Develop communication skills
  • 5. BENEFITS OF CBAP® CERTIFICATION INDIVIDUAL  ORGANIZATION
  • 6. BENEFITS OF CBAP® CERTIFICATION (contd.) Competence in the principles and practices of business analysis Advanced career potential Helps ensure continuous self improvement Participation in a recognized professional group Personal satisfaction of accomplishing a milestone Potential for higher income Recognition of professional competence Improved overall performance, removal of uncertainty, and wider market opportunities Recognition for having invested in a business analysis career INDIVIDUAL INDIVIDUAL Benefits to the individual  ORGANIZATION
  • 7. BENEFITS OF CBAP® CERTIFICATION (contd.) ORGANIZATION Professional development, advancement, and recognition for staff Improves staff responsibility, commitment, and motivation Enables to identify professional business analysts to clients and business partners Demonstrates use of industry-standard business analysis practices Helps in establishment and implementation of business analysis practices as outlined in the BABOK® Guide Demonstrates to the stakeholders that business is run effectively Ensures more reliable and higher quality results are produced with increased efficiency and consistency Demonstrates commitment to the field of business analysis Benefits to the organization  ORGANIZATION INDIVIDUAL
  • 8. ELIGIBILITY CRITERIA CBAP® CERTIFICATION Criteria Requirements for CBAP®® Certification Work Experience Minimum 7500 Hours of Business Analysis experience in the last 10 years Knowledge Area Expertise Minimum 900 hours in four of the six knowledge areas Professional Development 35 Hours in the last 4 Years References 2 Signed Code of Conduct Yes
  • 9. Step 4 Prepare for the exam Step 1 Apply for and pay for the certification Step 2 Pay for the exam Step 3 Register for the exam APPLICATION AND EXAM PROCESS Step 5 T ake the exam     
  • 10. APPLICATION AND EXAM FEES • Application and Exam Fees • Other Fees Fee Member Non-member Application Fee (non-refundable) $125 $125 Exam Fee – English and Japanese $325 $450 Exam Fee – German *$770 $880 Fee Member Non-member Exam Cancellation Fee (CBT) $50 $50 Exam Re-write Fee – English and Japanese $250 $375 Exam Re-write Fee – German *$430 $540 NOTE: • *IIBA® is partnering with The EuropeanAssociation of BusinessAnalysis (EABA) and, in a joint effort, is now offering business analysis certification exams in German-speaking Europe and at all test center locations where IIBA exams are offered. • All fees are payable in U.S. dollars (USD) plus GST/HST if you are a Canadian resident or a GST/HST registrant. • The application fee is not refundable regardless of whether an application is approved or approved pending audit and if an application audit is not passed.
  • 11. PREPARING FOR THE EXAM The CBAP® exam is a 3.5-hours long exam and consists of 150 multiple choice questions with four possible answers to select from. To prepare for the exam: Review the BABOK® Learning Guide found in the Online Library on the Community Network Review FrequentlyAsked Questions (FAQ) on the IIBA ® website Review recommended resources on the IIBA® website Attend training as needed Review the IIBA® BABOK® Guide Find a business analysis mentor Join a study group Attend local IIBA® Chapter meetings Review available study guide(s) Find opportunities to practice tasks by following the BABOK® Guide
  • 12. CBAP® EXAM BLUEPRINT Domain Percentage of Questions Business Analysis Planning and Monitoring 14% Elicitation and Collaboration 12% Requirements Life Cycle Management 15% Strategy Analysis 15% Requirements Analysis and Design Definition 30% Solution Evaluation 14%
  • 13. RECERTIFICATION REASONS The CBAP® Recertification program helps an ongoing professional individuals demonstrate commitment to the business analysis profession. Recertification Program
  • 14. RECERTIFICATION PURPOSE The Recertification program supports the ongoing professional development of individuals who have attained the CBAP® designation. Recertification Program Encourages and recognizes individualized learning Encourages the efforts of CBAP® recipients to give back to the profession Encourages the ongoing professional development of CBAP® recipients Offers a standard and objective mechanism for recording professional development activities Sustains the global recognition and value of the CBAP® designation 
  • 15. RECERTIFICATION NOTES All CBAP® recipients are required to meet continuing proficiency requirements to maintain their designation. CERTIFICATION RENEWAL IIBA® certification to be renewed every three years CBAP® Recertification Handbook and related forms are posted on the IIBA® website ADDITIONAL NOTES To ensure receiving all IIBA® communication: -Update your email address in your profile on the IIBA® website. - Check bulk mail folders -Add certification@iiba.org to your personal address book Report 60 accepted CDUs and submit the recertification application prior to the end of the three-year cycle CDU CATEGORIES 1. Formal AcademicEducation 2.Professional Development 3.Professional Activities 4.Self-directedLearning 5.VolunteerService 6.ProfessionalExperience
  • 16. KEY TAKEAWAYS The International Institute of Business Analysis or IIBA® assists business analysts by defining standards for business analysis, identifying the skills needed for business analysis, and recognizing business analysis professionals. With an IIBA® membership, you can elevate your skill set, develop leadership skills, and expand your network. Membership of IIBA® provides several benefits, both to the individual as well as to organizations. Topreparefortheexam,reviewtheBABOK®Guide,reviewtheFAQsontheIIBA® website,attendtrainingsasneeded,practicetasksbyfollowingtheBABOK® Guide, findaBAmentor, andreviewavailablestudyguides. Applying and taking the exam is a 5-step process. The CBAP® exam is a 3.5-hours long exam and consists of 150 multiple choice questions with four possible answers to select from. The CBAP® Recertification program helps individuals demonstrate an ongoing professional commitment to the business analysis profession.
  • 17. This concludes 0Introduction to CBAP® Certification.” Thenextlessonis “Introduction toBABOK®GuideVersion3.”
  • 19. Lesson2- IntroductiontoBABOK® V3 CBAP ®Exam Preparation Course CBAP, CCBA and BABOK are registered certification marks owned by International Institute of Business Analysis.
  • 20. WHAT’S IN IT FOR ME What is business analysis? What does a business analyst do? Business Analysis Core Concept Model Model TM — the Knowledge Areas defined in BABOK® Version 3 What are the underlying competencies of a business analyst? Techniques and perspectives in business analysis
  • 21. INTRODUCTION – WHY BUSINESS ANALYSIS
  • 22. INTRODUCTION – WHY BUSINESS ANALYSIS (contd.)
  • 23.  Who is a Business Analyst?  Business Analysis Core Concept Model  Business Analysis Knowledge Areas  Re D q e u fe irc etm pe en rtU sn ciltassification  Requirements and Design  Who is a Stakeholder? Lesson 2: Introduction to BABOK® V3 Topic2.1:KeyConceptsofBusinessAnalysis
  • 24. BUSINESS ANALYSIS WHAT IS BUSINESS ANALYSIS Where? Why? When? Business Analysis Define Needs Recommend Solutions Enable
  • 25. BUSINESS ANALYSIS WHO IS A BUSINESS ANALYST Performs tasks defined in the BABOK® Guide Discovers, synthesizes, and analyzes information BUSINESS ANALYST Elicits the needs from stakeholders Investigates and clarifies stakeholder desires Determines issues and their causes
  • 26. Usefulness to a stakeholder within a context Circumstances that influence or are influenced by, and provide understanding of change A group or individual related to a change, need, or solution Way to satisfy one or more needs in a context A problem or opportunity The act of transformation in response to need Change BUSINESS ANALYSIS BUSINESS ANALYSIS CORE CONCEPT MODEL TM Solution Need Stakeholder Context Value
  • 27. BUSINESS ANALYSIS BODY OF KNOWLEDGE KNOWLEDGE AREAS 1. Business Analysis Planning and Monitoring 2. Elicitation and Collaboration 3. Requirements Lifecycle Management 4. StrategyAnalysis 5. Requirements Analysis and Design Definition 6. Solution Evaluation
  • 28. BUSINESS ANALYSIS BODY OF KNOWLEDGE (contd.) KNOWLEDGE AREAS 1. Business Analysis Planning and Monitoring 2. Elicitation and Collaboration 3. Requirements Lifecycle Management 4. StrategyAnalysis 5. Requirements Analysis and Design Definition 6. Solution Evaluation It describes the tasks to be performed to organize and co-ordinate the effort of business analysts and stakeholders. Output from this knowledge area tasks is used as a key input or as guidelines for the tasks performed in other knowledge areas.
  • 29. BUSINESS ANALYSIS BODY OF KNOWLEDGE (contd.) KNOWLEDGE AREAS 1. Business Analysis Planning and Monitoring 2. Elicitation and Collaboration 3. Requirements Lifecycle Management 4. StrategyAnalysis 5. Requirements Analysis and Design Definition 6. Solution Evaluation It describes the tasks to be performed to prepare for elicitation, conduct the elicitation activity, and confirm elicitation results. It also explains how communication of business analysis information and ongoing collaboration must be done with the stakeholder.
  • 30. BUSINESS ANALYSIS BODY OF KNOWLEDGE (contd.) KNOWLEDGE AREAS 1. Business Analysis Planning and Monitoring 2. Elicitation and Collaboration 3. Requirements Lifecycle Management 4. StrategyAnalysis 5. Requirements Analysis and Design Definition 6. Solution Evaluation It describes the tasks to be performed to manage and maintain requirements and design information from inception to retirement.
  • 31. BUSINESS ANALYSIS BODY OF KNOWLEDGE (contd.) KNOWLEDGE AREAS 1. Business Analysis Planning and Monitoring 2. Elicitation and Collaboration 3. Requirements Lifecycle Management 4. StrategyAnalysis 5. Requirements Analysis and Design Definition 6. Solution Evaluation It describes the tasks to be performed for collaboration with the stakeholder to identify the real need and enable the organization to address that need. It also explains the tasks to align the resulting strategy for the change with higher- and lower-level strategies to address the need.
  • 32. BUSINESS ANALYSIS BODY OF KNOWLEDGE (contd.) KNOWLEDGE AREAS 1. Business Analysis Planning and Monitoring 2. Elicitation and Collaboration 3. Requirements Lifecycle Management 4. StrategyAnalysis 5. Requirements Analysis and Design Definition 6. Solution Evaluation It describes the tasks to be performed to structure and organize requirements discovered during the elicitation activity, specify and model requirements and design, validate and verify information, and identify solution options that meet the business need.
  • 33. BUSINESS ANALYSIS BODY OF KNOWLEDGE (contd.) KNOWLEDGE AREAS 1. Business Analysis Planning and Monitoring 2. Elicitation and Collaboration 3. Requirements Lifecycle Management 4. StrategyAnalysis 5. Requirements Analysis and Design Definition 6. Solution Evaluation It describes the tasks to be performed to assess the performance of and value delivered by a solution and recommend actions to be taken to realize the full value of the solution.
  • 34. BUSINESS ANALYSIS BODY OF KNOWLEDGE KNOWLEDGE AREAS – RELATIONSHIPS Business Analysis Planning and Monitoring Strategy Analysis Requirements Analysis and Design Definition Solution Evaluation Requirements Lifecycle Management Elicitation and Collaboration
  • 35. REQUIREMENTS REQUIREMENTS CLASSIFICATION Business Requirements Stakeholder Requirements Solution Requirements Transition Requirements Describe the needs of the stakeholders that must be met to achieve the business requirements Statements of goals, objectives, andoutcomethatdescribewhya changehasbeeninitiated Describe the capabilities that the solution must have to facilitate transition of the organization from the current state to the future state Describe the capabilities and qualities of a solution that meets stakeholder requirements; can be Functional requirements or Non- functional requirements
  • 36. REQUIREMENTS (contd.) REQUIREMENTS DOCUMENTS Business Requirements Documents that comprise both business and stakeholder requirements Functional Specification Document Solution Requirements Specification R R e e q qu uiir re em me e n n tt s s D D o o c c u u m m e e n n t t s s BRD FSD SRS
  • 37. REQUIREMENTS AND DESIGN DESIGN CYCLE Requirements Design Needs Solution Requirements lead to design. Requirements may be used to define design. Requirements are focused on needs and design is focused on the solution. Requirements and design could be recursive. A Business Analyst reviews designs to ensure they align with the requirements.
  • 38. BABOK® GUIDE SPECIFICATIONS STAKEHOLDERS Business Analyst Project Manager Implementation SME Regulator Sponsor Supplier Customer End User Domain SME Tester Operational Support A stakeholder is an individual or group that a Business Analyst is likely to interact with directly or indirectly.
  • 39.  Competencies of a business analyst Lesson 2: Introduction to BABOK® V3 Topic2.2:CompetenciesofaBusinessAnalyst
  • 40. COMPETENCIES OF A BUSINESS ANALYST CATEGORIES Analytical Thinking and Problem- Solving Skills (7) Behavioral Characteristics (5) Business Knowledge (5) Communication Skills (4) Tools and T echnology (3) Interaction Skills (5) Competencies
  • 41. COMPETENCIES OF A BUSINESS ANALYST (contd.) ANALYTICAL THINKING AND PROBLEM SOLVING Creative Thinking Ability to generate new ideas and approaches to solve problems Learning Ability to quickly absorb new and different types of information Conceptual Thinking Ability to understand how disparate information fits into a larger picture Decision Making Ability to understand the criteria needed to make decisions Ability to understand the underlying cause of a problem and use problem- solving techniques Visual Thinking Ability to visually communicate complex concepts and models to stakeholders     Problem Solving   System Thinking Ability to understand the interactions between people, process, and technology  Analytical Thinking and Problem-Solving Skills
  • 42. COMPETENCIES OF A BUSINESS ANALYST (contd.) BEHAVIORAL CHARACTERISTCS Behavioral Characteristics Ethics Enable a business analyst to earn the respect of stakeholders Trustworthiness Perception that one is worthy of trust Personal Accountability Enables a business analyst to achieve target goals Organization and Time Management Helps perform tasks effectively and use work time efficiently     Adaptability Ability to adjust behavioral style, methods, and approach to the environment 
  • 43. COMPETENCIES OF A BUSINESS ANALYST (contd.) BUSINESS KNOWLEDGE Business Knowledge Business Acumen Ability to understand business principles and practices and apply them at work Organization Knowledge Understanding the management structure and the business architecture of the enterprise Industry Knowledge Understanding how a company is positioned within an industry Solution Knowledge Understanding solutions that will help expedite the discovery of potential changes through elicitation     Methodology Knowledge Enables a business analyst to quickly adapt to, and perform in, a new environment 
  • 44. COMPETENCIES OF A BUSINESS ANALYST (contd.) COMMUNICATION SKILLS Communication Skills Verbal Communication Used to convey ideas, concepts, facts, and opinions to stakeholders Written Communication Practice of using text, symbols, models, pictures, or sketches to convey and share information Enables sending and receiving messages through body movements, postures, facial expressions, gestures, and eye contact Nonverba  l Communication Listening Enables accurate understanding of verbal communication    Communication skills refers to the ability to communicate effectively.
  • 45. COMPETENCIES OF A BUSINESS ANALYST (contd.) INTERACTION SKILLS Interaction Skills Facilitation Ability to moderate discussions within a group Teamwork Enables working with team members effectively Leadership and Influencing Involves motivating people to act in ways that enable them to work together Negotiation Involves mediating discussions and resolving differences in teams     Teaching Enables a business analyst to effectively communicate business information, concepts, ideas, and issues  Interaction Skills include the ability to relate to and co- operate and communicate with different kinds of people.
  • 46. COMPETENCIES OF A BUSINESS ANALYST (contd.) TOOLS AND TECHNOLOGY SKILLS Tools and Technology Skills Office Productivity Helps document and track information and artifacts Business Analysis Helps model, diagram, document, and manage the output of business analysis activities   Helps perform business analysis activities, manage teams, and collaborate with stakeholders Com  munication A business analyst uses a variety of software to support communication and collaboration, create and maintain requirement artifacts, and increase overall productivity.
  • 47.  T eDcehfneicqtupeesrrUefneirtredto by BABOK V3 Lesson 2: Introduction to BABOK® V3 Topic2.3:BusinessAnalysisTechniques
  • 48. BUSINESS ANALYSIS TECHNIQUES 50 TECHNIQUES 10.1 Acceptance and Evaluation Criteria 10.11 Concept Modelling 10.21 Focus Groups 10.31 Observation 10.41 Scope Modelling 10.2 Backlog Management 10.12 Data Dictionary 10.22 Functional Decomposition 10.32 Organizational Modelling 10.42 Sequence Diagrams 10.3 Balanced Scorecard 10.13 Data Flow Diagrams 10.23 Glossary 10.33 Prioritization 10.43 Stakeholder List, Map, or Personas 10.4 Benchmarking and Market Analysis 10.14 Data Mining 10.24 Interface Analysis 10.34 Process Analysis 10.44 State Modelling 10.5 Brainstorming 10.15 Data Modelling 10.25 Interviews 10.35 Process Modelling 10.45 Survey or Questionnaire 10.6 Business Capability Analysis 10.16 Decision Analysis 10.26 Item Tracking 10.36 Prototyping 10.46 SWOT Analysis 10.7 Business Cases 10.17 Decision Modelling 10.27 Lessons Learned 10.37 Reviews 10.47 Use Cases and Scenarios 10.8 Business Model Canvas 10.18 Document Analysis 10.28 Metrics and Key Performance Indicators (KPIs) 10.38 Risk Analysis and Management 10.48 User Stories 10.9 Business Rules Analysis 10.19 Estimation 10.29 Mind Mapping 10.39 Roles and Permissions Matrix 10.49 Vendor Assessment 10.10 Collaborative Games 10.20 Financial Analysis 10.30 Non functional Requirements Analysis 10.40 Root Cause Analysis 10.50 Workshops
  • 49.  Bu D s e in fe ec stspA en r a U ly ns itis perspectives Lesson 2: Introduction to BABOK® V3 Topic2.4:BusinessAnalysisPerspectives
  • 50. BUSINESS ANALYSIS PERSPECTIVES FIVE BUSINESS ANALYSIS PERSPECTIVES Perspectives provide focus to tasks and techniques specific to the context of an initiative. Perspectives are not mutually exclusive. Most initiatives are likely to engage one or more perspectives. BABOK® includes five perspectives. Agile Business Intelligence Information Technology Business Architecture Business Process Management
  • 51. BUSINESS ANALYSIS PERSPECTIVES (contd.) FIVE BUSINESS ANALYSIS PERSPECTIVES Agile Business Intelligence Information Technology Business Architecture Business Process Management An Agile initiative involves: • An agile mindset • Agile values and principles • Progressive elaboration of business planning, elicitation, analysis, design, and construction of solution A BusinessAnalyst is an active member of an Agile team.
  • 52. BUSINESS ANALYSIS PERSPECTIVES (contd.) FIVE BUSINESS ANALYSIS PERSPECTIVES Agile Business Intelligence Information Technology Business Architecture Business Process Management Business Intelligence: • Highlights the characteristics of business analysis in the context of transformation, integration, and enhancing data • Is the transformation of data into valuable information • Helps stakeholders make informed decisions • Helps stakeholders manage strategic, tactical, and operational performance
  • 53. BUSINESS ANALYSIS PERSPECTIVES (contd.) FIVE BUSINESS ANALYSIS PERSPECTIVES Agile Business Intelligence Information Technology Business Architecture Business Process Management Information Technology: • This perspective highlights the characteristics of business analysis in the context of the impact of change on IT systems in an organization. • Organizations need to undertake initiatives to upgrade or replace IT systems. • A business analyst plays the role of a translator between business and technology teams in the change process.
  • 54. BUSINESS ANALYSIS PERSPECTIVES (contd.) FIVE BUSINESS ANALYSIS PERSPECTIVES Agile Business Intelligence Information Technology Business Architecture Business Process Management BusinessArchitecture: • Highlights the unique characteristics of business analysis in the context of business architecture • Involves analytical thinking and applying architectural principles at the enterprise level • Supports ongoing business transformation efforts • Could include changes to business, operational model, or structure
  • 55. BUSINESS ANALYSIS PERSPECTIVES (contd.) FIVE BUSINESS ANALYSIS PERSPECTIVES Agile Business Intelligence Information Technology Business Architecture Business Process Management Business Process Management: • Highlights the characteristics of business analysis in the context of developing or improving business processes • Focuses on how an organization works across multiple functional areas to improve business processes • Is an integral part of ongoing management and operations of an organization
  • 56. Quiz
  • 57. a. b. c. d. Which of the following is NOT a component of the Business Analysis Core Concept Model TM (BACCMTM)? QUIZ 1 Stakeholder Content Solution Change
  • 58. The correct answer is c. Explanation: Content is not a component of Business Analysis Core Concept Model TM (BACCM). The components are Change, Need, Solution, Stakeholder, Value, and Context. a. b. c. d. Which of the following is NOT a component of the Business Analysis Core Concept Model TM (BACCMTM)? QUIZ 1 Change Solution Content Stakeholder
  • 59. a. b. c. d. are focused on the needs, and are focused on the solution. QUIZ 2 Designs; Requirements Solutions; Requirements Requirements; Designs Problems; Requirements
  • 60. The correct answer is b. Explanation: According to the Requirements and Design Cycle, Requirements are focused on the needs and Designs are focused on the solution. a. b. c. d. are focused on the needs, and are focused on the solution. QUIZ 2 Problems; Requirements Requirements; Designs Solutions; Requirements Designs; Requirements
  • 61. a. b. c. d. Which of the following is a business analysis knowledge area? QUIZ 3 Requirements Analysis and Management Solution Assessment and Validation Enterprise Analysis Elicitation and Collaboration
  • 62. The correct answer is a. Explanation: Elicitation and Collaboration is one of the business analysis knowledge areas. . a. b. c. d. Which of the following is a business analysis knowledge area? QUIZ 3 Elicitation and Collaboration Enterprise Analysis Solution Assessment and Validation Requirements Analysis and Management
  • 63. a. b. c. d. Which of the following is NOT a category of business analyst competencies as defined in BABOK®? QUIZ 4 Interaction Organizational Knowledge Communication Analytical Thinking and Problem Solving
  • 64. The correct answer is c. Explanation: Organizational Knowledge is not a category of business analyst competencies. It is, in fact, a core competency of the Business Knowledge category. a. b. c. d. Which of the following is NOT a category of business analyst competencies as defined in BABOK®? QUIZ 4 Analytical Thinking and Problem Solving Communication Organizational Knowledge Interaction
  • 65. KEY TAKEAWAYS Business analysis is the practice of enabling change in an enterprise by defining needs and recommending solutions that deliver value to stakeholders. A business analyst is a person who performs business analysis tasks described in the BABOK® Guide. The BusinessAnalysis Core Concept Model TM (BACCMTM) defines the conceptual framework for the practice of business analysis. The BABOK® Guide groups business analysis tasks into six knowledge areas. The BABOK® GUIDE describes 29 business analysis competencies, 50 techniques, and 5 perspectives.
  • 66. This concludes Introduction to BABOK® V3. Thenextlessonis“Business AnalysisPlanningandMonitoring.”
  • 68. Lesson3- BusinessAnalysisPlanningandMonitoring CBAP®Exam Preparation Course CBAP, CCBA and BABOK are registered certification marks owned by International Institute of Business Analysis.
  • 69. WHAT’S IN IT FOR ME How to plan the Business Analysis Approach How to plan Stakeholder Engagement How to plan Business Analysis Information Management and Governance How to identify Business Analysis Performance Improvements Understand tools and techniques used in Business Analysis Planning and Monitoring
  • 71. BUSINESS ANALYSIS PLANNING AND MONITORING Circumstances that influence or are influenced by, and provide understanding of change Way to satisfy one or more needs in a context A problem or opportunity INTRODUCTION The act of transformation in response to need Change Solution Need A group or individual related to a change, need, or solution Stakeholder Context Value Usefulness to a stakeholder within a context
  • 72. BUSINESS ANALYSIS PLANNING AND MONITORING INPUT , TASKS, AND OUTPUT External Influences Internal Influences Needs • Business AnalysisApproach • Stakeholder Engagement Approach • Governance Approach • Information Management Approach • BusinessAnalysis Performance Assessment Plan BusinessAnalysis Approach Plan Stakeholder Engagement Plan BusinessAnalysis Governance Plan BusinessAnalysis Information Management Identify Business Analysis Performance Improvements
  • 73.  define overall method to conduct business analysis activities  Defect per Unit Lesson 3: Business Analysis Planning and Monitoring Topic3.1:PlanBusinessAnalysisApproach
  • 74. PLAN BUSINESS ANALYSIS APPROACH PURPOSE Need • Creation or Selection of Methodology • Identify BA Deliverables • Define Activities and Tasks Planning of Business Analysis • Who will perform • Timing • Sequencing Business Analysis Approach Input Output
  • 75. PLAN BUSINESS ANALYSIS APPROACH ELEMENTS Planning methods falls between Predictive and Adaptive approaches Formality and level of details of BA deliverables are based on the planning approach Identify and break down activities into tasks Timing for the business analysis tasks to be performed Complexity and size of the change, and overall risk Business analysis approach reviewed and agreed upon by key stakeholders
  • 76. PLAN BUSINESS ANALYSIS APPROACH GUIDELINES AND TOOLS Guidelines and Tools while planning BA approach BusinessAnalysis Performance Assessment Stakeholder Engagement Approach Guidance while planning BA approach Business Policies Methodologies and Framework Expert judgement
  • 77. PLAN BUSINESS ANALYSIS APPROACH Brainstorming Business Cases Estimation TECHNIQUES Functional Document Analysis Financial Analysis Decomposition Interviews Item Tracking Lessons Learned Process Modeling Reviews Risk Analysis and Management Scope Modeling Survey and Questionnaire Workshops 1 2 3 4 5 6 7 8 9 10 11 12 13 14 15
  • 78. PLAN BUSINESS ANALYSIS APPROACH TECHNIQUE – LESSONS LEARNED – WHY? WHY? Lessons Learned Successes Opportunities for improvement Failures Recommendations for improvement
  • 79. PLAN BUSINESS ANALYSIS APPROACH TECHNIQUE – LESSONS LEARNED - OVERVIEW Lessons Learned Also known as Retrospective Formal or Informal Reinforcing positive experiences and successes ! Participants may be reluctant to document and discuss problems. Honest discussion may not occur.
  • 80. PLAN BUSINESS ANALYSIS APPROACH TECHNIQUE – LESSONS LEARNED - ELEMENTS Business analysis activities and deliverables The final solution, service or product Impact to organizational processes Performance expectations and results Positive or negative variance Root cause of results impacting performance Recommendations for behavioral approaches
  • 81. Domain Subject Matter Expert Project Manager Regulator Sponsor Stakeholders PLAN BUSINESS ANALYSIS APPROACH STAKEHOLDERS
  • 82.  plan an approach for establishing and maintaining effective working relationship with stakeholders  Defect per Unit Lesson 3: Business Analysis Planning and Monitoring Topic3.2:PlanStakeholderEngagement
  • 83. PLAN STAKEHOLDER ENGAGEMENT PURPOSE Plan an approach for establishing and maintaining effective working relationship with stakeholders Identify Stakeholder Analyze Stakeholder Characteristics Consider Stakeholder complexity while planning for stakeholder engagement. Needs and BA Approach Plan Stakeholder Engagement Approach
  • 84. PLAN STAKEHOLDER ENGAGEMENT ELEMENTS Perform Stakeholder Analysis Roles Attitude Decision Making Authority Level of Power or Influence Stakeholder Communication Needs Planning Stakeholder Collaboration
  • 85. PLAN STAKEHOLDER ENGAGEMENT GUIDELINES AND TOOLS Guidelines and Tools while planning Stakeholder Engagement BusinessAnalysis Performance Assessment Change Strategy Current State Description
  • 86. PLAN STAKEHOLDER ENGAGEMENT TECHNIQUES Business Rules Brainstorming Analysis Document Analysis Interviews Lessons Learned Mind Mapping Organizational Modeling Process Modeling Stakeholder List, Map or Personas Risk Analysis and Management Scope Modeling Survey & Questionnaire Workshops 1 2 3 4 5 6 7 8 9 10 11 12 13
  • 87. ORGANIZATIONAL MODEL PURPOSE Organizational Modeling • Describe the roles, responsibilities, and reporting structure • Align these structures with the organizational goals Organizational Model • Boundaries of a group • Formal relationships between members • Functional role of each person • Interfaces between unit or stakeholders • Common in most organizations
  • 88. ORGANIZATIONAL MODEL ELEMENTS Types of Organizational Models Functionally oriented Market oriented Matrix model Organization unit includes number of roles Interfaces with other organization units
  • 89. ORGANIZATIONAL MODEL ELEMENTS (contd.) Identify informal lines of communication which can influence business analysis activities Organization Chart Box depicts organization units, roles and people Line depicts reporting or formal communication channel
  • 90. ORGANIZATIONAL MODEL EXAMPLE Business Unit Head Program Management Sr. Project Manager Project Manager Development Development Manager Lead Quality Analyst Architect Operations Operations Manager Development Infrastructure Manager Logistics Finance Sr. Finance Manager Auditor Human Resource Sr. Human Resource Manager On boarding specialist Sales and Marketing Marketing Manager Brand Manager Sales Manager Event Manager Purchase Sr. Purchase manager Vendor Manager
  • 91. STAKEHOLDER LIST, MAP, OR PERSONAS OVERVIEW Identify Stakeholder Stakeholder Analysis • Identifying stakeholders List of stakeholders Stakeholder characteristics • Level of authority within the domain of change • Attitude toward or interest in the change • Attitude toward business analysis activities • Level of decision-making authority
  • 92. STAKEHOLDER LIST, MAP OR PERSONAS ELEMENTS Stakeholder Map • Stakeholder Matrix • Stakeholder Onion Diagram RACI Matrix (Responsible, Accountable, Consulted and Informed) Personas
  • 93. STAKEHOLDER LIST, MAP, OR PERSONAS EXAMPLE Ensure stakeholders remain satisfied Work closely with stakeholders and engage with them effectively Monitor to ensure stakeholder interest or influence does not change Keep stakeholders informed as they are likely to be very concerned Low Low High High Interest / Impact of Stakeholder Influence of Stakeholder
  • 94. STAKEHOLDER LIST, MAP OR PERSONAS STAKEHOLDER ONION DIAGRAM Solution Delivery Affected Organizational Unit Organization or Enterprise Affected External Stakeholders Customers, suppliers, regulators, and others Sponsors, executives, domain SMEs, and others who interact with the affected group End users, help desk, and other whose work changes when solution is delivered Project team and others directly involved with creating the solution
  • 95. STAKEHOLDER LIST, MAP OR PERSONAS RACI – RESPONSIBLE, ACCOUNTABLE, CONSULTED, INFORMED Task Sponsor Project Manager Implementation Team Operations Business Analyst Regulators Identify problem or opportunity R R C R A I Identify risk associated with business analysis work R R R R A I Conduct feasibility study C R R C A Recommend solution R C R R A I Approve Business Analysis Deliverables A C C C R I Analyze Requirements I I C I A Communicate Requirements and Design I C I I A I Assess solution performance C C C A R C
  • 96. STAKEHOLDER LIST, MAP OR PERSONAS PERSONAS Paul Atkins, Marketing Manager (Health Care) 32 Years, Married, One Child Role What is your job role? What is a typical day like? What skills, knowledge, and tools do you require? Who reports to you and whom do you report to? Goals What are you responsible for? Challenges What are your biggest challenges in your work? Company Which industry does your company work in? What is the total revenue of your company? How many employees work in your company? Personal Background: Age, Family (single, married, children), Education Preferences How do you prefer to interact (email, phone, in person)?
  • 97. PLAN STAKEHOLDER ENGAGEMENT STAKEHOLDERS Customer End User Supplier Regulator Sponsor Project Manager Domain Subject Matter Expert
  • 98.  define how decisions are made about requirements and designs, including reviews, change control, approval and prioritization  Defect per Unit Lesson 3: Business Analysis Planning and Monitoring Topic3.3:PlanBusinessAnalysisGovernance
  • 99. PLAN BUSINESS ANALYSIS GOVERNANCE PURPOSE How decisions are made about requirements and designs, including reviews, change control, approval and prioritization. BAApproach and Stakeholder Engagement Approach Governance Approach
  • 100. PLAN BUSINESS ANALYSIS GOVERNANCE ELEMENTS Elements Decision Making Change Control Process Plan Prioritization Approach Planfor Approval
  • 101. PLAN BUSINESS ANALYSIS GOVERNANCE GUIDELINES AND TOOLS Guidelines and Tools while planning Business Analysis Governance BusinessAnalysis Performance Assessment Business Policies Current State Description Legal / Regulatory Information
  • 102. PLAN BUSINESS ANALYSIS GOVERNANCE TECHNIQUES Brainstorming Document Analysis Interviews Lessons Learned Organizational Modeling Item Tracking Process Modeling Survey & Questionnaire Workshops 1 2 3 4 5 6 7 8 9 10 Reviews
  • 103. PLAN BUSINESS ANALYSIS GOVERNANCE STAKEHOLDERS Domain Subject Matter Expert Project Manager Regulator Sponsor
  • 104.  Defect per Unit  Storing and accessing business analysis information Lesson 3: Business Analysis Planning and Monitoring Topic3.4:PlanBusinessAnalysisInformationManagement
  • 105. PLAN BUSINESS ANALYSIS INFORMATION MANAGEMENT PURPOSE How to develop business analysis information management plan Business analysis information management approach • Business Analysis Approach • Stakeholder Engagement Approach • Governance Approach Inputs Output
  • 106. PLAN BUSINESS ANALYSIS INFORMATION MANAGEMENT ELEMENTS Organization of Business Analysis information Level of Abstraction Plan Traceability Approach Plan for Requirements Reuse Storage and Access Requirements Attributes
  • 107. PLAN BUSINESS ANALYSIS INFORMATION MANAGEMENT GUIDELINES AND TOOLS Guidelines and Tools while planning Business Analysis Information Management BusinessAnalysis Performance Assessment Business Policies Information Management Tools Legal / Regulatory Information
  • 108. PLAN BUSINESS ANALYSIS INFORMATION MANAGEMENT TECHNIQUES Brainstorming Interviews Lessons Learned Mind Mapping Item Tracking Process Modeling Surveys and Questionnaires Workshops 1 2 3 4 5 6 7 8
  • 109. PLAN BUSINESS ANALYSIS INFORMATION MANAGEMENT STAKEHOLDERS Regulator
  • 110.  Assess business analysis work and to plan to improve processes where required  Defect per Unit Lesson 3: Business Analysis Planning and Monitoring Topic3.5:Identify BusinessAnalysisPerformanceImprovements
  • 111. IDENTIFY BUSINESS ANALYSIS PERFORMANCE IMPROVEMENTS PURPOSE Assess Business Analysis Work and Plan to Improve Processes BAApproach, and Performance Objectives Business Analysis Performance Assessment Input Output
  • 113. IDENTIFY BUSINESS ANALYSIS PERFORMANCE IMPROVEMENTS GUIDELINES AND TOOLS Organizational Performance Standards includes details of performance metrics or expectations for business analysis work mandated by the organization.
  • 114. IDENTIFY BUSINESS ANALYSIS PERFORMANCE IMPROVEMENTS TECHNIQUES Brainstorming Interviews Lessons Learned Process modeling Item Tracking Process Analysis Metrics and Key Performance Indicators (KPIs) 1 2 3 4 5 8 9 10 6 7 11 12 13 Observation Reviews Risk Analysis and Management Root Cause Analysis Surveys and Questionnaires Workshops
  • 115. METRICS AND KEY PERFORMANCE INDICATORS (KPIs) OVERVIEW Metrics and Key Performance Indicators (KPIs) measure the performance of solutions, solution components, and other matters of interest to stakeholders. A Metrics is quantifiable level of an indicator. A Key Performance Indicator (KPI) measures progress towards a strategic goal or objective. KPIs derive from metrics. KPIs to measure the effectiveness of business analysis. Reporting is the process of informing stakeholder of metrics or indicators in specified format and specified intervals.
  • 116. METRICS AND KEY PERFORMANCE INDICATORS (KPIs) ELEMENTS Characteristics of Indicator • Clear • Relevant • Economical • Adequate • Quantifiable • Trustworthy and Credible Metrics • Specific point • Threshold • Range Structure • Monitoring • Evaluation Reporting • Baseline • Current and Target metrics Quality of metrics and KPIs • Reliability • Validity • Timeliness
  • 117. IDENTIFY BUSINESS ANALYSIS PERFORMANCE IMPROVEMENTS STAKEHOLDERS Stakeholders Domain Subject Matter Expert Project Manager Sponsor
  • 118.  Defect per Unit Lesson 3: Business Analysis Planning and Monitoring CASESTUDYEXERCISE
  • 119. CASE STUDY OVERVIEW BATONICS Pharma Company Unable to deliver products on time Customers switching to competitor products
  • 120. CASE STUDY BUSINESS ANALYSIS ACTIVITIES Identified the business analysis deliverables: Documentation of existing supply chain management process Root cause analysis Solution options Recommendation of tools and technologies Recommended optimal solution
  • 121. CASE STUDY BUSINESS ANALYSIS ACTIVITIES You have identified and analyzed the stakeholders involved in the process. You have gone through existing documents, meetings with subject matter experts and people involved in this process and come up with the activities and task. You have also estimated the task and who are involved in performing the task. You have established the process of decision making, change control, prioritization, and approval. You have determined how are you going to store and retrieve business analysis information. You have had meetings with the key stakeholders to review and get approval on the business analysis approach.
  • 122. CASE STUDY EXERCISE Questions Response 1 Which document is created to define decision making, change control, prioritization and approval process? Information Management Approach BA Governance Approach 2 Which methodology is used to define business analysis approach? Adaptive Predictive 3 Which document specifies the level of responsibility expected from each stakeholder? RACI Matrix Stakeholder Matrix 4 Which technique is used to identify roles and responsibilities within the organization? Organizational Modeling Technique BA Performance Assessment 5 Which document indicates how stakeholder is involved with the solution? Stakeholder Matrix Stakeholder Onion Diagram 6 Which task is performed to identify and analyze the stakeholders? Plan Stakeholder Engagement Personas 7 Among stakeholders, who are not likely to get involved in this initiative? Regulators Domain Subject Matter Expert
  • 123. CASE STUDY ANSWERS Questions Answers 1 Which document have your created to define decision making, change control, prioritization and approval process? BA Governance Approach 2 Which methodology is used to define business analysis approach? Adaptive 3 Which document specifies the level of responsibility expected from each stakeholder? RACI Matrix 4 Which technique is used to identify roles and responsibilities within organization? Organizational Modeling 5 Which document indicates how stakeholder is involved with the solution? Stakeholder Onion Diagram 6 Which task is performed to identify and analyze stakeholders? Plan Stakeholder Engagement 7 Among stakeholders, who are not likely to get involved in this initiative? Regulators, Customers
  • 124. Quiz
  • 125. QUIZ 1 a. b. c. d. What does RACI stand for? Responsible, Accountable, Collaborate, Informed Responsible, Author, Consulted, Informed Responsible, Accountable, Consulted, Informed Responsible, Authorized, Consulted, Informed
  • 126. QUIZ 1 The correct answer is c. Explanation: RACI stands for Responsible, Accountable, Consulted, Informed a. b. c. d. What does RACI stand for? Responsible, Accountable, Collaborate, Informed Responsible, Author, Consulted, Informed Responsible, Accountable, Consulted, Informed Responsible, Authorized, Consulted, Informed
  • 127. a. b. c. d. Which one of the following is not a characteristics of an indicator QUIZ 2 Relevant Economical Transparent Adequate
  • 128. QUIZ 2 The correct answer is c Explanation: Transparent is not a characteristics of an indicator. Characteristics of an indicator are Clear, Relevant, Economical, Adequate, Quantifiable, Trustworthy and Credible a. b. c. d. Which one of the following is not a characteristics of an indicator Relevant Economical Transparent Adequate
  • 129. QUIZ 3 a. b. c. d. Which one of the following is not an input to plan business analysis information management approach? Business Analysis Governance Approach Business Analysis Approach Stakeholder Engagement Approach Business Analysis Traceability Approach
  • 130. QUIZ 3 The correct answer is d Explanation: Business Analysis Traceability Approach is not an input to plan business analysis information management approach. Traceability approach is one of the element of Business Analysis Information Management Approach a. b. c. d. Which one of the following is not an input to plan business analysis information management approach? Business Analysis Governance Approach Business Analysis Approach Stakeholder Engagement Approach Business Analysis Traceability Approach
  • 131. KEY TAKEAWAYS (1 of 2) BA approach or plan documents - BA approach, Stakeholder engagement approach, BA governance approach, and BA information management approach. Business Analysis approach or plan is developed based on need, methodology, complexity, size and risk. The stakeholder engagement, governance, and information management plan can be separate or part of the business analysis plan, depending on the size of the initiative or organizational standard. Output of business analysis planning and monitoring knowledge area governs other tasks in all knowledge area.
  • 132. KEY TAKEAWAYS (contd.) Business analysis plan documents are reviewed and approved by key stakeholders. Performance baseline, metrics and KPIs, and the process of monitoring and evaluation are defined to improve performance of business analysis work. Business Analysis performance assessment is used as a guideline in all planning tasks. Business analysis techniques – Lessons Learned, Organizational Modeling, Stakeholder List, Map or Persona, RACI Matrix, Metrics and Key Performance Indicators or KPIs – are used when performing business analysis tasks.
  • 133. This concludes 0Business Analysis Planning and Monitoring.” Thenextlessonis“Elicitation andCollaboration.”
  • 135. Lesson4- ElicitationandCollaboration CBAP®Exam Preparation Course CBAP, CCBA and BABOK are registered certification marks owned by International Institute of Business Analysis.
  • 136. WHAT’S IN IT FOR ME Confirm Elicitation Results Prepare and Conduct Elicitation Communicate Business Analysis Information Manage Stakeholder Collaboration
  • 137. ELICITATION AND COLLABORATION KNOWLEDGE AREA OVERVIEW Change Solution Need Stakeholder Context Value
  • 138. ELICITATION AND COLLABORATION KNOWLEDGE AREA (contd.) OVERVIEW Identify business analysis information about the context that may effect the change Manage collaboration with the stakeholders • Elicit, confirm and communicate needs • Needs may evolve over time • Identify the characteristics of the change • Identify stakeholders concerns • Determine the extent of elicitation and collaboration required Change Solution Elicit, confirm and communicate desired characteristics of proposed solutions Need Stakeholder Context Value Assess the relative value of information provided during elicitation
  • 139. ELICITATION AND COLLABORATION KNOWLEDGE AREA TASKS AND OUTPUT Tasks 1. Prepare for Elicitation 2. Conduct Elicitation 3.Confirm Elicitation Results 4.Communicate Business Analysis Information 5.Manage Stakeholder Collaboration
  • 140. ELICITATION AND COLLABORATION KNOWLEDGE AREA (contd.) 1. Prepare for Elicitation 2. Conduct Elicitation 4.Communicate Business Analysis Information 5.Manage Stakeholder Collaboration 3. Confirm Elicitation Results Tasks Inputs • Needs • Business Analysis Information Input from other Knowledge Areas • Stakeholder Engagement Approach • Business Analysis Performance Assessment TASKS AND OUTPUT Sequential tasks
  • 141. ELICITATION AND COLLABORATION KNOWLEDGE AREA (contd.) 1. Prepare for Elicitation 2. Conduct Elicitation 4.Communicate Business Analysis Information 5.Manage Stakeholder Collaboration 3. Confirm Elicitation Results Tasks Outputs Inputs • Needs • Business Analysis Information Input from other Knowledge Area • Stakeholder Engagement Approach • Business Analysis Performance Assessment Elicitation Activity plan Elicitation Results (unconfirmed) Business Analysis Information (communicated) Stakeholder Engagement Elicitation Results (confirmed) TASKS AND OUTPUT
  • 142.  Overview  Elements  GD uie dfe elc in te p s ea r n U d ni T tools  Techniques  Stakeholders Lesson 4: Elicitation and Collaboration Topic4.1:PrepareforElicitation
  • 143. PREPARE FOR ELICITATION OVERVIEW Purpose  Understand the scope of elicitation activity  Select appropriate technique  Plan for supporting material and resources I N P U T S O U T P U T S  Needs  Stakeholder Engagement Approach Elicitation Activity Plan:  Logistics  Scope of the elicitation activity  Selected techniques  Supporting Material
  • 144. PREPARE FOR ELICITATION ELEMENTS Understand Scope Select Technique Set up Logistics Secure Supporting Material Prepare Stakeholders  
  • 145. PREPARE FOR ELICITATION GUIDELINES AND TOOLS Business Analysis Approach Business Objectives Potential Value Existing Business Analysis
  • 146. PREPARE FOR ELICITATION (contd.) GUIDELINES AND TOOLS Business Analysis Approach Business Objectives Potential Value Existing Business Analysis Provides a general strategy to be used as a guide Provides directions towards future state Describes the value to be realized Provides better understanding of scope
  • 147. PREPARE FOR ELICITATION TECHNIQUES Stakeholder List, Map, or Personas Brainstorming Mind Mapping Data Mining Estimation Interviews Risk Analysis and Management Document Analysis
  • 148. Stakeholder List, Map or Personas Brainstorming Mind Mapping Data Mining Estimation Interviews Risk Analysis and Management Document Analysis PREPARE FOR ELICITATION (contd.) TECHNIQUES Who needs to be consulted? Identify information for further investigation Identify existing business analysis information Select elicitation technique Estimate time, effort and cost Identify concern Identify source of supporting material Identify and assess risk
  • 149. PREPARE FOR ELICITATION STAKEHOLDERS Domain SME Project Manager Sponsor Business Analyst
  • 150. PREPARE FOR ELICITATION (contd.) STAKEHOLDERS Ensures the appropriate people and resources are available Approves or denies a planned elicitation event Provides supporting material and guidance. Domain SME Sponsor Business Analyst Project Manager
  • 151.  Overview  Elements  GD uie dfe elc in te p s ea r n U d ni T tools  Techniques  Stakeholders Lesson 4: Elicitation and Collaboration Topic4.2:ConductElicitation
  • 152. CONDUCT ELICITATION OVERVIEW Purpose Information relevant to change:  Draw out  Explore  Identify I N P U T S O U T P U T  Supporting material  Resources  Activity Plan Unconfirmed Elicitation Results
  • 153. CONDUCT ELICITATION (contd.) OVERVIEW  Interacts with the stakeholder  Relies on the stakeholders’ experience  Discovers information from documents  Stakeholders may not know about this information  Discovers information without stakeholders or documents  Prototyping or Proof of concept Collaborative Research Experiments
  • 154. CONDUCT ELICITATION ELEMENTS Guide the elicitation session Capture elicitation results Engage someone else to scribe
  • 155. CONDUCT ELICITATION GUIDELINES AND TOOLS Business Analysis Approach Existing Business Analysis Information Supporting Materials Stakeholder Engagement Approach
  • 156. CONDUCT ELICITATION (contd.) GUIDELINES AND TOOLS Influences elicitation techniques and business analysis deliverables Guides questions to be asked during elicitation Prepares the business analyst, participants and equipment for elicitation Provides collaboration and communication approach for elicitation Business Analysis Approach Existing Business Analysis Information Supporting Materials Stakeholder Engagement Approach
  • 157. CONDUCT ELICITATION TECHNIQUES Benchmarking and Market Analysis Document Analysis Observation Brainstorming Collaborative Games Workshops Focus Groups Survey and Questionnaire Interface Analysis Interviews Mind Mapping Prototyping Data Mining Concept Modeling Process Modeling Data Modeling Business Rules Analysis Process Analysis
  • 158. CONDUCT ELICITATION DOCUMENT ANALYSIS - OVERVIEW Limitations Strengths  Elicit business analysis information  Understand the context of a business need  Understand the solution currently being implemented  Validate findings from other elicitation efforts  Engage effectively with stakeholders  Wide range of sources make it time consuming  Outdated documents may cause confusion
  • 159. CONDUCT ELICITATION DOCUMENT ANALYSIS - ELEMENTS Preparation Document Review and Analysis Record Findings  ☑ Source document: • Detailed review and recording notes • Identify conflicting or duplicated notes Source document: • Relevant, current and has data or information required • Appropriate level of detail • Present information to improve understanding
  • 160. CONDUCT ELICITATION OBSERVATION - OVERVIEW Limitations Strengths  View and understand activities  Job shadowing  Approaches:  Active / Noticeable  Passive / Unnoticeable  Gain Realistic and practical insights  May be disruptive to the day-to- day operations  Participants may alter their work practices  Unsuitable for knowledge based activities
  • 161. CONDUCT ELICITATION OBSERVATION - ELEMENTS Define the objectives of the observation Prepare for observation Conduct observation session Confirm and present observation session findings  ☑
  • 162. CONDUCT ELICITATION BRAINSTORMING - OVERVIEW Strengths  Produce numerous ideas and a diverse set of options in a short span of time  Focus on the topic or problem  Encourage participants to use new ways  Foster creativity with a non- judgmental environment Limitations  Effectiveness depends on:  Individual creativity  Willingness to participate
  • 163. CONDUCT ELICITATION BRAINSTORMING - ELEMENTS Preparation Session Wrap-Up  Define area of interest  Determine time limit  Identify participants  Establish evaluation criteria  Re-iterate session details  Share ideas  Record ideas  Build on each other ideas  Elicit many ideas  Discuss and evaluate  Create list  Rate ideas  Distribute final list 📋
  • 164. CONDUCT ELICITATION COLLABORATIVE GAMES - OVERVIEW Strengths Limitations  Encourages collaboration in building common understanding  Follows adaptive or agile methodology  Sets rules to focus on objectives  Game environment may make some participants uncomfortable  Time consuming  Unproductive if objectives are unclear
  • 165. CONDUCT ELICITATION COLLABORATIVE GAMES - ELEMENTS Purpose Process Outcome Opening Exploration Closing Learn Rules Generate ideas Assess and Select idea ?   🖝 🞿 
  • 166. CONDUCT ELICITATION (contd.) COLLABORATIVE GAMES - ELEMENTS Product Product Box Identifyfeaturesofaproduct Affinity Map Identifyrelatedorsimilarfeatures Fishbowl Identifyhiddenassumptions
  • 167. CONDUCT ELICITATION WORKSHOPS - OVERVIEW Strengths  Most common technique  Focused event  Collaboration to achieve predefined goals  Make decisions and gain mutual understanding  Requires short period of time  Requires low cost Limitations  Availability of stakeholders difficult  Success depends on facilitator’s knowledge of participants  Too many participants slows down the process  Too few participants can lead to overlooking of needs
  • 168. CONDUCT ELICITATION WORKSHOPS - ELEMENTS Prepare Identify roles Conduct Post workshop Distribute outcome o Sponsor o Facilitator o Scribe o Timekeeper o Participants ?
  • 169. CONDUCT ELICITATION FOCUS GROUPS - OVERVIEW Strengths  Interactive group environment  Pre-qualified participants  Homogenous or Heterogeneous group  Qualitative research  Cost effective Limitations  Trust issues in participants  More vocal participants may influence the result
  • 170. CONDUCT ELICITATION FOCUS GROUPS- ELEMENTS Define objectives Plan Prepare guide Assign moderator and recorder Conduct post-discussion activities o Location o Budget o Timelines o Outcomes o Participants ? After the focus group – Transcribe the results as soon as possible
  • 171. CONDUCT ELICITATION SURVEY AND QUESTIONNAIRE - OVERVIEW Limitations Strengths  Distribution, collection and analysis  Elicit business information about customer, product, work practices and attitudes in a structured way and in a short period of time  Quick and relatively inexpensive to administer  Effective and efficient across geographies  Closed and open ended questions  Response may be low for statistical significance  Ambiguous questions not answered  Open-ended questions need more analysis
  • 172. CONDUCT ELICITATION SURVEYS AND QUESTIONNAIRES - ELEMENTS Prepare the survey or questionnaire Define Select Assign moderator and recorder Conduct and Post discussion activities o Objective o Target group o Appropriate survey type o Sample group o Distribution and collection method o Set target level and response time o Determine if individual interviews are required        
  • 173. CONDUCT ELICITATION PROTOTYPING - OVERVIEW Strengths  Iterative process  Creation of a model or design requirements  Optimizes user experience to evaluate design options  Basis of the final business solution  Identifies missing and incorrect requirements  Early inputs and feedback Limitations  Stakeholders may develop unrealistic expectations  Stakeholder may focus on the design of the solution  Focus of discussions may be on how are we going to build rather than what  Underlying technology needs to be understood
  • 174. CONDUCT ELICITATION PROTOTYPING - ELEMENTS Determine Approach Examples Methods Throw-away prototypes are discarded once the learning from the developed prototype and purpose are achieved. Evolutionary prototype is incrementally built to the final solution. Proof of Concept Usability prototype Visual prototype Functional prototype Storyboarding Paper prototyping Workflow modeling Simulation
  • 175. CONDUCT ELICITATION INTERFACE ANALYSIS - OVERVIEW Where What Why When How An Interface is a connection between two components or solutions. Most solutions require one or more interfaces. ? ? ? ? ? Identify
  • 176. CONDUCT ELICITATION (contd.) INTERFACE ANALYSIS - OVERVIEW ATM Laptop Mobile • External interface to the solution • Business processes • Data interface between systems • Application Programming Interfaces (APIs) • Hardware devices
  • 177. CONDUCT ELICITATION (contd.) INTERFACE ANALYSIS - OVERVIEW Understand interfaces used in existing business processes Provide context for eliciting more detailed functional requirements Use scope modelling, observation and interviews Provide interface of the solution with a context diagram
  • 178. CONDUCT ELICITATION (contd.) INTERFACE ANALYSIS - ELEMENTS Prepare for identification Conduct identification Define  Name  Coverage  Exchange method  Message format  Exchange frequency ?
  • 179. CONDUCT ELICITATION INTERVIEWS - OVERVIEW Definition Types Success A systematic approach to elicit business analysis information by asking relevant questions and documenting the responses from a person or a group of people. One-on-one interviews are the most common. Interviews can be synchronous, asynchronous, conducted face-to-face or through video conferencing. • Structured interview - Questions are pre-defined • Unstructured interview - Questions may vary based on the interviewee response • Interviewers - Domain knowledge, experience and skills for documenting the discussion • Interviewees - Readiness, degree of clarity about the goal of the interview • Rapport of the interviewer with the interviewee Constraint Significant time is required to plan and conduct interviews.
  • 180. CONDUCT ELICITATION INTERVIEWS - ELEMENTS Follow-up Plan Execute Define goals Communicate goals Identify interviewees Design questions Questions: 5W + 1H Open-ended questions Closed-ended questions Arrange logistics Flow: Open – During- Close Organize and confirm results 
  • 181. CONDUCT ELICITATION MIND MAPPING - OVERVIEW Used to Articulate MIND MAPPING Note taking T axonom y Capture ideas Main Topic T opic Sub-Topic Branches Color Images Non-linear Structure and logic
  • 184.  Overview  Elements  GD uie dfe elc in te p s ea r n U d ni T tools  Techniques  Stakeholders Lesson 4: Elicitation and Collaboration Topic4.3:ConfirmElicitationResults
  • 185. CONFIRM ELICITATION RESULTS OVERVIEW Purpose  Check gathered information  Confirm accuracy and consistency with other information I N P U T O U T P U T  Elicitation Results (Unconfirmed)  Elicitation Results (Confirmed)
  • 186. CONFIRM ELICITATION RESULTS ELEMENTS Confirmed elicitation results With other elicitation results Against source information   ☑
  • 187. CONFIRM ELICITATION RESULTS GUIDELINES AND TOOLS Elicitation Activity Plan Existing Business Analysis
  • 188. CONFIRM ELICITATION RESULTS TECHNIQUES Document Analysis  Reviews Interviews Workshops ☑
  • 189. CONFIRM ELICITATION RESULTS (contd.) TECHNIQUES Document Analysis Reviews Interviews Workshops • Confirm elicitation results with existing documents • Conduct review of elicitation results and solicit feedback  ☑
  • 191.  Overview  Elements  GD uie dfe elc in te p s ea r n U d ni T tools  Techniques  Stakeholders Lesson 4: Elicitation and Collaboration Topic4.4:CommunicateBusinessAnalysisInformation
  • 192. COMMUNICATE BUSINESS ANALYSIS INFORMATION OVERVIEW Purpose  Stakeholders have a shared understanding of business analysis information I N P U T S O U T P U T  Business Analysis Information  Stakeholder Engagement Approach  Business Analysis Information (communicated)
  • 193. COMMUNICATE BUSINESS ANALYSIS INFORMATION ELEMENTS Form of Package Depends on the business analysis approach  Predictive  Adaptive May be:  Formal documentation  Information documentation  Presentation ☑   Communicate business analysis package Determine objectives and format of communication 
  • 194.  COMMUNICATE BUSINESS ANALYSIS INFORMATION GUIDELINES AND TOOLS Business Analysis Approach Information Management
  • 195. COMMUNICATE BUSINESS ANALYSIS INFORMATION TECHNIQUES Interviews Reviews Workshops ☑
  • 196. COMMUNICATE BUSINESS ANALYSIS INFORMATION (contd.) TECHNIQUES Communicate information Review business analysis information Build consensus and seek approval Interviews Reviews Workshops ☑
  • 197. COMMUNICATE BUSINESS ANALYSIS INFORMATION STAKEHOLDERS End User Customer T ester Domain SME Implementation SME Common Understanding All Stakeholders
  • 198.  Overview  Elements  GD uie dfe elc in te p s ea r n U d ni T tools  Techniques  Stakeholders Lesson 4: Elicitation and Collaboration Topic4.5:ManageStakeholderCollaboration
  • 199. MANAGE STAKEHOLDER COLLABORATION OVERVIEW Purpose  Encourage stakeholders to work towards a common goal I N P U T S O U T P U T  Stakeholder Engagement Approach  Business Analysis Performance Assessment  Stakeholder engagement
  • 200. MANAGE STAKEHOLDER COLLABORATION ELEMENTS ☑ Gain agreements on commitments Monitor Stakeholder Engagement  Collaboration  Monitor the participation and performance of stakeholders  Monitor for risk related to stakeholder engagements  Maintain the free flow of information  Promote shared effort to resolve problems  Achieve desired results  Time commitment  Resource commitment
  • 201. MANAGE STAKEHOLDER COLLABORATION GUIDELINES AND TOOLS Business Analysis Approach Business Objectives Recommended Actions Future State Description Risk Analysis  
  • 202. MANAGE STAKEHOLDER COLLABORATION (contd.) GUIDELINES AND TOOLS Describes the level of engagement Business Analysis Approach Business Objectives Recommended Actions Future State Description Provides action to earn the support and focus of stakeholder Defines the desired future state and expected value Provides action to ensure stakeholder collaboration Provides directions towards future state Risk Analysis  
  • 203. MANAGE STAKEHOLDER COLLABORATION TECHNIQUES Collaborative Games Lessons Learned Risk Analysis and Management Stakeholder list, Map or Personas
  • 204. MANAGE STAKEHOLDER COLLABORATION (contd.) TECHNIQUES T eamwork and collaboration Stakeholder satisfaction or dissatisfaction Stakeholder participation and engagement risk Who needs to be involved? Collaborative Games Lessons Learned Risk Analysis and Management Stakeholder list, Map or Personas
  • 205. MANAGE STAKEHOLDER COLLABORATION STAKEHOLDERS All Stakeholders involved during change Sponsor Customer End User Implementation SMEs Domain SMEs Other Stakeholder
  • 206.  Defect per Unit Lesson 4: ELICITATION AND COLLABORATION CASESTUDYEXERCISE
  • 207. CASE STUDY PROBLEM STATEMENT BATONICS Pharma Company Unable to deliver products on time Customers switching to competitor products
  • 208. CASE STUDY ACTIVITIES The business analyst performed the following activities:  Spentsometimegoing throughexistingdocuments Createdahighlevelcontext diagram Createdalistofquestions forstakeholdersto understandtheirneeds Scheduledacoupleof meetingswithstakeholders  Documentedall thecollected informationandorganizedandanalyzed theinformation
  • 209. CASE STUDY EXERCISE Questions Response 1 Which technique has Paul used to create supporting material like a list of questions, As-Is process or any other? o Document Analysis o Interviews 2 What Business Analysis Approach is used to build the proposed system? o Adaptive o Restrictive 3 Which elicitation technique is used to design screens to capture customer information? o Prototyping o Group Discussion 4 Which elicitation technique is used to understand the flow of information between systems and users? o Interface Analysis o Workshops
  • 210. CASE STUDY EXERCISE Questions Response 5 Who is the end user of the proposed system? o Field Sales Agents o Underwriters 6 Which technique is used to understand how stakeholders are interacting in the process to complete their tasks or achieve goals? o Process Analysis o Customer Analysis 7 Who will be using the saved information of the customer and the quote? o Underwriters o Stakeholder
  • 211. CASE STUDY ANSWERS Questions Answers 1 Which technique has Paul used to create supporting material like a list of questions, As-Is process or any other etc.? Document Analysis 2 What Business Analysis Approach is used to build the a proposed system? Adaptive 3 Which elicitation technique is used to design screens to capture customer information? Prototyping 4 Which elicitation technique is used to understand the flow of information between systems and users? Interface Analysis 5 Who is the end user of the proposed system? Field Sales Agents 6 Which technique is used to understand how stakeholders are interacting in the process to complete their tasks or achieve goals? Process Analysis 7 Who will be using the saved information of the customer and the quote? Underwriters
  • 212. KEY TAKEAWAYS Elicitation is defined as “to draw forth or bring out” information. There are 18 commonly used Elicitation techniques. There are 3 tasks associated with Elicitation, 1 task with Communicate Business Analysis Information, and 1 task with Manage Stakeholder Collaboration. Select the appropriate technique or set of techniques based on the need and Business Analysis Approach. Measure performance of Elicitation activities throughout the project. Each Elicitation technique has the following tasks - prepare, execute or conduct, and wrap-up or close tasks. Frequently communicate Business Analysis Information and Manage Stakeholder Collaboration and communication during Elicitation activities.
  • 213. Quiz
  • 214. a. b. c. d. In which elicitation technique is Discussion Guide used? QUIZ 1 Workshops Brainstorming Focus Groups Interviews
  • 215. The correct answer is b. Explanation: Discussion Guide is used in Focus Group Sessions. Discussion Guide contains session objectives, topics for discussion, and scripts of specific questions. a. b. c. d. In which elicitation technique is Discussion Guide used? QUIZ 1 Interviews Focus Groups Brainstorming Workshops
  • 216. a. b. c. d. Which of the following elicitation techniques uses survey distribution and response collection? QUIZ 2 Interviews Surveys and Questionnaires Concept Modeling Workshop
  • 217. The correct answer is c. Surveys and Questionnaires is a technique in which first the surveys are distributed and then responses are collected for further analysis. a. b. c. d. Which of the following elicitation technique uses survey distribution and response collection? QUIZ 2 Workshop Concept Modeling Surveys and Questionnaires Interviews
  • 218. a. b. c. d. Which one of the following is not an output element of the task ‘Prepare for elicitation’? QUIZ 3 Elicitation results Elicitation technique Participant list Supporting material
  • 219. The correct answer is d. Explanation: Elicitation results is not an output element of the task ‘Prepare for elicitation’. . a. b. c. d. Which one of the following is not an output element of the task ‘Prepare for elicitation’? QUIZ 3 Supporting material Participant list Elicitation technique Elicitation results
  • 220. a. b. c. d. Which one of the following is not a common type of elicitation? QUIZ 4 Exploratory Experiments Research Collaborative
  • 221. The correct answer is d. Explanation: Exploratory is not a common type of elicitation. As per BABOK TM, there are three common types of elicitation – Collaborative, Research, and Experiments. a. b. c. d. Which one of the following is not a common type of elicitation? QUIZ 4 Collaborative Research Experiments Exploratory
  • 222. This concludes 0Elicitation and Collaboration” Thenextlessonis“RequirementsLifeCycleManagement”
  • 224. Lesson5- RequirementsLifeCycleManagement CBAP®Exam Preparation Course CBAP, CCBA and BABOK are registered certification marks owned by International Institute of Business Analysis.
  • 225. WHAT’S IN IT FOR ME Trace requirements Maintain requirements Prioritize requirements Assess changes to requirements Approve requirements
  • 226. INTRODUCTION Business Analyst  📋    Requirements Information • At start: The status of elicitation results is unconfirmed • Confirm elicitation results: The results are confirmed • Update status of elicitation results i.e. requirements or designs:All other knowledge areas Elicitation and Collaboration • Know the origin of the requirement or design. • Know whether one requirement is related or has dependency with other requirements. • Know which solution components fulfill the requirements. • Know which test cases will verify the functionality of the solution. • Know what will help you in assessing changes to the requirements. To update Elicitation results status
  • 227. INTRODUCTION Business Analyst Trace requirements Prioritize requirements Reuse requirements Assess requirements Approve requirements Allocate requirements Changes are inevitable 🞿 • The requirements need to be maintained throughout the project life cycles. CThhaengresuairreeminenvtitsanbelee,dhteonbcey m o a u i n n t e a e i n d e t d o t a h s r s o e u s g s h c o h u a t n t g h e e sp t o r o t jh eec t r el i q f eu c i ry ec m l ees n .ts and design, and recommend appropriate actions • C C h h a a n n g g e e s s t o t o t h t h e e r e r e q q u u i r i e r e m m e e n n t s t s a a n n d d t h t h e e a a p p p p r o r o v v a a llppr or occeessd d e e p p e e n n d d u u p p o o n n t h t h e e p p r o r o j e j e c c t t m m e e t h t h o o d d o o l o l o g g y y 🞿
  • 228. INTRODUCTION Business Analyst Maintain requirements Trace requirements Prioritize requirements Approve requirements The Requirements Life Cycle Management knowledge area describes the tasks that the business analysts performs in order to manage and maintain the requirements and design information from the point of inception till retirement.
  • 229. REQUIREMENTS LIFE CYCLE MANAGEMENT OVERVIEW Change Solution Need Stakeholder Context Value
  • 230. REQUIREMENTS LIFE CYCLE MANAGEMENT OVERVIEW Maintain requirements for reuse Support tracking and prioritization activities Work closely with key stakeholders Trace requirements and designs to solution components Trace, prioritize and maintain requirements Evaluate requirements and designs Change Solution Need Stakeholder Context Value
  • 231. REQUIREMENTS LIFE CYCLE MANAGEMENT TASKS AND OUTPUT Tasks Trace requirements Maintain requirements Prioritize requirements Assess requirements changes Approve requirements
  • 232. ELICITATION AND COLLABORATION KNOWLEDGE AREA TASKS AND OUTPUT Inputs • Requirements • Designs Tasks Trace requirements Maintain requirements Prioritize requirements Assess requirements changes Approve requirements Outputs Requirements • Traced, maintained, and prioritized Designs • Traced, maintained, and prioritized
  • 233. ELICITATION AND COLLABORATION KNOWLEDGE AREA TASKS AND OUTPUT Inputs • Requirements • Business Analysis Information • Proposed changes Tasks Trace requirements Maintain requirements Prioritize requirements Assess requirements changes Approve requirements Outputs • Requirements o Traced, maintained, and prioritized • Designs o Traced, maintained, and prioritized • Requirements Change Assessment • Design Change Assessment
  • 234. ELICITATION AND COLLABORATION KNOWLEDGE AREA TASKS AND OUTPUT Inputs • Requirements • Business Analysis Information • Proposed changes Input from other Knowledge Areas • Requirements (verified) Tasks Trace requirements Maintain requirements Prioritize requirements Assess requirements changes Approve requirements Outputs Requirements • Traced, maintained, and prioritized Designs • Traced, maintained, and prioritized • Requirements Change Assessment • Design Change Assessment • Requirements approved • Design approved
  • 235. What is traceability?  Overview  Elements  GD uie dfe e lc in te p s ea r n U d ni T tools  Techniques  Stakeholders Lesson 5: Requirements Life Cycle Management Topic5.1:TraceRequirements
  • 236. TRACE REQUIREMENTS OVERVIEW Purpose  Ensure requirements and designs at different levels are aligned to one another  Manage effects of change I N P U T S O U T P U T S  Requirements  Designs  Traced requirements  Traced designs
  • 237. TRACE REQUIREMENTS ELEMENTS Low level Origin High level Backward traceability Forward traceability Consider: • Value that each traceability link delivers • Use of specific relationship created ! Effort increases with: • Number of requirements and • Level of formality
  • 238. TRACE REQUIREMENTS ELEMENTS Low level Origin High level Backward traceability Forward traceability Consider: • Value that each traceability link delivers • Use of specific relationship created ! Effort increases with: • Number of requirements and • Level of formality Validate Satisfy   Depend One requirement is dependent on another requirement Derive 🖝 Implementation of test cases Solution components are implemented One requirement is derived from another requirement
  • 239. TRACE REQUIREMENTS GUIDELINES AND TOOLS    Legal or regulatory information Domain knowledge Information management approach Requirements management tool or repository
  • 240. TRACE REQUIREMENTS TECHNIQUES Business rules analysis Functional decomposition Scope modelling Process modelling
  • 241. TRACE REQUIREMENTS TECHNIQUES Business rules analysis Functional decomposition Scope modelling Process modelling Decompose solution scope into smaller components Trace the requirements to the business rules Show requirements tracing to future state process Show the scope of the business analysis work
  • 242. TRACE REQUIREMENTS STAKEHOLDERS Business Analyst Domain SME Implementation SME Operational support team Sponsor End user Supplier T ester Customers Project Manager
  • 243. What is requirements maintenance?  Overview  Elements  GD uie dfe e lc in te p s ea r n U d ni T tools  Techniques  Stakeholders Lesson 5: Requirements Life Cycle Management Topic5.2:MaintainRequirements
  • 244. MAINTAIN REQUIREMENTS OVERVIEW Purpose  Maintain accuracy and consistency of requirements  Support reuse of requirements I N P U T S O U T P U T S  Requirements  Designs  Maintained requirements  Maintained designs
  • 245. MAINTAIN REQUIREMENTS ELEMENTS Correctness, accuracy, relationship etc. Maintainattributes Source, category, requested by, created date, updated date, priority, complexity, status, target date, reusability etc. Reusedrequirements Clearly label, define, and store   Maintainrequirements
  • 246. MAINTAIN REQUIREMENTS GUIDELINES AND TOOLS  Information management approach Business Analyst Manage requirements for reuse ☑
  • 247. MAINTAIN REQUIREMENTS TECHNIQUES Business rules analysis Data flow diagrams Data modeling Process modeling Use cases and scenarios User stories Document analysis Functional decomposition
  • 248. MAINTAIN REQUIREMENTS TECHNIQUES Business rules analysis Data flow diagrams Data modeling Process modeling Use cases and scenarios User stories Document analysis Functional decomposition Identify which business rules, information flows, data structures, processes and requirements are reusable. Analyze existing document to provide a basis for maintaining and reusing requirements Identify requirements associated with components and available for reuse
  • 250. What is prioritizing requirements?  Overview  Elements  GD uie dfe e lc in te p s ea r n U d ni T tools  Techniques  Stakeholders Lesson 5: Requirements Life Cycle Management Topic5.3:PrioritizeRequirements
  • 251. PRIORITIZE REQUIREMENTS OVERVIEW Purpose  To rank requirements in the order of relative importance to maximize value delivery I N P U T S O U T P U T S  Requirements  Designs  Prioritized requirements  Prioritized designs
  • 252. PRIORITIZE REQUIREMENTS ELEMENTS Factors Challenges Process ?   Benefit Dependencies Time sensitivity Stability Regulatory or policy compliance Risk Cost Penalty Ongoing Relative value Stakeholders
  • 253. PRIORITIZE REQUIREMENTS GUIDELINES AND TOOLS Repository Solution scope Business constraints Change strategy Domain knowledge Governance approach Requirements architecture Requirements Management Tools / 
  • 254. PRIORITIZE REQUIREMENTS TECHNIQUES Backlog management Business cases Decision analysis Estimation Interviews Workshops Risk analysis and management Item tracking Financial analysis Prioritization   ☑
  • 255. PRIORITIZE REQUIREMENTS TECHNIQUES Benefit Cost Penalty Risk Assign weights (1 to 9) to factors benefit, penalty, cost and risk associated to a feature. Derive the total value using the formula as defined by Karl Weigers in relative weighting process.
  • 256. PRIORITIZE REQUIREMENTS BACKLOG MANAGEMENT - OVERVIEW Backlog  Record  Track  Prioritize remaining work items Items at the top have the highest business value and priority Backlog management • What work should be formally included • How to describe the work item • How to track the work items • How to periodically review and prioritize the work items in relation to all other items • How to select the work items to work on • How to remove the work items
  • 257. PRIORITIZE REQUIREMENTS BACKLOG MANAGEMENT - ELEMENTS Items in the backlog Prioritization Estimation Managing changes to backlog Use cases, user stories, change requests, defects, maintenance work records, documentation work records, functional requirements, non-functional requirements. New or changed requirements are identified and added to the backlog and ordered relative to the other items already there. Always emerging; Items are removed when they are completed or it is decided not to work on the item.
  • 258. PRIORITIZE REQUIREMENTS ESTIMATION - OVERVIEW  Forecast cost and effort Support decision making Iterative process Communicate constraints Communicate assumptions
  • 259. PRIORITIZE REQUIREMENTS ESTIMATION - ELEMENTS T op down Bottom up Parametric estimation Rough order of estimation (ROM) Rolling wave Delphi Three-point estimation Program evaluation and review • Accuracy • Source • Precision • Reliable • Deterring
  • 260. PRIORITIZE REQUIREMENTS TOP DOWN Vs BOTTOM UP Analogous estimation and expert judgment Used when quick estimates are required with low level of accuracy Fast No need to decompose work in greater detail Less costly to create and Good for similar projects Low level of accuracy Requires expertise and historical data Three point, Parametric or Analogous estimation Estimate require higher level of accuracy Higher level of accuracy Better team buy-in and commitment Best for complex projects Provides more accurate baseline to track. Takes more time and incurs more expenses Work needs to be defined in great detail
  • 261. PRIORITIZE REQUIREMENTS THREE POINT AVERAGE AND PERT ESTIMATION Optimistic – Most likely – Pessimistic Optimistic – Best case Pessimistic – Worst case Triangular distribution: Estimate based on the three points, optimistic, most likely and pessimistic Final estimate is the average of all three estimates PERT = Program (or Project) Evaluation and Review Technique Uses weighted average of three points It’s more accurate than 3-point average PERT is also referred as a “Beta Distribution” OR “Double Triangular Distribution”
  • 262. PRIORITIZE REQUIREMENTS THREE POINT AVERAGE AND PERT ESTIMATION Optimistic: 3 days, Most Likely: 5 days and Pessimistic: 10 days Then the final estimate, with averaging formula is 6 days Applying weighted average formula the final estimate = 5.5 days Standard deviation = 1.167 Optimistic 3 Most likely Pessimistic 3 3 5 10 Optimistic 6 Pessimistic 4* Most likely Pessimistic 6 Optimistic THREE POINTAVERAGE PERT ESTIMATION
  • 263. PRIORITIZE REQUIREMENTS PARAMETRIC AND ROLLING WAVE ESTIMATION Uses a mathematical model. Extremely accurate Reduces overall risk Based on historical information Takes time, Requires expensive tools, Requires considerable experience Difficult for calculating “soft” costs Difficult for broadly defined projects Iterative estimation Rough Order of Magnitude (ROM) estimate for overall initiative or project. High level of accuracy - Activities performed are in the near term Low level of accuracy – Activities performed are in the longer duration This estimation approach is used in change driven plans or adaptive or agile plans, also when planning is iterative, and hence is referred to as rolling wave planning.
  • 264. PRIORITIZE REQUIREMENTS PRIORITIZATION - OVERVIEW Backlog management Provides a framework for the BusinessAnalyst: • To facilitate stakeholder decisions • To understand the relative importance of business analysis information Importance of business analysis information is based on: Value, Risk, Difficulty of implementation Business analyst revisit priorities when changes occur in the business environment, stakeholder needs
  • 265. PRIORITIZE REQUIREMENTS PRIORITIZATION - APPROACHES Grouping Ranking Business analysis information is classified into categories such as, high, medium and low priority. Business analysis information is ordered based on relative importance. Business analysis information is prioritized based on the amount of work a project team can deliver in given time or budget. Requirements are prioritized based on the consensus among stakeholders.  Time boxing or Budgeting Negotiation 🞿
  • 266. PRIORITIZE REQUIREMENTS ITEM TRACKING - OVERVIEW Backlog management • Item tracking is used to capture and assign responsibility for issues and stakeholder concerns • Viable stakeholder concern is classified into specific item types • Item types are Actions,Assumptions, Constraints, Dependencies, Defects, Enhancements, and Issues • Items are assigned to one or more stakeholders who are responsible for its resolution • Item tracking may be shared with stakeholders to ensure transparency and visibility into the status
  • 267. PRIORITIZE REQUIREMENTS ITEM TRACKING - ELEMENTS Item-identifier Summary or Description Category Type Date identified Identified by Impact Priority Resolution date Owner Resolver or assigned to Agreed strategy Status Resolution updates Escalation matrix
  • 268. PRIORITIZE REQUIREMENTS STAKEHOLDERS Customer Sponsor Regulator End user Implementation SMEs Project manager Business Analyst
  • 269. How changes to the requirements are assessed?  Overview  Elements  GD uie dfe e lc in te p s ea r n U d ni T tools  Techniques  Stakeholders Lesson 5: Requirements Life Cycle Management Topic5.4:AssessRequirementsChanges
  • 270. ASSESS REQUIREMENTS CHANGES OVERVIEW Purpose  Evaluate the implications of proposed changes to requirements and designs I N P U T S O U T P U T S  Proposed changes  Existing requirements  Existing designs  Requirements change assessment  Designs change assessment
  • 271. ASSESS REQUIREMENTS CHANGES ELEMENTS Predictive Approach Adaptive approach Assessment formality Impact analysis Review related requirements and components Assess the impact by considering: • Benefit • Cost • Schedule • Impact • Urgency Impact resolution • All proposed changes need to be approved, rejected or deferred • All impacts and resolutions are documented and communicated to the stakeholders.
  • 272. ASSESS REQUIREMENTS CHANGES GUIDELINES AND TOOLS  Domain knowledge Legal or regulatory information Solution scope Requirements architecture Governance approach Change strategy   
  • 273. ASSESS REQUIREMENTS CHANGES TECHNIQUES Business case Financial analysis Document analysis Business rules analysis Estimation Interface analysis Risk analysis and management Item tracking Workshops and interviews Decision analysis   ☑
  • 274. ASSESS REQUIREMENTS CHANGES STAKEHOLDERS Business Analyst Customer Domain SME End user Regulator Operational support Sponsor Project manager T ester
  • 275. How are requirements approved?  Overview  Elements  GD uie dfe e lc in te p s ea r n U d ni T tools  Techniques  Stakeholders Lesson 5: Requirements Life Cycle Management Topic5.5:ApproveRequirements
  • 276. APPROVE REQUIREMENTS OVERVIEW Purpose  To obtain agreement on requirements and designs  To obtain approval on requirements and designs I N P U T S O U T P U T S  Verified requirements  Verified designs  Approved requirements  Approved designs
  • 277. APPROVE REQUIREMENTS ELEMENTS Who need to be involved in decision making Who are authorized for sign-off Resolve conflicts, build consensus, make stakeholders understand Facilitate communication between stakeholders, they may have different view points and conflicting priority ☑Gain consensus Obtain approvals from the stakeholder Record the approval decisions Maintain change log ☑ ☑ ☑ Track and communicate approval Conflict and issue management Understand stakeholder roles
  • 278. APPROVE REQUIREMENTS GUIDELINES AND TOOLS Change Strategy Requirement Management Tools / Repository Governance approach Legal or Regulatory information   Solution scope 
  • 279. APPROVE REQUIREMENTS TECHNIQUES Acceptance and evaluation criteria Item tracking Decision analysis Reviews Workshops ☑
  • 280. APPROVE REQUIREMENTS ACCEPTANCE AND EVALUATION CRITERIA - OVERVIEW management Backlog Acceptance criteria – • Used to define the requirements, outcome or conditions that must be met in order to consider solution to be acceptable to the key stakeholders. • Minimum set of requirements that must be met. It’s typically used when only one possible solution is being evaluated. Evaluation criteria – • Used to assess a set of requirements in order to choose between multiple solutions. • May be cost, performance, usability, performance etc.
  • 281. APPROVE REQUIREMENTS ACCEPTANCE AND EVALUATION CRITERIA - OVERVIEW Strengths • All requirements with testable acceptance criteria • Agreement upon acceptance criteria required • Acceptance criteria is necessary in case of contractual obligations • Evaluation criteria assist in the delivery of potential value • Evaluation criteria helps in defining priorities Limitations • Acceptance criteria may express contractual obligation • Achieving agreement on evaluation criteria for different needs can be challenging
  • 282. APPROVE REQUIREMENTS ACCEPTANCE AND EVALUATION CRITERIA - ELEMENTS Value attributes Testability Measures Usability, security, reliability, scalability, performance, availability of specific feature, ability to perform specific operations etc. User acceptance testing  ☑  Continuous or discrete scale
  • 283. APPROVE REQUIREMENTS STAKEHOLDERS Business Analyst Customer Domain SME End user Regulator Operational support Sponsor Project manager T ester
  • 284. KEY TAKEAWAYS 5 Tasks in the Requirements Life Cycle Management Knowledge Area. Trace requirements and designs at different levels, to ensure alignment to one another. Maintain backward traceability and forward traceability. Prioritize requirements and design in the order of relative importance to maximize value delivery. Maintain requirements for reusability. Retain requirements accuracy and consistency throughout the entire requirements life cycle. Assess implications of proposed changes to requirements and designs. Obtain agreement on and approval of requirements and designs for business analysis work to continue solution construction. Analysis, specification, modelling, verification, validation, and communication of requirements are performed in other knowledge areas.
  • 285.  fect per Unit Lesson 5: Requirements Life Cycle Management CASESTUDYEXERCISE
  • 286. CASE STUDY PROBLEM STATEMENT  .....  ….  …. To deliver a solution for improving ‘customer connect’ that:  Reduces the time between the customers search for a policy and the field agent’s response  Ensures that the customer’s questions are clarified as they are searching for the policy actively (Hot Lead).
  • 289. CASE STUDY EXERCISE Questions Response 1 In the given case study, what is the approach for business analysis? Iterative Approach Incremental Approach Predictive Adaptive 2 What should Paul do when he identifies a couple of requirements, which can be used in future projects? Hold for the next projects Label and store for reusability Out of scope as it is general requirement None of the above 3 What can be the reason for the requirements not in scope getting approved and allocated to a solution component for implementation? Change control process was not effectively implemented Impact Analysis was not performed Missing traceability Stakeholder urgency
  • 290. CASE STUDY EXERCISE Questions Response 4 What can be the reason for missing to invite key stakeholders for the prioritization meeting, when they are required for providing approvals? Forgot to invite Prioritization approach was not adequately defined in the business analysis approach Prioritization approach was not adequately defined in the business analysis governance approach Prioritization approach was not adequately defined in the business analysis information management approach 5 When requirements are prioritized based on only value, what flaw does the approach have? No impact Must prioritize high value requirements Missed considering relationship with other requirements None of the above
  • 291. CASE STUDY ANSWERS Questions Answers 1 In the given case study, what is the approach for business analysis? Adaptive 2 What should Paul do when he identifies a couple of requirements, which can be used in future projects? Label and store for reusability 3 What can be the reason for the requirements not in scope getting approved and allocated to a solution component for implementation? Missing traceability 4 What can be the reason for missing to invite a key stakeholder, who is required for an approval into prioritization meeting? Prioritization approach was not adequately defined in the business analysis governance approach. 5 When requirements are prioritized based on only value, what flaw does the approach has? Missed considering relationship with other requirements.
  • 292. Quiz
  • 293. a. b. c. d. Which one of the following business analysis technique is not used when prioritizing requirements? QUIZ 1 Workshops Brainstorming Item Tracking Decision Analysis
  • 294. The correct answer is a, b and d. Explanation: Decision analysis, item tracking and workshops are the techniques used to prioritize requirements. a. b. c. d. Which one of the following business analysis techniques is used when prioritizing requirements? QUIZ 1 Decision Analysis Item Tracking Brainstorming Workshops
  • 295. a. b. c. d. What is Double Triangular Distribution? If most likely is 9, best case is 5 and worst case estimate is 11. QUIZ 2 1 8.33 8.66 8